Está en la página 1de 59

Natural Resources 1st Exam (2017-2018)

THE PUBLIC LAND ACT (COMMONWEALTH ACT 141)


Act classification,
delimitation,
Preliminaries
survey,
and disposition of
Governing Law
alienable lands of
the public
Land Governing Law
domain.
Lands of the Public Domain CA 141 (The Public Land
Act) which was enacted on
Policy Considerations
November 7, 1936
Timber and Mineral Lands Special Laws EDU. Exploration, Development and Utilization of Natural
Friar Lands & Governed by laws presently Resources shall be for the benefit of the Filipino People.
Private Lands Reverted to in force or which may
RP hereafter be enacted The DENR shall be primarily responsible for the
implementation of the foregoing policy. It shall be in charge
of carrying out the State‘s constitutional mandate to control
Historical Background and supervise the exploration, development, utilization and
conservation of the country‘s natural resources.
Before CA 141. ACT 926 and Act 2874.
Regalian Doctrine
Act 926 was passed in pursuance of the provisions of the
Philippine Bill of 1902. It governed the disposition of lands This is the same as Jura Regalia. All lands belong to the
of the public domain. It prescribed rules and regulations for State which is the source of any asserted right of ownership
the homesteading, selling, and leasing of portions of the to the land. All lands not appearing to be clearly of private
public domain. It also provided for the issuance of patents dominion presumptively belong to the State. Accordingly,
to certain native settlers upon public lands, for the public lands not shown to have been reclassified or
establishment of town sites and sale of lots therein, for the released as alienable agricultural land or alienated to a
completion of imperfect titles, and for the cancellation or private person by the State remain part of the inalienable
confirmation of Spanish Concessions and grants in the public domain.
island.
Except for agricultural lands, natural resources cannot be
Act 2874 was the second Public land Act. This was passed alienated. The exploration, development and utilization of
under the Jones law. It was more comprehensive in scope natural resources shall be under the full control and
but limited the exploitation of agricultural lands to Filipinos supervision of the State. The Regalian Doctrine reserved to
and Americans and citizens of other countries which gave the State all natural wealth that may be found in the
Filipinos the same privilege. bowels of the earth even if the land where the discovery is
made be private. Thus, once minerals are discovered in the
The Public Land Act, which compiled the then existing laws lands, whatever the use to which it is devoted, such use
on lands of the public domain, remains to this day the may be discontinued by the State to enable it to extract the
existing general law governing the classification and minerals therein in the exercise of its sovereign prerogative.
disposition of lands of the public domain other than timber
and mineral lands. The Regalian Doctrine is enshrined in Section 2 Article 12 of
the Constitution (1935, 1973 and 1987) on National
Law Charter Scope Economy and patrimony.
Act 926 Philippine Bill of Disposition of
First Public Land 1902 Lands of the Cases:
Act Public Domain
Act 2874 Jones Law More The Supreme Court stated that we adhere to the Regalian
Second Public comprehensive in Doctrine wherein all agricultural, timber and mineral lands
Land Act scope but limited are subject to the dominion of the State. Thus, before any
the exploitation of land may be classified as alienable land, there must be a
agricultural lands positive act from the government.
to Filipinos and
Americans and Sunbeam Convenience Food vs Court of Appeals
citizens of other (181 SCRA 443)
countries which Sunbeam Convenience Foods, Inc. is the recipient of a
gave Filipinos the Sales Patent issued by the Bureau of Lands over two
same privilege parcels of land in Bataan. An OCT was thereby issued. The
CA 141 1935 The general law Solicitor-General filed an action for reversion on the ground
The Public Land Constitution governing the that the lots were forest lands and therefore inalienable.

Notes of Cesnee Tan Page 1


Natural Resources 1st Exam (2017-2018)

CA ruled, upholding the Solicitor-General's contention. take part therein.

ISSUE: Whether or not land is alienable and disposable land As for the Heirs of Sandoval et.al.'s contention, they
of the State. asseverate that the land is not a public land as the
possessory information title in their name and of their
HELD: The SC affirmed. predecessors-in-interest, the pre-war certification appearing
in the Bureau of Archives, and the fact that the proceeding
Our adherence to the Regalian Doctrine subjects all of the registration was brought under the Torrens act which
agricultural, timber, and mineral lands to the dominion of presupposes an existing title to be confirmed, are all
the State. Thus, before any land may be declassified from evidences that the land is a private land.
the forest group and converted into alienable or disposable
land for agricultural purposes, there must be a positive act ISSUE: W/N the respondent's evidences can be considered
from the Government. Even rules on the confirmation of as proof that the lot 7454 is a private land.
imperfect titles do not apply unless and until the land
classified as forest land is released in an official RULING: NO. Under the Regalian Doctrine, all lands not
proclamation to that effect so that it may form part of the otherwise appearing to be clearly within private ownership
disposable agricultural lands of the public domain. are presumed to belong to the State. Hence it is that all
applicants in land registration proceedings have the burden
The mere fact that a title was issued by the Director of of overcoming the presumption that the land thus sought to
Lands does not confer any validity on such title if the be registered forms part of the public domain. Unless the
property covered by the title or patent is part of the public applicant succeeds in showing by clear and convincing
forest. evidence that the property involved was acquired by him or
his ancestors either by composition title from the Spanish
What is the presumption? The Supreme Court held that in Government or by possessory information title, or any other
the absence of proof that the property is privately owned, means for the proper acquisition of public lands, the
the presumption is that it belongs to the State. If there is property must be held to be part of the public domain. The
no showing that the land hand been classified as alienable applicant must present competent and persuasive proof to
before the title was issued, possession thereof no matter substantiate his claim; he may not rely on general
how lengthy cannot ripen into ownership. statements, or mere conclusions of law other than factual
evidence of possession and title.
Republic vs Sayo
(191 SCRA 71) In the proceeding at bar, it appears that the principal
FACTS: The case at bar started at 1961 when the spouses document relied upon and presented by the applicants for
Casiano and Luz Sandoval filed an application for a parcel registration, to prove the private character of the large tract
of land, Lot 7454 originally party of Santiago but had since of land subject of their application was a photocopy of a
then been transferred to Nueva Vizcaya. certification of the National Library. But, as this Court has
already had occasion to rule, that Spanish document cannot
The registration was opposed by Bayaua, Reyes, and the be considered a title to property, it not being one of the
Philippine Cacao and Farm Products. grants made during the Spanish regime, and obviously not
constituting primary evidence of ownership. It is an
The case went on until on 1981, 20 years after, the Heirs of inefficacious document on which to base any finding of the
Sandoval, Heirs of Bayaua, and the Bureau of Lands and private character of the land in question.
Bureau of Forest Development entered a compromise
agreement, which effectively distributed parts of lot 7454 It thus appears that the decision of the Registration Court a
among the aforesaid parties and the counsel of the Heirs of quo is based solely on the compromise agreement of the
Sandoval as attorney's fees. The compromise agreement parties. But that compromise agreement included private
was approved by the court and confirmed the title and persons who had not adduced any competent evidence of
ownership of the parties in accordance with its terms. their ownership over the land subject of the registration
proceeding. Portions of the land in controversy were
Having knowledge of the incident, the Solicitor General filed assigned to persons or entities who had presented nothing
a complaint before the court to annul the decision rendered whatever to prove their ownership of any part of the land.
by the court a quo for being void and made in excess of The assent of the Directors of Lands and Forest
jurisdiction or with grave abuse of discretion. Development to the compromise agreement did not and
could not supply the absence of evidence of title required of
The Solicitor General contended that the the Heirs of the private respondents.
Sandoval et. al. did not present any evidence to support
their claims of ownership or registration, nor did the As to the informacion posesoria invoked by the private
government agencies involve have a authority to enter into respondents, it should be pointed out that under the
the compromise agreement, and finally, that he was not Spanish Mortgage Law, it was considered a mode of
notified of the proceedings and so had not opportunity to acquiring title to public lands, subject to two (2) conditions:

Notes of Cesnee Tan Page 2


Natural Resources 1st Exam (2017-2018)

first, the inscription thereof in the Registry of Property, and Director of Lands vs Intermediate Appellate Court
second, actual, public, adverse, and uninterrupted (219 SCRA 108)
possession of the land for twenty (20) years (later reduced Facts: The land involved is an island known as Tambac
to ten [10] years); but where, as here, proof of fulfillment Island in Lingayen Gulf situated in the Municipality of Bani,
of these conditions is absent, the informacion posesoria Pangasinan. Pacific Farms, Inc. initially applied for
cannot be considered as anything more than prima facie registration the subject property under the Land
evidence of possession. Registration Act (Act No. 496). The Republic of the
Finally, it was error to disregard the Solicitor General in the Philippines, thru the Director of Lands opposed the
execution of the compromise agreement and its submission application alleging that Pacific Farms, Inc. does not
to the Court for approval. It is, after all, the Solicitor possess a fee simple title to the land nor did its
General, who is the principal counsel of the Government; predecessors possess the land for at least thirty (30) years
this is the reason for our holding that "Court orders and immediately preceding the filing of application. The
decisions sent to the fiscal, acting as agent of the Solicitor opposition likewise specifically alleged that the applicant is
General in land registration cases, are not binding until they a private corporation disqualified under the (1973) new
are actually received by the Solicitor General." Philippine Constitution from acquiring alienable lands of the
public domain citing Section 11, Article 14. The Director of
Seville vs National Development Company Forest Development also entered its opposition alleging that
(GR No. 129401, February 2, 2001) the land is within the unclassified public land and, hence,
FACTS: LSBDA (Leyte Sub-A Basin Development Authority) inalienable. Other private parties also filed their
was created by virtue of PD no. 625. In connection with oppositions, but were subsequently withdrawn. Pacific
this, a letter of instruction was executed authorizing LSBDA Farms, Inc. filed a manifestation-motion to change the
to purchase or acquire privately owned land. applicant from Pacific Farms, Inc. to J. Antonio Araneta.
In June 14, 1980, certain Calixto Yap sold to LSBDA a
464,920 square meter parcel of land. The LSBDA was able Issue:
to register it under its name through the approved WON the land known as the "Tambac Island" can be
miscellaneous sales application with the Bureau of Lands. subject of registration.
However in May 11, 1990, petitioner Seville filed a
complaint for recovery or real property, rentals and Held:
damages against LSBDA, contending that he acquired the No. The Court held that lands of the public domain are
property and its predecessor-in-interest for more than 30 classified under three main categories, namely: Mineral,
years and therefore they are the rightful owner of it. Forest and Disposable or Alienable Lands. Under the
The RTC rendered judgment in favor of the petitioner, when Commonwealth Constitution, only agricultural lands were
the case reached in the CA, the latter reversed the RTC allowed to be alienated. Their disposition was provided for
decision on the ground that petitioner‘s possession of the under Commonwealth Act No. 141 (Secs. 6-7), which states
said property had never ripen into ownership since it still that it is only the President, upon the recommendation of
remains under the inalienable land of public domain. the proper department head, who has the authority to
classify the lands of the public domain into alienable or
ISSUE: W/N LSBDA ‗s title to property valid? disposable, timber and mineral lands. Mineral and Timber
W/N petitioner‘s claim of ownership over the property or forest lands are not subject to private ownership unless
was correct? they are first reclassified as agricultural lands and so
released for alienation. In the absence of such
RULING: The SC ruled in favor of LSBDA. According to the classification, the land remains as unclassified land until
SC, Seville or the Ortega estate did not become the owner released therefrom and rendered open to disposition.
of the subject property even though they possessed the Courts have no authority to do so.
land for more than 30 years, because at the time they
occupied the said land, it is still part of inalienable land of This is in consonance with the Regalian doctrine that all
public domain. Until the State declares it to be alienable lands of the public domain belong to the State, and that the
and disposable, no matter how long they possessed it, it State is the source of any asserted right to ownership in
may not ripen into ownership. land and charged with the conservation of such patrimony.
- Here the petitioner failed to discharge the burden of Under the Regalian Doctrine, all lands not otherwise
proving in court that the subject property is already outside appearing to be clearly within private ownership are
the component of inalienable land of public domain when presumed to belong to the State. Hence, a positive act of
they occupied the same. the government is needed to declassify a forest land into
- LDBDA is a rightful owner of the said land because, alienable or disposable land for agricultural or other
it acquired ownership and title over it through Sales patent purposes.
application which was approved by the Bureau of Land, this The burden of proof in overcoming the presumption of
shows that the said property is no longer under the ambit state ownership of the lands of the public domain is on the
of inalienable public land. person applying for registration that the land subject of the
application is alienable or disposable.

Notes of Cesnee Tan Page 3


Natural Resources 1st Exam (2017-2018)

Since the subject property is still unclassified, whatever of Lands and ultimately the Secretary of Environment and
possession the applicant may have had and however long, Natural Resources. It is the prerogative of the President
cannot ripen into private ownership. The conversion of upon recommendation of the Secretary of the DENR. The
subject property does not automatically render the property Secretary is the alter ego of the President and has the
as alienable and disposable. mechanism to determine the specifics regarding the land.

Now, how about mining claims? Is it covered by the Republic vs Register of Deeds of Quezon
concept of Jura Regalia? In the case of United Paracale vs (224 SCRA 537)
Dela Rosa, the Supreme Court held that under the Regalian FACTS: Petitioner was awarded a 17-hectare parcel of land,
Doctrine, the State has the power to control mining claims by virtue of which he was issued an OCT.
as provided under PD 1214.
Through an investigation conducted by the Bureau of
United Paracale vs Dela Rosa Lands, it was found that the free patent acquired by
(221 SCRA 108) Petitioner was fraudulent. A case for falsification of public
Facts: United Paracale Mining Company, Inc. and Coco documents was filed by Petitioner and was acquitted of the
Grove, Inc., seek to set aside the Order of dismissal of the crime.
case they filed with the trial court for the ejectment of their
respective defendants from the mining claims which were Subsequently, the Solicitor-General filed a complaint against
allegedly privately owned by them having been located and Petitioner, praying for the declaration of nullity of the Free
perfected under the provisions of the Philippine Bill of 1902 Patent and the OCT.
and Act No. 624.
Petitioner's main contention was that the land in question
ISSUE: W/N PD 1214 constitutional (REQUIRING was no longer within the unclassified public forest land
ALLLOCATORS UNDER THE ACT OF CONGRESS OFJULY 1, because by the approval of his application for Free Patent
1902, AS AMENDED, TO SECURE MINING LEASE by the Bureau of Lands, the land was already alienable and
CONTRACTS UNDER THE PROVISIONS OFPRESIDENTIAL disposable public agricultural land. He also claimed that the
DECREE NO. 463 INVOLVING THEIRPATENTABLE MINING land was a small portion of Lot 5139, an area which had
CLAIMS) been declared disposable public land by the cadastral court.

RULING: Yes. As held in the case Santa Rosa Mining ISSUE: Whether or not the land is alienable and disposable
Company, Inc. vs. Leido, Jr. 11 thus:"(W)e hold that public land
Presidential Decree No. 1214 is not unconstitutional. ** It is
a valid exercise of the sovereign power of the State, as HELD: Under the Regalian Doctrine, all lands not otherwise
owner, over lands of the public domain, of which clearly appearing to be privately-owned are presumed to
petitioner's mining claims still form apart, and over the belong to the State. Forest lands, like mineral or timber
patrimony of the nation, of which mineral deposits are a lands which are public lands, are not subject to private
valuable asset. It may be underscored, in this connection, ownership unless they under the Constitution become
that the Decree does not cover all mining claims located private properties. In the absence of such classification, the
under the Phil. Bill of1902, but only those claims over which land remains unclassified public land until released
their locators had failed to obtain a patent. And even then, therefrom and rendered open to disposition.
such locators may still avail of the renewable twenty-five
year (25) lease prescribed by Pres. Decree No. 463, the The task of administering and disposing lands of the public
Mineral Development Resources Decree of 1974. domain belongs to the Director of Lands, and ultimately the
Secretary of Agriculture and Natural Resources.
Mere location does not mean absolute ownership over the Classification of public lands is, thus, an exclusive
affected land or the mining claim. It merely segregates the prerogative of the Executive Department, through the
located land or area from the public domain by barring Office of the President. Courts have no authority to do so.
other would-be locators from locating the same and Thus, in controversies involving the disposition of public
appropriating for themselves the minerals found therein. To agricultural lands, the burden of overcoming the
rule otherwise would imply that location is all that is needed presumption of state ownership of lands of the public
to acquire and maintain rights over a located mining claim. domain lies upon the private claimant.
This, we cannot approve or sanction because it is contrary
to the intention of the lawmaker that the locator should In the present case, Petitioner failed to present clear,
faithfully and consistently comply with the requirements for positive and absolute evidence to overcome said
annual work and improvements in the located mining claim. presumption and to support his claim.

Now, who has the task of administering lands and to whom Moreover, the fact the Petitioner acquired a title to the land
does it belong? The Director of Lands which is now the is of no moment, notwithstanding the indefeasibility of title
Land Management Bureau. The task of administering lands issued under the Torrens System. The indefeasibility of a
is purely an executive privilege that belongs to the Director certificate of title cannot be invoked by one who procured

Notes of Cesnee Tan Page 4


Natural Resources 1st Exam (2017-2018)

the same by means of fraud. we said: "Thus, before any land may be declassified from
the forest group and converted into alienable or disposable
What is the ratification if you occupy forest lands? Forest land for agricultural or other purposes, there must be a
lands are not capable of private appropriation. Possession positive act from the government. Even rules on the
thereof cannot ripen into ownership. confirmation of imperfect titles do not apply unless and
until the land classified as forest land is released in an
Ituralde vs Falcasantos official proclamation to that effect so that it may form part
(GR No. 128017, January 20, 1999) of the disposable agricultural lands of the public domain."
Facts: On October 17, 1986, Ramon Ituralde, herein Hence, a positive act of the government is needed to
petitioner, acquired by purchase from the heirs of Pedro declassify a forest land into alienable or disposable land for
Mana-ay a parcel of land, with an area of 6.0000 hectares, agricultural or other purposes. "
more or less. However, on November 3, 1986, respondent And the rule is "Possession of forest lands, however long,
Alfredo Falcasantos applied with the Bureau of Lands for cannot ripen into private ownership."
the award to him of the same parcel of land under free What is more, there is yet no award or grant to petitioner
patent, which application was protested by Ramon Ituralde. of the land in question by free patent or other ways of
acquisition of public land. Consequently, he cannot lawfully
On February 7, 1989, the Regional Director of Lands claim to be the owner of the land in question.
rendered a decision giving respondent a period of one
hundred twenty (120) days to exercise the right to How about minerals or wealth found in the land? Is it
repurchase the land by reimbursing petitioner of all subject to Jura Regalia? Even if it is registered in your
expenses he incurred in the purchase of the property in name, so it is already private property, if there are minerals
question, and held in abeyance respondent's application for found within that land, the State can discontinue the
free patent. Alfredo failed to exercise the right of private use thereof and convert or revert it back to the
repurchase, which amounted to a waiver. The Director of government. However, in the absence of fraud in your
Land then rejected Alfredo‘s application and allowed Ramon registration, you can demand payment of just
Ituralde to file a public land application for the subject land. compensation.
Meanwhile, in a case for the recovery of ownership and
possession filed by Ituralde against Alfredo, the trial court Republic vs. Court of Appeals and dela Rosa
rendered a decision declaring petitioner the owner and the (GR No. L-43938, 04/15/1988)
possessor of the subject parcel of land with all the FACTS: The case is about a parcel of land whose ownership
improvements existing thereon, with an area of 3.1248 is being disputed by four parties: the Dela Rosas, Benguet
hectares, and ordering respondent to vacate the land in Consolidated Inc. (BCI), Atok Corp. (Atok) and the Bureau
question. Ramon appealed to the decision. However the of Forestry Development (BFD).
Court of Appeals reversed the decision of the trial court and
dismissed Roman‘s appeal. Hence this case. Sometime in 1965, Jose dela Rosa, on his own behalf and
on behalf of this three children, filed an application for the
Issue: WON the Court of Appeals committed an error in registration of a parcel of land divided into 9 lots in
reversing the decision of the trial court and dismissing Benguet. According to the children, they had acquired the
petitioner‘s appeal. land by virtue of prescription. As evidenced, they produced
tax declarations and realty tax receipts.
Ruling: NO.
Petitioner argued that the Court of Appeals erred in setting BCI and Atok opposed their application claiming that the
aside the trial court's decision in his favor and dismissing lots were covered by mineral claim sold to it in 1934. BCI,
the complaint because when the Director of Lands allowed on lots 1-5, contented that it had been in actual,
petitioner to file a public land application for said property, continuous and exclusive possession of the land in the
it was equivalent to a declaration that said land was no concept of an owner. As evidence, BCI presented geological
longer part of the public domain. maps, payments of taxes and construction on the land.
We deny the petition. The Court of Appeals correctly held
that "the evidence is unrebutted that the subject land is Atok claims of ownership on a portion of lots 1-5 and all of
within the Forest Reserve Area as per L.C. Map No. 1557 lots 6-9 is similar to that of BCI, that a mineral claim
certified on August 13, 1951'." and, hence, not capable of covering that the lots had been sold to them in 1931 and
private appropriation and occupation. produced evidence such as the construction and tax
In Republic vs. Register of Deeds of Quezon, we held that payments.
"Forest land, like mineral timber lands which are public
lands, are not subject to private ownership unless they BFD objected that the land in dispute was covered by the
under the Constitution, become private properties. In the Central Cordillera Forest Reserve under Proc. No. 217. And
absence of such classification, the land remains unclassified since it was a forest land, it cannot be subject to alienation,
public land until released therefrom and rendered open to pursuant to the 1935 and 1973 Constitution.
disposition."
In Sunbeam Convenience Foods Inc. vs. Court of Appeals, The Lower Court denied the application, while the CA

Notes of Cesnee Tan Page 5


Natural Resources 1st Exam (2017-2018)

reversed the decision, affirming the surface rights of the


Dela Rosas, while also the sub-surface rights of the BCI and Heirs of Gozo v. Philippine Union Mission Corp. of
Atok. In other words, the CA ruled that the surface was to the Seventh Day Adventist Church,
be agricultural and mineral underneath. G.R. No. 195990, [August 5, 2015])
FACTS: Petitioners are heirs of Spouses Rafael and
ISSUE: Concepcion Gozo who possessed a property in Brgy. Lala,
1. Who has better claim on the land in dispute Kolambugan, Lanao del Norte of 236,638 sq.m.
2. WON land can be classified as both agricultural and
mineral at the same time. Respondent (PUMC-SDA) claimed that a portion of that
property was donated to it by spouses Gozo based on the
HELD: deed of donation dated 28 Feb. 1937.

1ST ISSUE: When spouses Gozo donated the property in 1937, they
were not yet the registered owners of the property
The Court held that BCI and Atok have vested rights over although they are the lawful possessors thereof.
the land. The Court found that the mineral claims of both
BCI and Atok have been perfected prior to the approval of In 1953, the land was registered under the name of
the 1935 Constitution. As a legal effect of a valid mineral spouses Gozo and an OCT was issued in their favor
claim, it segregates the area from the public domain and pursuant to Homestead Patent granted by the President of
confers to the locator the beneficial ownership of the claim. the Phil.
As of 1935, they were removed from the public domain and
had become private properties of BCI and Atok. Even if the After Rafael Gozo‘s dealth, his wife Concepcion and their 6
land was included in the Central Cordillera forest reserve, it children, herein petitioners, caused the extrajudicial
did not impair the rights vested in both mining companies. partition of the property. Thus, the Register of Deeds
The claim of the Dela Rosas was disregarded for weak issued a TCT under the names of the heirs/petitioners in
evidence, and even so, they could not have acquired the 1954.
land through prescription since the lad had already been
converted to mineral land. When Concepcion caused the survey and the subdivision, it
was at this point that respondent brought to the attention
2ND ISSUE: of Concepcion that 5,000 sq.m. portion of the property is
already owned by it based on the deed of donation. Upon
No. Land cannot be classified as half agricultural and half learning from the ROD that the donation was not annotated
mineral. The Court held that the classification of land must in the title, petitioners refused to recognize the donation.
be categorical: the land must either completely be mineral After Concepcion‘s death, the heirs/petitioners continued to
or agricultural. In this case, while the land was initially pursue their claims to recover the subject property from
classified as forest land, it ceases to be so and completely respondent.
became mineral when the mining claims were perfected.
Even if the surface was being tilled by the Dela Rosas, it is In 2000 or after 6 decades after the alleged deed of
still to be considered as mineral land. donation was executed, petitioners filed an action for
declaration of nullity of deed of donation and recovery of
As long as mining operations were being undertaken possession and ownership against respondent. Petitioners
thereon or underneath, it is still mineral, and not argued that the signatures of their parents in the deed are
agricultural. not genuine and the donation lacks the formality of
acceptance, hence invalid.
Hence, the Court ruled that BCI and Atok have exclusive
rights to the property in question by virtue of their Respondent argued that since 1937 they are in open,
respective mining claims which they validly acquired before continuous, exclusive and adverse possession of the
the 1935 Constitution which prohibits the alienation of the property in the concept of an owner, that they have built
lands of the public domain except agricultural lands. It therein a church and an elementary school and that the
could not have been transferred to private respondents by petitioners‘ claim of lack of acceptance 63 years after the
virtue of acquisitive prescription not could its use be shared deed was executed is already barred by laches.
simultaneously by them and the mining companies for
agricultural and mineral purposes. In 2004, the RTC rendered a decision in favor of petitioners
declaring that since the deed is void for lack of acceptance,
Prior to the issuance of patent, can a subsequent title in they are the rightful owners thereof. The Court of Appeals
their name transfer the property in favour of another? So reversed the RTC decision on the ground of laches stating
that was not allowed since they have no title yet over the that petitioners failed to assert their rights over the land for
property and therefore they cannot transfer it. Remember more than 60 years, thus laches had set in.
in your Civil Law, nemo dat quod non habet. No one gives
what he does not have. ISSUE:

Notes of Cesnee Tan Page 6


Natural Resources 1st Exam (2017-2018)

Whether or not the donor is necessarily the owner of the


real estate donated? On the basis of the order of award of the Director of Lands,
the Undersecretary of Agriculture and Natural Resources
RULING: issued Miscellaneous Sales Patent No. V-1209 pursuant to
which an OCT was issued by the Register of Deeds of Naga
No. The donors may be the possessor at the time of the City to Aniano David.
donation in 1937, but they are not the owners thereof as
the subject property was part of the public domain. It was Since the filing of the sales application of Aniano David and
only in 1953 that ownership of the property was vested by during all the proceedings in connection with said
the State to the spouses Gozo by virtue of its issuance of application, up to the actual issuance of the sales patent in
the OCT pursuant to the Homestead Patent granted by the his favor, Lee Hong Kok did not put up any opposition or
President of the Phil. Hence the donor is not necessarily adverse claim to Lot 2892.
the owner of the real property they donated when the said
property was still part of the public domain (inalienable). The opposition was fatal because after the registration and
The donors cannot give what they do not have, nemo dat issuance of the certificate and duplicate certificate of title
quod non habet. based on a public land patent, the land covered thereby
automatically comes under the operation of RA 496 subject
Imperium and Dominium to all the safeguards provided therein. Under Section 38 of
RA 496, any question concerning the validity of the
Imperium Dominium certificate of title based on fraud should be raised within 1
The State‘s authority to The capacity of the State to year from the date of the issuance of the patent otherwise
govern is embraced in the own and acquire property. It the certificate of title becomes indefeasible after the lapse
concept of sovereignty that covers such rights as title to of 1 year.
includes passing laws land, exploitation and use of
concerning a territory, it, and disposition or sale of The contention of Lee Hong Kok was that David‘s lot is a
maintaining peace and order the same. private property for it was formed thru the process of
over it, and defending it It refers to lands held by the accretion.
against foreign invasion. government in a proprietary
character: can provide for Issue:
the exploitation and use of Can the patent certificate of Aniano David to the property
lands and other natural be nullified?
resources.
When the State acts in this The Regalian Doctrine Held:
capacity, jure imperii, it whereby all lands of the No. There is no legal justification for nullifying the right to
generally enjoys state public domain belong to the the disputed lot arising from the grant made in his favor.
immunity. State, and anyone claiming The lot in question is NOT a private property as the Director
title has the burden to show of Lands and the Secretary of Agriculture and Natural
ownership, comes within Resources have always sustained its public character for
this concept. In this having been formed by reclamation and not accretion.
capacity, jure gestium, the
State descends to the status RECLAMATION is the act of filling submerged land by
of ordinary persons and thus deliberate acts and reclaiming title thereto.
becomes liable as such. ACCRETION - is the process whereby the soil is deposited;
accretion resulting from the gradual deposit by or
sedimentation from the waters belongs to the owners of
Lee Hong Kok vs. David the land bordering on streams, torrents, lakes, or rivers
(GR No.G.R. No. L-3038 12/27/1972)
Nature: Therefore the only remedy available to the appellants is an
Lee Hong Kok, et al filed an appeal on certiorari seeking to action for reconveyance on the ground of fraud.
reverse the decision of the Court of Appeals which affirmed
the decision of the lower court in dismissing the complaint Issue: Did Aniano David commit fraud?
to have the Torrens Title of Aniano David be declared null
and void. NO. Aniano David has not committed any fraud in applying
for the purchase of the Lot because everything was done in
Facts: Aniano David acquired lawful title to a parcel of land the open. The notices regarding the auction sale of the land
pursuant to his miscellaneous sales application. An order of were published, the actual sale and award thereof to
award and for issuance of a sales patent was made by the Aniano David were not clandestine but open and public
Director of Lands on June 18, 1958, covering Lot 2892 official acts of an officer of the Government. The
containing an area of 226 square meters, which is a portion application was merely a renewal of his deceased wife's
of Lot 2863 of the Naga Cadastre. application who had occupied the land since 1938.

Notes of Cesnee Tan Page 7


Natural Resources 1st Exam (2017-2018)

As far back as 1919, in Aquino v. Director of Lands, Justice


If the grant was presumed to be invalid, who has the right Malcolm, speaking for the Court, stated: "The proceedings
to question it? under the Land Registration Law and under the provisions
of Chapter VI of the Public Land Law are the same in that
Only the Government, represented by the Director of Lands both are against the whole world, both take the nature of
or the Secretary of Agriculture and Natural Resources can judicial proceedings, and for both the decree of registration
bring an action to cancel a void certificate of title pursuant issued is conclusive and final."
to a void patent.
The private parties cannot claim that the patent and title Meanwhile in Cabacug v. Lao. There is this revealing
issued for the land involved are void since they are not the excerpt appearing in that decision: "It is said, and with
registered owners thereof nor had they been declared as reason, that a holder of a land acquired under a free patent
owners in the cadastral proceedings of the Naga Cadastre is more favorably situated than that of an owner of
after claiming it as their private property. registered property. Not only does a free patent have a
Citing Maninang vs Consolacion: ‗The fact that the grant force and effect of a Torrens Title, but in addition the
was made by the government is undisputed. Whether the person to whom it is granted has likewise in his favor the
grant was in conformity with the law or not is a question right to repurchase within a period of five years." It is quite
which the government may raise, but until it is raised by apparent, therefore, that petitioners' stand is legally
the government and set aside, the defendant cannot indefensible.
question it. The legality of the grant is a question between
the grantee and the government. RULING:
The decision of the CA of January 31, 1969 and its
By what authority does the government have in disposing resolution of March 14, 1969 are affirmed.
of the land in question?
b. IPRA Law (Indigenous Peoples Act of 1997)
IMPERIUM refers to the government authority possessed by (RA 8371)
the state which is appropriately embraced in the concept of
sovereignty. DOMINIUM, on the other hand, is appropriate Under the IPRA Law, indigenous peoples may obtain the
with reference to lands held by the state in its proprietary recognition of their right of ownership over ancestral lands
character. In such capacity, it may provide for the and ancestral domains by virtue of native title.
exploitation and use of lands and other natural resources,
including their disposition, except as limited by the 1. Ancestral Lands and Ancestral Domains
Constitution.
Ancestral Lands Ancestral Domain
The manifestation of the concept of jura regalia, which was Small parcels of lands Broader, it includes all the
adopted by the present Constitution, was embodied in the granted to individual natural resources included
universal feudal theory that all lands were held from the indigenous people therein
Crown, the ownership however is vested in the state rather Lands occupied by Areas generally belonging to
than the head thereof. individuals, families, and indigenous cultural
clans who are members of communities.
As to the unappropriated public lands constituting the indigenous cultural
public domain, the sole power of legislation is vested in communities.
Congress. These lands are required to
have been occupied,
There being no evidence whatever that the property in possessed, and utilized
question was ever acquired by the applicants or their by them or their ancestors
ancestors either by composition title from the Spanish ―since time immemorial,
Government or by possessory information title or by any continuously to the present‖.
other means for the acquisition of public lands, the property
must be held to be public domain (Heirs of Datu Pendatun
v. Director of Lands). 2. Constitutionality of the IPRA LAW

―No public land can be acquired by private persons without


any grant, express or implied, from the government.‖ 3. Native Title: Exception to Jura Regalia
Therefore it is indispensable that there be a showing of a
title from the state or any other mode of acquisition Cases:
recognized by law otherwise the property is and remains
part of the public domain. IPRA law granted vast authority to the IPs which is in
contravention of the Regalian Doctrine. In this case, the
Was David‘s title to the property already indefeasible? Supreme Court was not able to rule on the substance of the
case as to whether or not the IPRA Law is unconstitutional.

Notes of Cesnee Tan Page 8


Natural Resources 1st Exam (2017-2018)

The petition was dismissed. However, in the separate rights to which the King has by virtue of his prerogatives
opinion of Justice Kapunan, he stated that jura regalia does > The theory of jure regalia was therefore nothing more
not negate a native title. than a natural fruit of conquest
CONNECTED TO THIS IS THE STATE‘S POWER OF
Opinion of Justice Kapunan in the case of Cruz vs Secreaty DOMINUUM
(2000): Regalian Theory does not negate native title to > Capacity of the state to own or acquire property—
lands held in private ownership since time immemorial. In foundation for the early Spanish decree embracing the
Cariño VS Insular Government (1907), when as far back as feudal theory of jura regalia
testimony or memory goes, the land has been held by > This concept was first introduced through the Laws of
individuals under the claim of private ownership, it will be the Indies and the Royal Cedulas
presumed to have been held in the same way from before > The Philippines passed to Spain by virtue of discovery
the Spanish conquest, and never to have been public land. and conquest. Consequently, all lands became the exclusive
patrimony and dominion of the Spanish Crown.
Cruz vs Secretary of DENR > The Law of the Indies was followed by the Ley
(GR No. 135385, 12/6/2000) Hipotecaria or the Mortgage Law of 1893. This law provided
Petitioners Isagani Cruz and Cesar Europa filed a suit for for the systematic registration of titles and deeds as well as
prohibition and mandamus as citizens and taxpayers, possessory claims
assailing the constitutionality of certain provisions of > The Maura Law: was partly an amendment and was the
Republic Act No. 8371, otherwise known as the Indigenous last Spanish land law promulgated in the Philippines, which
People‘s Rights Act of 1997 (IPRA) and its implementing required the adjustment or registration of all agricultural
rules and regulations (IRR). The petitioners assail certain lands, otherwise the lands shall revert to the State
provisions of the IPRA and its IRR on the ground that these
amount to an unlawful deprivation of the State‘s ownership In support of their theory that ancestral lands and ancestral
over lands of the public domain as well as minerals and domains are part of the public domain and, thus, owned by
other natural resources therein, in violation of the regalian the State, pursuant to Section 2, Article XII of the
doctrine embodied in section 2, Article XII of the Constitution, petitioners and the Solicitor General advance
Constitution. the following arguments:
ISSUE: • First, according to petitioners, the King of
Do the provisions of IPRA contravene the Constitution? Spain under international law acquired exclusive dominion
over the Philippines by virtue of discovery and conquest.
HELD: No, the provisions of IPRA do not contravene the They contend that the Spanish King under the theory of
Constitution. Examining the IPRA, there is nothing in the jura regalia, which was introduced into Philippine law upon
law that grants to the ICCs/IPs ownership over the natural Spanish conquest in 1521, acquired title to all the lands in
resources within their ancestral domain. Ownership over the archipelago.
the natural resources in the ancestral domains remains with • Second, petitioners and the Solicitor General
the State and the rights granted by the IPRA to the submit that ancestral lands and ancestral domains are
ICCs/IPs over the natural resources in their ancestral owned by the State. They invoke the theory of jura regalia
domains merely gives them, as owners and occupants of which impute to the State the ownership of all lands and
the land on which the resources are found, the right to the make the State the original source of all private titles. They
small scale utilization of these resources, and at the same argue that the Philippine State, as successor to Spain and
time, a priority in their large scale development and the United States, is the source of any asserted right of
exploitation. ownership in land.
• Third, petitioners and the Solicitor General
Additionally, ancestral lands and ancestral domains are not concede that the Cariño doctrine exists. However,
part of the lands of the public domain. They are private petitioners maintain that the doctrine merely states that
lands and belong to the ICCs/IPs by native title, which is a title to lands of the public domain may be acquired by
concept of private land title that existed irrespective of any prescription. The Solicitor General, for his part, argues that
royal grant from the State. However, the right of ownership the doctrine applies only to alienable lands of the public
and possession by the ICCs/IPs of their ancestral domains domain and, thus, cannot be extended to other lands of the
is a limited form of ownership and does not include the public domain such as forest or timber, mineral lands, and
right to alienate the same. national parks.
• Fourth, the Solicitor General asserts that even
SEPARATE OPINION; JUSTICE KAPUNAN assuming that native title over ancestral lands and ancestral
domains existed by virtue of the Cariño doctrine, such
JURA REGALIA native title was extinguished upon the ratification of the
• private title to land must be traced to some 1935 Constitution.
grant, express or implied, from the Spanish Crown or its • Fifth, petitioners admit that Congress is
successors, the American Colonial Government, and mandated under Section 5, Article XII of the Constitution to
thereafter, the Philippine Republic protect that rights of indigenous peoples to their ancestral
> In a broad sense, the term refers to royal rights, or those lands and ancestral domains. However, they contend that

Notes of Cesnee Tan Page 9


Natural Resources 1st Exam (2017-2018)

the mandate is subject to Section 2, Article XII and the records that the framers of the Constitution did not intend
theory of jura regalia embodied therein. According to Congress to decide whether ancestral domains shall be
petitioners, the recognition and protection under R.A. 8371 public or private property. Rather, they acknowledged that
of the right of ownership over ancestral lands and ancestral ancestral domains shall be treated as private property, and
domains is far in excess of the legislative power and that customary laws shall merely determine whether such
constitutional mandate of Congress. private ownership is by the entire indigenous cultural
• Finally, on the premise that ancestral lands and community, or by individuals, families, or clans within the
ancestral domains are owned by the State, petitioners posit community.
that R.A. 8371 violates Section 2, Article XII of the • The framers of the Constitution inserted the
Constitution which prohibits the alienation of non- phrase ―subject to the provisions of this Constitution‖ in the
agricultural lands of the public domain and other natural Section 2, Article XII of the Constitution mainly to prevent
resources. the impairment of Torrens titles and other prior rights in the
determination of what constitutes ancestral lands and
Ownership is the crux of the issue of whether the provisions ancestral domains.
of IPRA pertaining to ancestral lands, ancestral domains,
and natural resources are unconstitutional. The B. The provisions of R.A. 8371 do not infringe upon the
fundamental question is, who, between the State and the State‘s ownership over the natural resources within the
indigenous peoples, are the rightful owners of these ancestral domains.
properties? • Although under Section 3(a) of IPRA states the
inclusion of natural resources in the definition of ancestral
A. The provisions of IPRA recognizing the ownership of domains purportedly results in the abdication of State
indigenous peoples over the ancestral lands and ancestral ownership over these resources; however, Section 3(a)
domains are not unconstitutional. merely defines the coverage of ancestral domains, and
describes the extent, limit and composition of ancestral
Cariño vs. Insular Government domains by setting forth the standards and guidelines in
x x x Every presumption is and ought to be taken against determining whether a particular area is to be considered
the Government in a case like the present. It might, as part of and within the ancestral domains.
perhaps, be proper and sufficient to say that when, as far • In other words, Section 3(a) serves only as a
back as testimony or memory goes, the land has been held yardstick which points out what properties are within the
by individuals under a claim of private ownership, it will be ancestral domains. It does not confer or recognize any
presumed to have been held in the same way from before right of ownership over the natural resources to the
the Spanish conquest, and never to have been public land. indigenous peoples. Its purpose is definitional and not
x x x. declarative of a right or title.
The phrase ―private but community property‖ is merely
ICPs acquired their ancestral domain through "ownership descriptive of the indigenous peoples‘ concept of ownership
by virtue of native title" and not through "ownership of land as distinguished from that provided in the Civil Code.
by acquisitive prescription against the State" • In Civil Law, ―ownership‖ is the ―independent
• Ownership by virtue of native title presupposes and general power of a person over a thing for purposes
that the land has been held by its possessor and his recognized by law and within the limits established
predecessors-in-interest in the concept of an owner since thereby.‖ It has the following attributes:
time immemorial. The land is not acquired from the State, jus utendi or the right to receive from the
that is, Spain or its successors-in-interest, the United States thing that which it produces,
and the Philippine Government. There has been no transfer jus abutendi or the right to consume the thing
of title from the State as the land has been regarded as by its use,
private in character as far back as memory goes. jus disponendi or the power to alienate,
• Ownership of land by acquisitive prescription encumber, transform and even destroy that which is owned
against the State involves a conversion of the character of and
the property from alienable public land to private land, jus vidicandi or the right to exclude other
which presupposes a transfer of title from the State to a persons from the possession the thing owned.
private person. • In contrast, the indigenous peoples‘ concept of
Acquisition of Spain on certain parts of the Phil. through ownership emphasizes the importance of communal or
terra nullius (no man's land) group ownership. By virtue of the communal character of
• Although Spain was deemed to have acquired ownership, the property held in common ―cannot be sold,
sovereignty over the Philippines, this did not mean that it disposed or destroyed‖ because it was meant to benefit the
acquired title to all lands in the archipelago. By virtue of whole indigenous community and not merely the individual
the colonial laws of Spain, the Spanish Crown was member. Thus, the IPs could not alienate, encumber or sell
considered to have acquired dominion only over the those of pertaining to natural resources.
unoccupied and unclaimed portions of our islands.
Intent of the framers of the Constitution C. The provisions of IPRA pertaining to the utilization of
• It is readily apparent from the constitutional natural resources are not unconstitutional.

Notes of Cesnee Tan Page 10


Natural Resources 1st Exam (2017-2018)

• The IPRA provides that indigenous peoples interpretation of an almost forgotten law of Spain.
shall have the right to manage and conserve the natural The grant to the plaintiff was the result of the principle of
resources found on the ancestral domains, to benefit from Prescription as mentioned in the royal cedula of 1754 states
and share in the profits from the allocation and utilization : ― Where such possessors shall not be able to produce
of these resources, and to negotiate the terms and title deeds, it shall be sufficient if they s hall show that
conditions for the exploration of such natural resources. ancient possession, as a valid title by prescription‖ .
The statute also grants them priority rights in the Moreover, the Decree of June 25, 1880 states that
harvesting, extraction, development or exploitation of any possessors for certain time shall be deemed owners, if a
natural resources within the ancestral domains. cultivated land 20 years, if uncultivated 30 years.
• Although Section 2, Article XII of the Mateo‘s father was the owner of the land by the very terms
Constitution provides in paragraph 1 thereof that the of this decree- by Organic Act of July 1, 1902 ,all the
exploration, development and utilization of natural property and rights acquired there by the United States are
resources must be under the full control and supervision of to be administered ―for the benefit of the inhibitants
the State, the Constitution itself states in the third thereof.‖
paragraph of the same section that Congress may, by law,
allow small-scale utilization of natural resources by its
citizens. c. Reservations of lands of the public domain are valid
assertions of Regalian Right
Cariño vs Insular Government
41 Phil. 935 (1909) Reserving certain lands of the public domain for specific
Facts: public purposes have the character of official assertions of
Mateo Carino(appellant) filed his petition in the Court of ownership, and the presumption is that they have been
Land Registration to be granted a parcel of land consisting issued by right of sovereignty and in the exercise of the
of 40 Hectares,1are and 13 Centares in Baguio, Province of State‘s dominical authority. These are valid asseveration of
Benguet together with a house erected thereon. It was Regalian right over the property.
granted, but the Government of the Philippines and also on
behalf of the United States averred having taken possession In Acting Registrars of Land Titles and Deeds of Pasay VS
of the property for public and military purposes, RTC (1990), what the government occupies is what the
Respondents also asserted that they had title to all the land government owns, unless otherwise shown.
in the Philippines except to permit private lands to be
acquired and no prescription runs against the Spanish Cases:
crown. The US succeeded the title of Spain through Treaty
of Paris and Mateo‘s land was not registered and that he Acting Registrars of Land Titles and Deeds of Pasay
had lost all his rights and now is deemed to be a mere City vs. Regional Trial Court Branch 57 Makati
trespasser. Then the Court of First Instance dismissed the (GR No. 81564, 4/26/1990)
application for Mateo since he did not possess the land FACTS:
since time immemorial and the land was property of the On November 5, 1985, Domingo Palomares, as
Government. The decision was affirmed by the Philippine administrator of the heirs of Delfin Casal, commenced suit
Supreme Court. Thus the case was brought to the US with the Regional Trial Court, Branch 132, Makati, Metro
Supreme Court by virtue of Writ of Error(general method of Manila for declaratory relief, quieting of title, cancellation of
bringing cases to this court, an appeal the exception, Transfer Certificate of Title No. 192, and cancellation of
confined to equity in the main. entries upon Original Certificate of Title No. 291.

Mateo Carino in his appeal stated that he is an Igorot of the However, during the trial the court found hard evidence on
Province of Benguet, they have owned the land for more record that: (1) the property covered by OCT No. 291 had
than 50 years before the Treaty of Paris, and they all had been conveyed to the United States of America; (2) it had
been recognized as owners by the Igorots and had been later ceded to the Republic of the Philippines; and (3)
inherited or received his land from his father in accordance as a consequence, OCT No. 291 was cancelled upon final
with Igorot custom. However, no document of title had orders of Judge Ostrand.
been issued from the Spanish Crown
During the pendency of the case also, Proclamation No. 192
Issue : WON Carino owns the land. (―RESERVING FOR THE VETERANS CENTER SITE
PURPOSES CERTAIN PARCEL OF LAND OF THE PUBLIC
DOMAIN SITUATED IN THE PROVINCE OF RIZAL, ISLAND
Held : Yes OF LUZON‖) and Proclamation No. 423 (―RESERVING FOR
The US Supreme Court reversed the decision of the MILITARY PURPOSES CERTAIN PARCELS OF THE PUBLIC
Philippine Supreme Court. DOMAIN SITUATED IN THE MUNICIPALITY OF PASIG,
Mateo Carino should be granted what he seeks and should TAGUIG, AND PARAÑAQUE, PROVINCE OF RIZAL, AND
not be deprived of what by the practice and belief of those PASAY CITY‖) were issued by the government.
among whom he lived, was his property, through a refined ISSUE: Whether or not, Proclamation 192 and 423 were

Notes of Cesnee Tan Page 11


Natural Resources 1st Exam (2017-2018)

valid in the absence of proof of ownership of the property


by the government? Classification of Lands

HELD: Yes. Such proclamations have the character of Lands of the Public Lands of the Private
official assertions of ownership, and the presumption is that Domain Domain
they have been issued by right of sovereignty and in the Lands which are either Land belonging to and
exercise of the State‘s dominical authority. We take not only alienable or inalienable. owned by the State as a
judicial notice thereof but accept the same as a valid private individual, without
asseveration of regalian right over property. being devoted for public
use, public service or
Organizational Structure development of national
wealth
A. DENR, functions of the DENR Secretary, Director of LMB (similar to patrimonial
The DENR Secretary is the executive officer charged with properties of the State)
carrying out the provision of the Public Land Act, through
the Director of Lands, who shall act under his immediate NCC: Properties of Public NCC: Patrimonial
control. Dominion Properties of the State
Those intended for public Properties other than
The Director of Lands, shall have direct executive control of use or those which belong properties of the public
the survey, classification, lease, sale, or any other form of to the State without being dominion or former
concession or disposition and management of the lands of for public use, intended for properties of the public
the public domain. some public service or for dominion that are no longer
the development of intended for public use
The jurisdiction of the Department over public lands does national wealth or public service.
not negate the authority of the court to resolve questions of
possession and the court‘s decision stands in meantime that As to the general classification of the land, there are 2.
the DENR has not settled the respective rights of public First, lands of the public domain then lands of private
land claimants. But once DENR has decided, particularly domain. Now under the lands of public domain, this
with the grant of a public land patent, for instance, and includes alienable and non-alienable lands. Alienable lands
issuance of the corresponding certificate of title, DENR‘s are limited to agricultural lands. If you also remember
decision prevails. under your Constitution, there is what we call Public
Agricultural Land. Those are the only ones that can be
The administration and distribution of public land is registered. Non alienable, of course, mineral, timber lands
committed by law to the Director of Lands and ultimately to and foreshore lands.
the DENR Secretary.
There is also a separate classification under the Civil Code.
General Rule: Under EO 192, the newly created Lands 1. Properties of the Public Dominion or those intended for
Management Bureau (LMB) has absorbed the functions and public use
powers of the Bureau of Lands 2. Those which belong to the State, without being for
public use, intended for some public service or for the
Exception: Except those line functions and powers which development of national wealth
were transferred to the regional field offices. 3. Patrimonial properties of the State

DENR LMB So for public use, some examples would be roads, canals
A Regional Office is headed Is headed by a Director and and bridges. Lands of public service may be used only by
by a Regional Executive assisted by an Assistant duly authorized persons. An example is a public national
Director who shall be Director who shall advise government building. Lastly, patrimonial properties are
assisted by 5 Regional the DENR Secretary on properties of the State which are declared to be no longer
Technical Directors. matters pertaining to needed in public use or for public purpose.
Note: The Regional Offices rational land classification
of the then Bureau of Forest management and Under the 1935 Constitution, there were 3 classifications:
Development, disposition. 1. Agricultural
Bureau of Mines and 2. Timber
Geosciences, and Bureau of 3. Mineral
Lands are now integrated ―Public Agricultural Lands‖ are neither timber nor mineral,
into Regional Environment and includes residential lands.
and Natural Resources
Office. Under the 1973 Constituion, the classification are as
follows:
1. Agricultural

Notes of Cesnee Tan Page 12


Natural Resources 1st Exam (2017-2018)

2. Industrial such period as it may determine measures to prohibit


3. Commercial logging in endangered forests and watershed areas.
4. Residential
5. Resettlement SECTION 5. The State, subject to the provisions of this
6. Mineral Constitution and national development policies and
7. Timber or forest programs, shall protect the rights of indigenous cultural
8. Grazing lands communities to their ancestral lands to ensure their
9. Such other classes provided by law economic, social, and cultural well-being.

There is also a separate classification of lands found in the The Congress may provide for the applicability of customary
1987 Constitution. There are only 4: laws governing property rights or relations in determining
1. Agricultural land, the ownership and extent of ancestral domain.
2. Forest/Timber lands
3. Mineral Lands and Cases:
4. National Parks
How about lands covered by forest reservations? In the
Only Agricultural lands are open for alienation and case of Palomo, the land was reserved for the Tiwi Hot
disposition. Springs. They were able to register it in their name. The
Supreme Court held that it is invalid. Possession of lands
For the purpose of the administration and disposition of subject to forest reservations, no matter how long, cannot
alienable or disposable public lands, the President, upon ripen into ownership. The Supreme Court held that the
recommendation of the Secretary of DENR, shall from time classification of land is descriptive of its legal nature and
to time declare what lands are open to disposition or not how the land actually looks like. Even if it looks like an
concession under the Act. agricultural land, then classified as timber land, it‘s actually
timber land and you cannot possess it.
Sec. 9 of CA 141 provides that alienable or disposable lands
shall be classified, for the purpose of administration and
disposition into: ARER 1. Agricultural; 2. Residential,
commercial, industrial, or for similar productive purposes; Palomo v. Court of Appeals
3. Educational, charitable, or other similar purposes; and 4. 266 SCRA 392
Reservations for town sites and for public and quasi- public FACTS:
purposes In 1913, some 440,530 square meters of land in Albay were
Constitution (Section 3-5, Article XII) reserved for provincial park purposes by virtue of EO No.
40. Of said area, 15 parcels of land were registered in the
SECTION 3. Lands of the public domain are classified into name of Diego Palomo by the Court of First Instance.
agricultural, forest or timber, mineral lands, and national
parks. Agricultural lands of the public domain may be In 1937, Diego Palomo donated these lands to his heirs
further classified by law according to the uses which they Ignacio and Carmen Palomo. Claiming that the aforesaid
may be devoted. Alienable lands of the public domain shall original certificates of title were lost during the Japanese
be limited to agricultural lands. Private corporations or occupation, Ignacio Palomo filed a petition for
associations may not hold such alienable lands of the public reconstitution with the Court of First Instance of Albay on
domain except by lease, for a period not exceeding twenty- May 30, 1950. The Register of Deeds of Albay issued
five years, renewable for not more than twenty-five years, Transfer Certificates of Title Nos. 3911, 3912, 3913 and
and not to exceed one thousand hectares in area. Citizens 3914 sometime in October 1953.
of the Philippines may lease not more than five hundred
hectares, or acquire not more than twelve hectares thereof On July 10, 1954, President Magsaysay issued Proclamation
by purchase, homestead, or grant. No. 47 converting the area covered by EO 40 into the Tiwi
Taking into account the requirements of conservation, Hot Spring National Park. The Palomos contended that they
ecology, and development, and subject to the requirements have been in possession of the subject lands and have
of agrarian reform, the Congress shall determine, by law, introduced improvements thereon.
the size of lands of the public domain which may be
acquired, developed, held, or leased and the conditions ISSUE: Were the Original Certificate of Titles issued to the
therefor. petitioners valid? –NO

SECTION 4. The Congress shall, as soon as possible, HELD


determine by law the specific limits of forest lands and Before the Treaty of Paris in 1899, the lands, whether
national parks, marking clearly their boundaries on the agricultural, mineral, or forest were under the exclusive
ground. Thereafter, such forest lands and national parks patrimony and dominion of the Spanish crown.Private
shall be conserved and may not be increased nor ownership of land could only be acquired through royal
diminished, except by law. The Congress shall provide, for concessions which were documented in various forms, such

Notes of Cesnee Tan Page 13


Natural Resources 1st Exam (2017-2018)

as (1) Titulo Real or Royal Grant," (2) Concession Especial the same after a claim of ownership for the required
or Special Grant, (3) Titulo de Compra or Title by Purchase number of years to constitute a grant from the State. A
and (4) Informacion Posesoria or Possessory Information mere casual cultivation of portions of the land by the
title obtained under the Spanish Mortgage Law or under the claimant does not constitute sufficient basis for a claim of
Royal Decree of January 26, 1889. ownership, such possession is not exclusive and notorious
as to give rise to presumptive grant from the state. In light
The decision of the CFI relied upon by petitioners were not of the foregoing, the petition of the Republic of the
signed by the judge but were merely certified copies of Philippines is granted.
notification to Diego Palomo bearing the signature of the
Clerk of Court. This case discussed the requirement for the presentation of
the proof. There are actually 2 conflicting cases. On the
More importantly, the lands in question were not classified first case, general rule Republic vs TAN Properties June 26,
as alienable lands. Since the lands were made part of a 2008, the applicant must submit or present a copy of the
reservation for provincial park purposes, they form part of original classification approved by the DENR Secretary. An
the forest zone. Thus, they cannot be the valid subject of exception is Republic vs Serrano, allows such not to be
alienation. submitted (classification). In this case of Leonidas vs
Vargas, the Supreme Court reiterated that you always
What must the applicant show? This pertains to when you follow the general rule and that is, to present the
apply for registration. To prove that the land subject for classification approved by the DENR Secretary. Substantial
application is alienable, an applicant must conclusively compliance applies on a case to case basis. That is, based
establish the existence of a positive act of the government on the sound discretion of the Court.
such as a Presidential Proclamation, Executive Order or
Administration. This was held in the case of Republic vs Leonidas vs Vargas
Candy Maker. 201031 dec 14 2017
[I]n Republic v. TA.N Properties, Jnc., which was
Republic vs Candy Maker Inc promulgated on 26 June 2008 x x x we held that applicants
(GR No. 163766, June 22, 2006) for land registration must present a copy of the original
FACTS: classification approved by the DENR Secretary and certified
On April, 29, 1999, Antonia, Eladia, and Felisa, all as true copy by the legal custodian of the official records.
surnamed Cruz, executed a Deed of Absolute Sale in favor In Republic v. Serrano (decided on 24 February 2010)], we
of Candy Maker, Inc. for a parcel of land located below the allowed the approval of a land registration application even
reglementary lake elevation of 12.50m, about 900 meters without the submission of the certification from the DENR
away the Laguna de Bay. Candy Maker, Inc. as applicant, Secretary. As this ruling presented an apparent
filed an application with the MTC of Taytay, Rizal for contradiction with our earlier pronouncement in Republic v.
registration of its alleged title over the lot. The CENRO of TA.N Properties, Inc., we sought to harmonize our previous
Antipolo City declared the land to fall within the alienable rulings in Republic v. vega [(decided on 17 January 2011)J.
and disposable zone. On the other hand, the Land We then said that the applications for land registration may
Registration Authority recommended the exclusion of lot no. be granted even without the DENR Secretary's certification
3138-B on the ground that it is a legal easement and provided that the application was currently pending at the
intended for public use, hence, inalienable and time Republic v. vega was promulgated.
indisposable. On July 2001, the Republic of the Philippines,
the Laguna Lake Development Authority (LLDA) filed its It is worth stressing, however, that the foregoing ruling is
opposition which alleged that the lot subject of the the exception, not the rule. As explicitly elucidated in
application for registration may not be alienated and Republic v. Vega:
disposed since it is considered part of the Laguna Lake Bed, It must be emphasized that the present ruling on
a public land within, its jurisdiction. substantial compliance applies pro hac vice. It does not in
any way detract from our rulings in Republic i~ TA.N
ISSUE: Properties, Inc., and similar cases which impose a strict
Whether the property subject of the amended application is requirement to prove that the public land is alienable and
alienable and disposable property of the State, and if so, disposable, especially in this case when the Decisions of the
whether respondent adduced the requisite quantum of lower court and the [CA] were rendered prior to these
evidence to prove its ownership over the property? rulings. To establish that the land subject of the application
is alienable and disposable public land, the general rule
HELD: remains: all applications for original registration under [PD
The property subject of this application was alienable and 1529] must include both (1) a CENRO or PENRO
disposable public agricultural land. However, respondent certification and (2) a certified true ~Jf the original
failed to prove that it possesses registrable title over the classification made by the DENR Secretary.
property. The statute of limitations with regard to public
agricultural lands does not operate against the statute As an exception, however, the courts - in their sound
unless the occupant proves possession and occupation of discretion and based solely on the evidence presented on

Notes of Cesnee Tan Page 14


Natural Resources 1st Exam (2017-2018)

record - may approve the application, pro hac vice, on the Development Plan of the Southern Reclamation Project-
ground of substantial compliance showing that there has Manila Cavite Coastal Road Reclamation Project. The JVA
been a positive act of government to show the nature and was later amended giving AMARI an option to reclaim an
character of the land and an absence of effective opposition additional 350 hectares of submerged area. Part of the
from the government. This exception shall only apply to consideration for AMARI‘s work is the conveyance of 70%
applications for registration currently pending before the of the total net usable reclaimed area – equivalent to 367.5
trial court prior to this Decision and shall be inapplicable to hectares, title of which will be in AMARI‘s name.
all future applications. (Underscoring and emphases in the
original) That said, we hold that both the petitioner and Issue: Whether or not AMARI, a private corporation, can
Tancredo failed to establish clearly and convincingly their acquire and own under the Amended JVA 367.5 hectares of
respective rights to registration of imperfect titles under CA reclaimed foreshore and submerged areas in Manila Bay
141 and PD 1529, as will be discussed below.
Held: No. AMARI as a private corporation cannot acquire
WHAT IS THE POLICY OF THE STATE IN CONNECTION the reclaimed Freedom Islands, though alienable lands of
WITH ALIENABLE PUBLIC LANDS? the public domain, except by lease, as provided under
Section 3, Article XII of the Constitution. The still
TO ENCOURAGE AND PROMOTE THE DISTRIBUTION OF submerged areas (i.e., the more or less additional 250 and
ALIENABLE PUBLIC LANDS TO SPUR ECONOMIC GROWTH 350 hectares of submerged areas) in Manila Bay are
BUT SAFEGUARDS ARE IMPOSED LEST SUCH LANDS FALL inalienable lands of the public domain; as such, they are
INTO WRONG HANDS TO THE PREJUDICE OF THE beyond the commerce of man, as provided under Section 2,
NATIONAL PATRIMONY. Article XII of the Constitution.

It is axiomatic of course that ―[i]t is the policy of the State The reclaimed Freedom Islands: The assignment to PEA of
to encourage and promote the distribution of alienable the ownership and administration of the reclaimed areas in
public lands as a spur to economic growth and in line with Manila Bay, coupled with President Aquino‘s actual issuance
the social justice ideal enshrined in the Constitution. At the of a special patent covering the Freedom Islands, is
same time, the law imposes stringent safeguards upon the equivalent to an official proclamation classifying the
grant of such resources lest they fall into the wrong hands Freedom Islands as alienable or disposable lands of the
to the prejudice of the national patrimony.‖63 This ruling public domain. They also constitute a declaration that the
controls the present case. Freedom Islands are no longer needed for public service.
The Freedom Islands are thus alienable or disposable lands
How about submerged lands? Can you register it? of the public domain, open to disposition or concession to
Submerged lands cannot be registered. However, if it will qualified parties.
be reclaimed, meaning land will be placed, that may be
registered provided that it is declared open for disposition, The submerged areas: The mere reclamation of foreshore
alienable and disposable. and submerged areas by PEA does not convert these
inalienable natural resources of the State into alienable or
As to foreshore lands, they may be subject of foreshore disposable lands of the public domain. There must be a law
lease application but they cannot be registered in the name or presidential proclamation officially classifying these
of one person. reclaimed lands as alienable or disposable and open to
disposition or concession. Moreover, these reclaimed lands
Chavez vs Public Estates Authority cannot be classified as alienable or disposable if the law
[G.R. No. 133250. July 9, 2002] has reserved them for some public or quasi-public use.
Facts: Public Estates Authority (PEA) is a wholly
government-owned and –controlled corporation which is PEA‘s authority to sell: In order for PEA to sell its reclaimed
the primary implementing agency of the National foreshore and submerged alienable lands of the public
Government to reclaim foreshore and submerged lands of domain, there must be legislative authority empowering
the public domain. By virtue of a Special Patent issued by PEA to sell these lands, in view of the requirement under
President Corazon Aquino, the Register of Deeds of the CA No. 141. Without such legislative authority, PEA could
Paranaque, in April 1988, issued certificates of title, in the not sell but only lease its reclaimed foreshore and
name of PEA, covering three reclaimed islands known as submerged alienable lands of the public domain. PEA‘s
the Freedom Islands located at the southern portion of the Charter grants it such express legislative authority to sell its
Manila-Cavite Coastal Road, Paranaque City. The Freedom lands, whether patrimonial or alienable lands of the public
Islands have a total land area of 157.841 hectares. domain. Nevertheless, any legislative authority granted to
PEA to sell its reclaimed alienable lands of the public
In April 1995, PEA entered into a Joint Venture Agreement domain would be subject to the constitutional ban on
(JVA) with AMARI, a private corporation, to develop the private corporations from acquiring alienable lands of the
Freedom Islands. The JVA also required the reclamation of public domain. Hence, such legislative authority could only
an additional 250 hectares of submerged areas surrounding benefit private individuals.
these islands to complete the configuration in the Master

Notes of Cesnee Tan Page 15


Natural Resources 1st Exam (2017-2018)

Registration of alienable lands of the public domain: Classification of Public lands open to disposition
Registration of land under Act No. 496 or PD No. 1529 does
not vest in the registrant private or public ownership of the Now, how about lands classified under CA 141? There are
land. Registration is not a mode of acquiring ownership but only 3; Alienable and disposable, timber and mineral lands.
is merely evidence of ownership previously conferred by Now, specifically, this alienable and disposable lands may
any of the recognized modes of acquiring ownership. be classified further to which purposes such land are
Registration does not give the registrant a better right than destined; such as, Agricultural, Residential, Educational,
what the registrant had prior to the registration. The and Reservation for Camp sites and for public and quasi-
registration of lands of the public domain under the Torrens public purposes.
system, by itself, cannot convert public lands into private
lands. Jurisprudence holding that upon the grant of the From the foregoing classification, public agricultural land
patent or issuance of the certificate of title the alienable may be defined as those alienable portions of the public
land of the public domain automatically becomes private domain which are neither timber nor mineral lands. Thus,
land cannot apply to government units and entities like the term includes residential, commercial and industrial
PEA. lands for the reason that these lands are neither timber nor
mineral lands.
Lands registered under Act No. 496 or PD No. 1529 are not
exclusively private or patrimonial lands. Lands of the public Cases:
domain may also be registered pursuant to existing laws.
Several laws authorize lands of the public domain to be Alba vs Court of Appeals
registered under the Torrens System or Act No. 496, now (GR No. 120066, 9/9/1999)
PD No. 1529, without losing their character as public lands. FACTS:
For instance, In 1958, Lachica filed an application for title to a 4,485
sq.m. parcel of land which he had acquired through
- Under the Revised Administrative Code of purchase dating back to 1945. However, some parties filed
1987, private property purchased by the National their opposition thereto invoking that they are owners of
Government for expansion of an airport may be titled in the some parts of land within the total area applied for. The
name of the government agency tasked to administer the lower court ruled in favor of Lachica and held, among
airport. Private property donated to a municipality for use others, that Lachica had been in actual, open and
as a town plaza or public school site may likewise be titled continuous possession of the subject land in the concept of
in the name of the municipality. All these properties owner since 1945 and thus had acquired it through
become properties of the public domain, and if already prescription.
registered under Act No. 496 or PD No. 1529, remain
registered land. There is no requirement or provision in any ISSUE:
existing law for the de-registration of land from the Torrens Did Lachica acquire the subject property by prescription?
System.
RULING:
- Private lands taken by the Government for The Court ruled on the negative.
public use under its power of eminent domain become
unquestionably part of the public domain. Nevertheless, Based from the facts and evidences presented, it was
Section 85 of PD No. 1529 authorizes the Register of Deeds proven that Lachicha only had a title to a 620sq.m. portion
to issue in the name of the National Government new of the total area. Prescription cannot be had on the
certificates of title covering such expropriated lands. remaining area as the Court held that the law applicable in
this case is Sec. 48 of CA 141 (wc deals with registration of
Public Land Act lands of public domain) and not Sec. 19 of Act 496 (wc
deals with registration of private lands) and with which the
Under Section 6 of the Public Land Act, the prerogative of lower courts had relied on.
classifying or reclassifying lands of the public domain
belongs to the executive branch of the government and not The law in force at the time an action accrues is what
the court. The onus to overturn, by incontrovertible governs the proceeding consistent with the fundamental
evidence, the presumption that the land of an application dictum that laws shall have no retroactive effect, unless the
for registration is alienable and disposable rests with the contrary is proved. In this case, the lower courts relied on
applicant. the provisions on prescription with the assumption that the
subject property is a private land. However, the application
Section 6 of the Public Land Act classifies lands of the for registration should be that of a judicial confirmation of
public domain into: 1. Alienable or disposable 2. Timber an imperfect title considering that the land is presumed
and 3. Mineral lands. The President may at any time and in under the Regalian Doctrine to be part of the public
a like manner transfer such lands from one class to another, domain.
for the purposes of their administration and disposition.
Public lands are classified into (1) alienable or disposable

Notes of Cesnee Tan Page 16


Natural Resources 1st Exam (2017-2018)

lands which includes agricultural lands and (2) inalienable foreshore land based on an investigation conducted by the
or non-disposable lands or those not susceptible of private DENR, Region V, Legazpi City. Respondents, on the other
appropriation which includes Timber lands and Mineral hand contend that Director of Lands found Jose Baritua's
lands. For purposes of administration and disposition, the land covered by TCT No.18655, which stemmed from OCT
lands of the public domain classified as "disposable" or 408(500), to be "definitely outside of the foreshore area."
"alienable" are further sub-classified into a.] Agricultural;
b.] Residential, commercial, industrial or for similar Within the time for pleading, private respondents EANCRA
productive purposes; c.] Educational, charitable or other Corporation, Lolita Alcazar and Salvador Alcazar filed their
similar purposes, and d.] Reservations for town sites and answer with cross-claim, while the rest, namely, Felix S.
for public and quasi-public purposes. Imperial, Feliza S. Imperial, Elias S. Imperial and Miriam S.
Imperial filed a motion to dismiss. They contended that the
Possession of public agricultural land, however long the adjudication by the cadastral court is binding against the
period may have extended, never confers title thereto upon whole world including the Republic since the cadastral
the possessor and it is because the statute of limitations proceedings are in rem and the government itself through
with regard to public agricultural land does not operate the Director of Lands instituted the proceedings and was a
against the State, unless the occupant can prove direct and active participant therein. Petitioner, through the
possession and occupation of the same under claim of Office of the Solicitor General, filed an objection to the
ownership for the required number of years to constitute a motion to dismiss. After hearing the motion to dismiss, the
grant from the State. trial court dismissed the complaint on the ground that the
judgment rendered by the cadastral court in G.R. Cad. Rec.
Under (b), Sec. 48, CA 141, confirmation of an imperfect No. 88 and the Courts resolution in the petition to quiet
title to a public domain requires that: title, G.R. 85770, both decreed that the parcel of land
1. There be an open, continuous, exclusive and covered by OCT No. 408 (500) was not foreshore. Petitioner
notorious possession and occupation of agricultural lands of appealed to the Court of Appeals. The appellate court
the public domain; denied petitioners motion for reconsideration for lack of
2. It should be under a bona fide claim of ownership; merit and for failure to file the appellants brief within the
and extended period granted to petitioner.
3. possession should be for at least thirty years
immediately preceding the filing of the application for Hence, the present petition.
confirmation of title except when prevented by war or force
majeure ISSUE: Whether or not the petition should be granted.

In this case, Lachica had not yet satisfied the requirement HELD: Yes.
of the 30 years possession, hence, prescription cannot be At the core of the controversy is whether the parcels of
granted in favor of him. land in question are foreshore lands. Foreshore land is a
part of the alienable land of the public domain and may be
D. Classification of Lands an Executive Privilege disposed of only by lease and not otherwise. It was defined
as "that part (of the land) which is between high and low
Who classifies the lands? That is the prerogative of the water and left dry by the flux and reflux of the tides." It is
President upon the recommendation of the Secretary of the also known as "a strip of land that lies between the high
DENR. A positive act from the government is required and and low water marks and, is alternatively wet and dry
that is by virtue of a Presidential Proclamation or according to the flow of the tide."
Administrative Enactment.
The classification of public lands is a function of the
Cases: executive branch of government, specifically the director of
lands (now the director of the Lands Management Bureau).
Republic vs Imperial The decision of the director of lands when approved by the
(GR No. 130906, 2/11/1990) Secretary of the Department of Environment and Natural
FACTS: Resources (DENR) as to questions of fact is conclusive upon
On September 12, 1917, the late Elias Imperial was issued the court.
Original Certificate of Title (OCT) 408 (500) pursuant to
Decree No. 55173 of then Court of First Instance of Albay. There is allegedly a conflict between the findings of the
OCT No. 55173 was subdivided and further subdivided Director of Lands and the DENR, Region V, in the present
resulting in the issuance of several titles, which are now the case. Respondents contend that the Director of Lands
subjects of herein petition in the name of private found Jose Baritua's land covered by TCT No.18655, which
respondents. Petitioner Republic of the Philippines filed a stemmed from OCT 408(500), to be "definitely outside of
case with the trial court to judicially declare the Transfer the foreshore area." Petitioner, on the other hand, claims
Certificates of Title (TCT) issued to herein private that subsequent investigation of the DENR, Region V,
respondents null and void on the ground that the subject Legazpi City, disclosed that the land covered by OCT No.
land, on which the OCT was based, has the features of a 408 (500) from whence the titles were derived "has the

Notes of Cesnee Tan Page 17


Natural Resources 1st Exam (2017-2018)

features of a foreshore land." The contradictory views of and jurisdiction of the cadastral court to register under the
the Director of Lands and the DENR, Region V, Legazpi City, Torrens System.
on the true nature of the land, which contradiction was
neither discussed nor resolved by the RTC, cannot be the E. Public Land vs Government land
premise of any conclusive classification of the land
involved. Now what is the difference of government lands and public
lands? Government lands cover public lands. Public lands
The need, therefore, to determine once and for all whether means public domain or lands open for private
the lands subject of petitioner's reversion efforts are appropriation.
foreshore lands constitutes good and sufficient cause for
relaxing procedural rules and granting the third and fourth Cases:
motions for extension to file appellant's brief. Petitioner's
appeal presents an exceptional circumstance impressed Montano vs Insular Government
with public interest and must then be given due course. (GR No. 3714, 1//26/1909)
Facts: Montano applied for land registration of a parcel of
In the case at bar, the need to determine once and for all land used for fishery in Libis, Caloocan. This was opposed
whether the lands subject of petitioners reversion efforts to by the Solicitor-General (Sol. Gen.) contending that the
are foreshore lands constitutes good and sufficient cause land is owned by the US Government as well as Obras Pias
for relaxing the procedural rules and granting the third and de la Sagrda Mitra as it was the absolute owner of the lot.
fourth motions for extensions to file appellants brief.
Petitioner Republics appeal presented an exceptional The Court of Land Registration granted the application of
circumstance impressed with public interest which in the Montano. Only the Sol. Gen. appealed the decision.
Courts discretion must be given due course.
Issue: Whether the lands used as a fishery, for the growth
Director of Lands vs Court of Appeals and Bisnar of nipa, and as salt deposits, inland some distance from the
(GR No. 83609, 10/26/1989) sea, and asserted, though not clearly proved to be
Facts: On July 20,1976, Ibarra and Amelia Bisnar filed their overflowed at high tide could be registered as private
joint application for the registration of two parcels of land, property.
located in the province of Capiz, in the CFI of Capiz. They
claimed that they inherited those parcels of land. The Held: Yes, it can.
Director of Lands and Director of the Bureau of Forest The Supreme Court held that since fish ponds are not
Development opposed the application on the ground that classified as forest or mineral lands, it must necessarily be
said parcels of land were part of a timberland, a public agricultural land. The Court noted that before Montano
dominion, so it cannot be the subject of the registration owned the lot, bacawan had been sown and propagated in
proceedings. After the hearing, the CFI ordered the the mud by the owner.
registration of the title of the lots in the names of the
applicants, herein private respondents after finding that the The Court also noted that government land is NOT
applicants and their predecessors- in-interest have been in similar to public domain or public land. Government land
open, public, continuous, peaceful and adverse possession includes not only public domain, but also other lands of the
of the subject parcels of land under bona fide claims of Government already reserved or developed to public use or
ownership for more than 80 years. The CA affirmed the subject to private right.
CFI‘s decision, holding that the classification of the lots as Government owns real estate which is part of public lands
timberland by the Director of Forestry cannot prevail in the and other real estate which is not part thereof. There is
absence of proof that the said lots are indeed more much real property belonging to the Government which is
valuable as forest land than as agricultural land, citing as not affected by statutes for the settlement, prescription or
authority the case of Ankron vs. Government of the sale of public lands. Examples in point are properties
Philippine Islands (40 Phil.10). occupied by public buildings or devoted to municipal or
Issue/s: Whether or not the possession of forestlands or other governmental uses.
timberlands for 80 years can ripen to private ownership.
Ruling: Modes of Disposition
No. The Court ruled that possession of forestlands, however
long, cannot ripen into private ownership. It emphasized No public land can be acquired by private persons without
that a positive act of the government, particularly the any grant, express or implied from the government. In
Executive Department is needed to declassify land, which is other words, it is indispensable that there be a showing of
classified as forest, and to convert it into alienable or a title from the state. This may come in the form of a
disposable land for agricultural or other purposes before homestead, sales or free patent or grant. One claiming
registration of which may proceed. The Court, citing various private rights must prove that he has complied with the
cases, stated that a parcel of forestland is within the Public Land Act which prescribes the substantive as well as
exclusive jurisdiction of the Bureau of Forestry, an office the procedural requirements for acquisition of public lands.
under the Executive Department, and beyond the power

Notes of Cesnee Tan Page 18


Natural Resources 1st Exam (2017-2018)

Public lands suitable for agricultural purposes can be proceedings of Naga Cadastre after claiming it as their
disposed of only as follows: private property.
1. For homestead settlement
2. By sale Well-settled Rule : no public land can be acquired by
3. By lease private persons without any grant, express or implied, from
4. By confirmation of imperfect or incomplete titles the government
a. By judicial legalization
b. By administrative legalization (free patent) Cabacug v. Lao: holder of a land acquired under a free
patent is more favorably situated than that of an owner of
The words alienation, disposition or concession as used in registered property. Not only does a free patent have a
the Public Land Act means any of the methods authorized force and effect of a Torrens Title, but in addition the
by the Act for the acquisition, lease, use or benefit of the person to whom it is granted has likewise in his favor the
lands of the public domain other that timber or mineral right to repurchase within a period of 5 years.
lands. Each mode of disposition is appropriately covered by
separate chapters of the Public land Act because there are Imperium v. Dominium
specific requirements and application procedure for every
mode. 1. Imperium - government authority possessed by the
state which is appropriately embraced in the concept of
Cases: sovereignty

Lee Hong Kok vs David 2. Dominium - capacity to own or acquire


(GR No.G.R. No. L-30389, 12/27/1972) property. The use of this term is appropriate with reference
FACTS: Aniano David acquired lawful title pursuant to his to lands held by the state in its proprietary character. In
miscellaneous sales application in accordance with which an such capacity, it may provide for the exploitation and use of
order of award and for issuance of a sales patent (*similar lands and other natural resources, including their
to public auction) was made by the Director of Lands on disposition, except as limited by the Constitution.
June 18, 1958, covering Lot 2892.
Director of Lands vs CA and Valeriano
On the basis of the order of award of the Director of (GR No. 58867, 6/22/1984)
Lands the Undersecretary of Agriculture and Natural Faustino Ignacio filed an application to register a parcel of
Resources issued on August 26, 1959, Miscellaneous Sales land (mangrove) which he alleged he acquired by right of
Patent No. V-1209 pursuant to which OCT No. 510 was accretion since it adjoins a parcel of land owned by the
issued by the Register of Deeds of Naga City on October Ignacio. His application is opposed by the Director of Lands,
21, 1959. Laureano Valeriano, contending that said land forms part of
the public domain. The Trial Court dismissed the application
Land in question is not a private property as the Director holding that said land formed part of the public domain.
of Lands and the Secretary of Agriculture and Natural Thus the case at bar.
Resources have always sustained the public character for
having been formed by reclamation (as opposed to ISSUE:
petitioners contention that it is accretion) Whether or not the land forms part of the public domain
The only remedy: action for reconveyance on the ground
of fraud - But there was no fraud in this case HELD: YES

ISSUES: 1. The law on accretion cited by Ignacio in inapplicable in


1. W/N Lee Hong Kok can question the grant. - NO the present case because it refers to accretion or deposits
on the banks of rivers while this refers to action in the
2. W/N David has original acquisition of title. - YES Manila Bay, which is held to be part of the sea

HELD: Court of Appeals Affirmed. (no legal justification for 2. Although it is provided for by the Law of Waters that
nullifying the right of David to the disputed lot arising from lands added to shores by accretions caused by actions of
the grant made in his favor by respondent officials) the sea form part of the public domain when they are no
Only the Government, represented by the Director of longer necessary for purposes of public utility, only the
Lands, or the Secretary of Agriculture and Natural executive and the legislative departments have the
Resources, can bring an action to cancel a void certificate authority and the power to make the declaration that any
of title issued pursuant to a void patent. The legality of the said land is no longer necessary for public use. Until such
grant is a question between the grantee and the declaration is made by said departments, the lot in question
government. Private parties like the plaintiffs cannot claim forms part of the public domain, not available for private
that the patent and title issued for the land involved are appropriation or ownership.
void since they are not the registered owners thereof nor
had they been declared as owners in the cadastral

Notes of Cesnee Tan Page 19


Natural Resources 1st Exam (2017-2018)

Menguito vs Republic
(GR No. 134308, 12/14/2000) ―(b) Who by themselves, or their predecessors in interest
FACTS: A petition for review assailing the Court of Appeals have been in open, continuous, exclusive, and notorious
Sept 30, 1997 decision against the Menguito‘s –the occupation of the land in the concept of the owner.‖
petitioners, with promulgated resolution 10-Mos. later
reversing the decision of the Regional Trial Court of Pasig The Court thus observed that the petitioners have
City. The RTC decision confirmed the application for the insufficient evidence on this.‖
titling of the parcel of land with aggregate area of 2112
sqm located at Brgy Ususan, Taguig, Metro Manila, in favor The Court likewise pointed that;
of the Menguito‘s.
• Applicants to avail the OCT they must
The application of TCT was brought by the operation of the overcome the presumption that the land they applied for
Land Registration of Authority as amended by the Property forms part of the public domain and if so, it has to be
Registration Decree No. 1529 proceeding to declare: shown as re-classified or alienated to a private person by
the state.
1. That its applicants –the Menguito‘s, are owners, in fee • Otherwise, the land remains inalienable lands
simple, these 11 parcels of land. of the public domain.
2. Listing the applicants lot title numbers, attendant • To overcome this, an incontrovertible or clear
documents given with their respective Technical and convincing evidence must be presented.
Descriptions. • The court observed that applicants possession
were tacked only as far back as 1948 to its predecessors of
For the said application, the RTC of Pasig issued a interest –the same must have shown that their
notice of its initial hearing against the whole world predecessors in interest were in possession of the property
publishing the same at Abante Tabloid on April 5, 1989. by some 30-years back or 1938 before WW-2.

Seven days earlier, however, March 30, 1989, the The court find the petitioners failed to show it
Office of Solicitor General, filed its contention as: even as they claimed that Cirilo Menguito once declared the
1. Applicants nor its predecessor‘s in interest were neither land for tax purposes in 1943 –yet failed to show
in open, continuous, exclusive, adverse and notorious documentary evidence to support the claim.
possession or occupation of the land they applied for since The court therefore find no reason to modify the appellate
1945. court‘s decision -thus denied the petition and declared cost
2. Applicants don‘t have competent and sufficient evidence against petitioner.
of bonafide acquisition –without open, uninterrupted-
continuous, exclusive, adverse or notorious occupation of Note:
the lot in the concept of the owner and so appearing not
genuine or indicative of pretended possession. Menguito v. Republic:
3. Titling from fee simple with Spanish grant title
were not anymore available after Feb 16, 1976 as required ―Unless public land is shown to have been reclassified or
by PD 892. alienated to a private person by the State, it remains part
4. That the said parcel of land applied for is part of the inalienable public domain. Indeed, occupation
of the public domain and belonging to the Republic of the thereof in the concept of owner, no matter how long,
Philippines –and not subject to private appropriation. cannot ripen into ownership and be registered as a title.

The OSG thus stated its valid opposition on the presented


documents by the applicants leaving its prayers that said A. Homestead Settlement
application be denied and land in question be reverted to
the ownership of the Republic of the Philippines. The law provides that any citizen of the Philippines over the
age of 18 years, or the head of the family, may enter a
At the appellate court, the RTC‘s decision favoring the homestead of not exceeding 12 hectares of agricultural land
registrations of the land applied were reversed and thus of the public domain. The applicant must have cultivated
this petition. and improved at least 1/5 of the land continuously since the
approval of the application and resided for at least one year
ISSUE: WON the CA‘s did not err in its decision to reverse in the municipality in which the land is located, or in a
the trial court findings. municipality adjacent to the same, and then, upon payment
of the required fee, he shall be entitled to a patent.
HELD: No. The petition is without merit.
Effect of compliance with legal requirements
The Court cited Sec. 48 of Commonwealth Act
No. 141 as amended, provide the registration the When a homesteader has complied with all the terms and
registration of the title of lands in this wise; conditions which entitle him to a patent for a particular

Notes of Cesnee Tan Page 20


Natural Resources 1st Exam (2017-2018)

tract of public land, he acquires a vested interest therein, ISSUE: WON the homestead patent given to defendant
and is to be regarded as the equitable owner thereof. The Pineda be declared null and void
execution and delivery of the patent, after the right to a
particular parcel of land has become complete, are the RULING: In view of the fact that plaintiff was able to prove
mere ministerial acts of the officer charged with that duty. his open, continuous, exclusive possession of the disputed
Even without a patent, a perfected homestead is a property land for more than thirty years or since 1914 and that lot is
right in the fullest sense, unaffected by the fact that the at present subject of registration proceeding. Plaintiff is
paramount title to the land is still in the government. Such deemed to have acquired the lot by grant of the state, it
land may be conveyed or inherited. No subsequent law can follows that the same had ceased to part of the public and
deprive him of that vested right. had become private property and therefore beyond the
control of the Director of Land. The homestead patent
Transfer of Rights issued to defendant therefore is null and void and for
having it issued through fraud, deceit and
Section20; CA 141: If at any time after the approval of the misrepresentation. The case was remanded to the trial
application and before the patent is issued, the applicant court for further proceedings.
shall prove to the satisfaction of the Director of Lands that
he has complied with all the requirements of the law, but Susi vs Razon
cannot continue with his homestead, through no fault of his (G.R. No. L-24066 December 9, 1925)
own, and there is a bona fide purchaser for the rights and FACTS: On September 5, 1899, Valentin Susi bought a
improvements of the applicant on the land, and that the parcel of land from Apolonio Garcia and Basilio Mendoza.
conveyance is not made for purposes of speculation, then Prior to such purchase, the said sellers have been in an
the applicant, with the previous approval of the Director of open, continuous, adverse and public possession of the
Lands may transfer his rights to the land and improvements land for 19 years.
to any person legally qualified to apply for a homestead, After the purchase, Valentin Susi took possession of the
and immediately after such transfer, the purchaser shall file land in an open, continuous, adverse and public manner
a homestead application for the land so acquired and shall and without interruption. However on September 13, 1913,
succeed the original homesteader in his rights and the possession was interrupted when Angela Razon filed an
obligations beginning with the date of the approval of said action to recover possession of the land. The Court of First
application of the purchaser. Any person who has so Instance dismissed Angela Razon‘s complaint.
transferred his rights may not again apply for a new Despite the dismissal of the case, Angela Razon filed for the
homestead. Every transfer made without the previous purchase of the disputed land with the Director of Lands.
approval of the Director of Lands shall be null and void and Valentin Susi filed his opposition and asserted his right over
shall result in the cancellation of the entry and the refusal the land. However, the Director of Land overruled the
of the patent. opposition and sold the land the Angela Razon on the
ground that the land was still and public land and Valentin
Registration court cannot be divested of jurisdiction by Susi does not have a title over it.
subsequent issuance of a homestead patent over the same
land subject of registration ISSUE:

Cases: WON Valentin Susi already acquired title to the land.

Mesina vs Sonza HELD:


(GR No. L-14722, 5/25/1960)
Plaintiff Mesina claimed to be the owner of a parcel of land YES. Valentin Susi has been in possession of the land in
located in San Antonio, Nueva, Ecija. He has been in question openly, continuously, adversely and publicly
possession of the subject land openly, publicly and personally and through his predecessors for 45 years. This
peacefully since 1914. The said lot was subject of being the case, the doctrine laid down by the Supreme
registration proceedings. Surprisingly, the defendant Pineda Court of United States in Cariño vs Government of the
without knowledge of the Plaintiff was able to procure a Philippine Islands is applicable. Here, it held that To acquire
homestead patent in the same court were the registration a right to a certificate of title over a land of the public
of property was pending of the same land by the plaintiff, domain, under the provisions of Chapter VI of Act No. 926,
despite of the fact that defendant had not complied with as amended by Chapter VIII of Act No. 2874, an open,
the requirements of CA 141. That the said title was adverse, public and continuous possession from July
procured by defendants through frauds, deception and 26,1894, is sufficient, provided the possessor makes
misrepresentation since they knew that the lot belong to application therefor under the provisions of section 47 of
the plaintiff. Thus, the Plaintiff sought to annull and cancel Act No. 2874. The possessor under such circumstances
the patent issued to defendant and prayed that this acquires by operation of law, not only a right to a grant, but
registration case pending in the same court be given due a grant of the government, and the actual issuance of a
course. title is not necessary in order that said grant may be
sanctioned by the courts.

Notes of Cesnee Tan Page 21


Natural Resources 1st Exam (2017-2018)

In the present case all the requirements for a grant were Philippines, occupying lands of the public domain or
complied with since Valentin Susi has been in possession of claiming to own any such lands or an interest therein, but
the land in question openly, continuously, adversely, and whose titles have not been perfected or completed, may
publicly, personally and through his predecessors, since the apply to the Court of First Instance of the province where
year 1880, that is, for about forty-five years. By a legal the land is located for confirmation of their claims and the
fiction, Valentin Susi had acquired the land in question by a issuance of a certificate of title therefor, under the Land
grant of the State, it had already ceased to be the public Registration Act , to wit:
domain and had become private property, at least by
presumption, of Valentin Susi, beyond the control of the (a) Those who prior to the transfer of sovereignty from
Director of Lands. Consequently, in selling the land in Spain to the prior United States have applied for the
question to Angela Razon, the Director of Lands disposed of purchase, composition or other form of grant of lands of
a land over which he had no longer any title or control, and the public domain under the laws and royal decrees then in
the sale thus made was void and of no effect, and Angela force and have instituted and prosecuted the proceedings in
Razon did not thereby acquire any right. connection therewith, but have with or without default
upon their part, or for any other cause, not received title
B. Sale of Public Agricultural Land therefor, if such applicants or grantees and their heirs have
The acquisition of public agricultural lands by purchase is occupied and cultivated said lands continuously since the
governed by Chapter V (Sale) of the public land Act. Any filing of their applications.
citizen of the Philippines of lawful age or the head of a
family may purchase any tract of public agricultural land not (b) Those who by themselves or through their predecessors
to exceed 12 hectares which shall be sold through sealed in interest have been in open, continuous, exclusive, and
bidding. The land shall be awarded to the highest bidder, notorious possession and occupation of agricultural lands of
but the applicant may equal the highest bid. The purchase the public domain, under a bona fide claim of acquisition or
price may be paid in full upon the making of the award or ownership, for at least thirty years immediately preceding
in not more than ten equal annual instalments from the the filing of the application for confirmation of title except
date of the award. It is required that the purchase shall when prevented by war or force majeure. These shall be
have not less than 1/5 of the land cultivated within 5 years conclusively presumed to have performed all the conditions
from the date of the award, and before any patent is essential to a Government grant and shall be entitled to a
issued, he must show actual occupancy, cultivation, and certificate of title under the provisions of this chapter.
improvement of at least 1/5 of the land until the date of
final payment. (c) Members of the national cultural minorities who by
themselves or through their predecessors-in-interest have
C. Lease been in open, continuous, exclusive and notorious
possession and occupation of lands of the public domain
Any citizen of lawful age of the Philippines and any suitable to agriculture, whether disposable or not, under a
corporation or association of which at least 60 per centum bona fide claim of ownership for at least 30 years shall be
of the capital stock of any interest in said capital stock entitled to the rights granted in sub-section (b) hereof.
belongs wholly to citizens of the Philippines, may lease any
tract of agricultural public land available for lease under Ownership based on adverse possession
Chapter VI (Lease) of the Act. Private corporations or
associations may not hold such alienable lands of the public As a rule, no title or right to, or equity in, any lands of the
domain except by lease, for a period not exceeding twenty- public domain may be acquired by prescription or by
five years, renewable for not more than twenty-five years, adverse possession or occupancy except as expressly
and not to exceed one thousand hectares in area. Citizens provided by law. However, the public land Act recognized
of the Philippines may lease not more than five hundred the concept of ownership under the civil law. This
hectares, or acquire not more than twelve hectares thereof, ownership is based on adverse possession and the right of
by purchase, homestead, or grant. acquisition is governed by Chapter VIII on judicial
confirmation of imperfect or incomplete titles. The applicant
D. Confirmation of imperfect or incomplete title must prove that:
(Judicial Legalization or Administrative 1. The land is alienable public land
Legalization) 2. His possession and occupation has been exercised
The confirmation of imperfect or incomplete titles may be in the manner and for the period prescribed by law,
done in 2 ways: or since June 12, 1945. Registration under the Act
1. Judicial legalization or judicial confirmation of presumes that the land was originally public
imperfect or incomplete titles under Chater VIII of agricultural land but because of adverse possession
the Public land Act since June 12, 1945, the land has become private.
2. Administrative legalization or free patents under
Chapter VII of the same act. Period of Possession

Section48. The following-described citizens of the RA 1942 – possession of at least 30 years immediately

Notes of Cesnee Tan Page 22


Natural Resources 1st Exam (2017-2018)

preceding the filing of the application. Only applications was purchased by him and his wife, Adela Raz from one
filed prior to January 25, 1977 may invoke RA 1942. Eulalio Raz. The documents attached to the application are:
technical description, surveyor's certificate, certification by
PD 1073 (repealed ra 1942) – possession since June 12, the chief deputy assessor of Aklan and the blue print of
1945 or earlier. Psu-161277. The land applied for is residential.
a. Also extended the period for the filing of The initial hearing was held on and an order of general
applications for judicial confirmation of imperfect default was issued but those who presented their
and incomplete titles to December 31, 1987 opposition, namely, Octabela Alba Vda. De Raz, Manuel and
b. Limited the area of the land applied to 144 hectares Susana Braulio, Jose Rago, representing Apolonia Rebeco.
c. Repealed section 48 (b) on judicial confirmation of Petitioners contended that they were the real owners of the
incomplete titles to public land based on subject land, which they have been possession
unperfected Spanish grants continuously, openly and peacefully under claim of
d. Amended Sections 48 (b) and (c) in the sense that ownership for not less than 70 years together with their
these provisions shall apply only to alienable and predecessor-in-interest, and the applicant is only entitled to
disposable lands of the public domain which have have the portion of 620 square meters which is contrary to
been in open, continuous, exclusive and notorious his application of 4,845 square meters.
possession and occupation by the applicant himself The trial court finds that Dr. Jose Lachica as the absolute
or through his predecessors in interest, under a owner in fee simple of the land described in his application
bona fide claim of acquisition of ownership since for its original registration in his name.
June 12, 1945.
Issue:
RA 9176 has extended period of filing to December 31, WON the private respondent/applicant is entitled to the
2020 confirmation of his ownership in fee simple for the 4, 845
1. Extended the period to file and application for square meter parcel of land he applied for.
judicial confirmation of imperfect or incomplete
titles to December 21, 2020 Held: NO
2. Further limiting the area applied for to 12 hectares In Section 48 of Commonwealth Act 141, (which is the law
3. Providing that all pending applications filed before enforced during the filing of application) as amended by RA
the effectivity of the amendatory act shall be Nos. 1942 and 6236, which states that:
treated as having been filed in accordance with the Sec. 48. The following-described citizens of the Philippines,
provisions thereof. occupying lands of the public domain or claiming to own
any such lands or an interest therein, but whose titles have
Requisites for availment of Chapter VIII not been perfected or completed, may apply to the Court of
1. Must be a Filipino citizen First Instance of the province where the land is located for
2. He must have, by himself or through his confirmation of their claim and issuance of a certificate of
predecessors in interest, possessed and occupied title therefor, under the Land Registration Act, to wit:
an alienable and disposable agricultural portion of (a) Those who prior to the transfer of sovereignty from
the public domain Spain to the United States have applied for the purchase,
3. The possession is open, continuous, exclusive, composition or other form of grant of lands of the public
notorious and in the concept of an owner since domain under the laws and royal decrees then in force and
June 12, 1945 have instituted and prosecuted the proceedings in
4. The application must be filed with the proper court connection therewith, but have with or without default
upon their part, or for any other cause, not received title
Only when these conditions are met may the possessor of therefor, if such applicants or grantees and their heirs have
the land acquire, by operation of law, a right to a grant, a occupied and cultivated said lands continuously since the
government grant, without the necessity of a certificate of filing of their applications. 49
title being issued. The application for confirmation or (b) Those who by themselves or through their predecessors
imperfect title must be filed in accordance with the in interest have been in open, continuous, exclusive and
procedural requirements specified in Section 50 of the notorious possession and occupation of agricultural lands of
Property Registration Decree (PD 1529). the public domain under a bona fide claim of ownership, for
at least thirty years immediately preceding the filing of the
Land must be alienable and disposable at the time the application for confirmation of title except when prevented
application for confirmation is filed by war or force majeure. These shall be conclusively
presumed to have performed all the conditions essential to
Cases: a Government grant and shall be entitled to a certificate of
title under the provisions of this chapter. 50
Alba vs Court of Appeals (c) Members of the national cultural minorities who by
(GR No. 120066, 9/9/1999) themselves or through their predecessors-in-interest have
Applicant Jose Lachica filed this application for title to land been in open, continuous, exclusive and notorious
on April 28, 1958 with the claim that the land applied for possession and occupation of lands of the public domain

Notes of Cesnee Tan Page 23


Natural Resources 1st Exam (2017-2018)

suitable to agriculture, whether disposable or not, under a Naguit and her predecessors-in-interest had occupied the
bona fide claim of ownership for at least 30 years shall be land openly and in the concept of owner without any
entitled to the rights granted in subsection (b) hereof. 51 objection from any private person or even the government
A circumspect scrutiny of the assailed Decision readily until she filed her application for registration.
shows that in the affirming the ruling of the trial court, the
Court of Appeals relied on the provisions of Section 19 of The OSG argued that the property which is in open,
Act 496 52 in relation to the Civil Code's provision's on continuous and exclusive possession must first be alienable.
prescription on the assumption that the subject land is Since the subject land was declared alienable only on
private land. Therein lies the flaw in the appellate court's October 15, 1980, Naguit could not have maintained a bona
postulate. The application for registration of private fide claim of ownership since June 12, 1945, as required by
respondent is for judicial confirmation of an imperfect title Section 14 of the Property Registration Decree, since prior
considering that the land is presumed under the Regalian to 1980, the land was not alienable or disposable.
Doctrine to be part of the public domain.
The private respondent failed to satisfy the Supreme Court The OSG suggested an interpretation that all lands of the
that he complied the condition set forth for the judicial public domain which were not declared alienable or
confirmation of his title for the following reasons, to wit; disposable before June 12, 1945 would not be susceptible
a. as to the (a) of Section 48 of CA 141- respondent did to original registration, no matter the length of
present any imperfect title with Spanish decree. unchallenged possession by the occupant.
b. as to the (b) of Section 48 of CA 141- prescription
cannot be applied as there were no express grant from the Issue: Whether or not it is necessary under Section 14(1) of
government that the subject land was amongst the the Property Registration Decree that the subject land be
agricultural land certified as alienable and disposable. first classified as alienable and disposable before the
Ultimately, No public land can be acquired by private applicant‘s possession under a bona fide claim of ownership
persons without any grant, express or implied from the could even start.
government; it is indispensable that there be a showing of
title from the state. Held: Section 14 of the Property Registration Decree,
c. nor he is members as specified on the (c) condition of CA governing original registration proceedings, provides:
141.
SECTION 14. Who may apply.— The following persons may
This case discussed that it is important that the land is file in the proper Court of First Instance an application for
declared alienable and disposable at the time of the filing of registration of title to land, whether personally or through
application. their duly authorized representatives:

Republic vs Court of Appeals and Naguit (1) those who by themselves or through their predecessors-
(GR No. 1/17/2005) in-interest have been in open, continuous, exclusive and
On January 5, 1993, Naguit filed a petition for registration notorious possession and occupation of alienable and
of title of a parcel of land. The application sought a judicial disposable lands of the public domain under a bona fide
confirmation of imperfect title over the land. claim of ownership since June 12, 1945, or earlier.
(2) Those who have acquired ownership over private lands
The public prosecutor, appearing for the government, and by prescription under the provisions of existing laws.
Angeles opposed the petition. The court issued an order of
general default against the whole world except as to There are three obvious requisites for the filing of an
Angeles and the government. application for registration of title under Section 14(1) –
that the property in question is alienable and disposable
The evidence revealed that the subject parcel of land was land of the public domain; that the applicants by
originally declared for taxation purposes in the name of themselves or through their predecessors-in-interest have
Urbano in 1945. Urbano executed a Deed of Quitclaim in been in open, continuous, exclusive and notorious
favor of the heirs of Maming, wherein he renounced all his possession and occupation, and; that such possession is
rights to the subject property and confirmed the sale made under a bona fide claim of ownership since June 12, 1945
by his father to Maming sometime in 1955 or 1956. or earlier.
Subsequently, the heirs of Maming executed a deed of
absolute sale in favor of respondent Naguit who thereupon The OSG's interpretation would render paragraph (1) of
started occupying the same. Section 14 virtually inoperative and even precludes the
government from giving it effect even as it decides to
Naguit constituted Blanco, Jr. as her attorney-in-fact and reclassify public agricultural lands as alienable and
administrator. The administrator introduced improvements, disposable. The unreasonableness of the situation would
planted trees in addition to existing coconut trees which even be aggravated considering that before June 12, 1945,
were then 50 to 60 years old, and paid the corresponding the Philippines was not yet even considered an independent
taxes due on the subject land. state.

Notes of Cesnee Tan Page 24


Natural Resources 1st Exam (2017-2018)

The more reasonable interpretation of Section 14(1) is that earlier. He also did not qualify under Section 14(2) because
it merely requires the property sought to be registered as the prescriptive period only began to run upon the
already alienable and disposable at the time the application declaration that the land was no longer needed for public
for registration of title is filed. If the State, at the time the use and public purpose. So, even if he occupied the land for
application is made, has not yet deemed it proper to a very long time, he is still not qualified because yung
release the property for alienation or disposition, the recognized lang ng law, the period of possession na
presumption is that the government is still reserving the nacover nya is only 2 years from the date of the
right to utilize the property; hence, the need to preserve its declaration.
ownership in the State irrespective of the length of adverse
possession even if in good faith. However, if the property Now, Section 14 (1) and Section 14 (2) that is what you call
has already been classified as alienable and disposable, as as Judicial Confirmation of Imperfect Title.
it is in this case, then there is already an intention on the
part of the State to abdicate its exclusive prerogative over Heirs of Malabanan v. Republic,
the property. G.R. No. 179987, [April 29, 2009], 605 PHIL 244-
326)
In this case, the 3 requisites for the filing of registration of On 20 February 1998, Mario Malabanan filed an application
title under Section 14(1) had been met by Naguit. The for land registration before the RTC of Cavite-Tagaytay,
parcel of land had been declared alienable; Naguit and her covering a parcel of land situated in Silang Cavite,
predecessors-in-interest had been in open, continuous, consisting of 71,324 square meters. Malabanan claimed
exclusive and notorious possession and occupation of the that he had purchased the property from Eduardo Velazco,
land evidenced by the 50 to 60-year old trees at the time and that he and his predecessors-in-interest had been in
she purchased the property; as well as the tax declarations open, notorious, and continuous adverse and peaceful
executed by the original owner Urbano in 1954, which possession of the land for more than thirty (30) years.
strengthened one's bona fide claim of ownership. Velazco testified that the property originally belonged to a
twenty-two hectare property owned by his great-
Now we go to the favourite Bar question. This case grandfather, Lino Velazco. Lino had four sons– Benedicto,
discussed the main difference between Section 14(1) and Gregorio, Eduardo and Esteban–the fourth being Aristedes‘s
Section 14(2) of PD 1529. Section 14(1) is very similar to grandfather. Upon Lino‘s death, his four sons inherited the
Section 48(b) of CA 141. However, the subsequent property and divided it among themselves. But by 1966,
enactments of the law say the availing of Section 48(b) can Esteban‘s wife, Magdalena, had become the administrator
only be availed until December 31, 2013. of all the properties inherited by the Velazco sons from their
father, Lino. After the death of Esteban and Magdalena,
What are the requirements of Section 14(1)? There are 3. their son Virgilio succeeded them in administering the
1. Possession by themselves or thru their properties, including Lot 9864-A, which originally belonged
predecessors in interest under a bone fide to his uncle, Eduardo Velazco. It was this property that was
ownership since June 12, 1945 or earlier; sold by Eduardo Velazco to Malabanan.
2. Possession and ownership since June 12, 1945 or
earlier; Among the evidence presented by Malabanan during trial
3. Land is declared alienable and disposable at the was a Certification dated 11 June 2001, issued by the
time of the filing of the application Community Environment & Natural Resources Office,
Department of Environment and Natural Resources
Now, if you are not qualified, just look at the period to (CENRO-DENR), which stated that the subject property was
determine what to apply; either Section 14(1) or Section ―verified to be within the Alienable or Disposable land per
14(2). If the possession is earlier than June 12, 1945, you Land Classification Map No. 3013 established under Project
apply Section 14 (1). If you are not qualified under that, No. 20-A and approved as such under FAO 4-1656 on
look at Section 14 (2). March 15, 1982.‖ On 3 December 2002, the RTC approved
the application for registration.
These are the requirements under Section 14 (2):
1. Land is patrimonial property of the State. Meaning, The Republic interposed an appeal to the Court of Appeals,
no longer intended for public use or public purpose. arguing that Malabanan had failed to prove that the
2. there must be an express declaration that it is no property belonged to the alienable and disposable land of
longer needed for public use and public purpose the public domain, and that the RTC had erred in finding
3. Period of possession. 30 years, without good faith that he had been in possession of the property in the
and just title. 10 years, with good faith and just manner and for the length of time required by law for
title. This descriptive period will only begin to run confirmation of imperfect title. On 23 February 2007, the
at the time it was declared no longer needed for Court of Appeals reversed the RTC ruling and dismissed the
public use and public purpose. appliocation of Malabanan.
ISSUES:
In this case, he did not qualify applying Section 14(1)
because the tax declarations do not cover June 12, 1945 1. In order that an alienable and disposable land of the

Notes of Cesnee Tan Page 25


Natural Resources 1st Exam (2017-2018)

public domain may be registered under Section 14(1) of must also be an express government manifestation that the
Presidential Decree No. 1529, otherwise known as the property is already patrimonial or no longer retained for
Property Registration Decree, should the land be classified public service or the development of national wealth, under
as alienable and disposable as of June 12, 1945 or is it Article 422 of the Civil Code. And only when the property
sufficient that such classification occur at any time prior to has become patrimonial can the prescriptive period for the
the filing of the applicant for registration provided that it is acquisition of property of the public dominion begin to run.
established that the applicant has been in open,
continuous, exclusive and notorious possession of the land (a) Patrimonial property is private property of the
under a bona fide claim of ownership since June 12, 1945 government. The person acquires ownership of patrimonial
or earlier? property by prescription under the Civil Code is entitled to
secure registration thereof under Section 14(2) of the
2. For purposes of Section 14(2) of the Property Property Registration Decree.
Registration Decree may a parcel of land classified as
alienable and disposable be deemed private land and (b) There are two kinds of prescription by which patrimonial
therefore susceptible to acquisition by prescription in property may be acquired, one ordinary and other
accordance with the Civil Code? extraordinary. Under ordinary acquisitive prescription, a
person acquires ownership of a patrimonial property
3. May a parcel of land established as agricultural in through possession for at least ten (10) years, in good faith
character either because of its use or because its slope is and with just title. Under extraordinary acquisitive
below that of forest lands be registrable under Section prescription, a person‘s uninterrupted adverse possession of
14(2) of the Property Registration Decree in relation to the patrimonial property for at least thirty (30) years,
provisions of the Civil Code on acquisitive prescription? regardless of good faith or just title, ripens into ownership.
It is clear that the evidence of petitioners is insufficient to
4. Are petitioners entitled to the registration of the subject establish that Malabanan has acquired ownership over the
land in their names under Section 14(1) or Section 14(2) of subject property under Section 48(b) of the Public Land
the Property Registration Decree or both? Act. There is no substantive evidence to establish that
Malabanan or petitioners as his predecessors-in-interest
HELD: have been in possession of the property since 12 June 1945
or earlier. The earliest that petitioners can date back their
The Petition is denied. possession, according to their own evidence—the Tax
Declarations they presented in particular—is to the year
(1) In connection with Section 14(1) of the Property 1948. Thus, they cannot avail themselves of registration
Registration Decree, Section 48(b) of the Public Land Act under Section 14(1) of the Property Registration Decree.
recognizes and confirms that ―those who by themselves or
through their predecessors in interest have been in open, Neither can petitioners properly invoke Section 14(2) as
continuous, exclusive, and notorious possession and basis for registration. While the subject property was
occupation of alienable and disposable lands of the public declared as alienable or disposable in 1982, there is no
domain, under a bona fide claim of acquisition of competent evidence that is no longer intended for public
ownership, since June 12, 1945‖ have acquired ownership use service or for the development of the national
of, and registrable title to, such lands based on the length evidence, conformably with Article 422 of the Civil Code.
and quality of their possession. The classification of the subject property as alienable and
disposable land of the public domain does not change its
(a) Since Section 48(b) merely requires possession since 12 status as property of the public dominion under Article
June 1945 and does not require that the lands should have 420(2) of the Civil Code. Thus, it is insusceptible to
been alienable and disposable during the entire period of acquisition by prescription.
possession, the possessor is entitled to secure judicial
confirmation of his title thereto as soon as it is declared Heirs of Malabanan v. Republic,
alienable and disposable, subject to the timeframe imposed G.R. No. 179987, [September 3, 2013
by Section 47 of the Public Land Act. This case is a resolution of the Motions for Reconsideration,
filed by the parties who both assail the decision of the
(b) The right to register granted under Section 48(b) of the Court promulgated on April 29, 2009. In the decision, the
Public Land Act is further confirmed by Section 14(1) of the Court upheld the ruling of the Court of Appeals which
Property Registration Decree. denied the application of the petitioners for the registration
of a parcel of land situated in Barangay Tibig, Silang,
(2) In complying with Section 14(2) of the Property Cavite, on the ground that they had not established by
Registration Decree, consider that under the Civil Code, sufficient evidence their right to the registration in
prescription is recognized as a mode of acquiring ownership accordance with either section 14 (1) or 14 (2) of the
of patrimonial property. However, public domain lands Property Registration Decree (P.D. 1529).
become only patrimonial property not only with a
declaration that these are alienable or disposable. There On February 20, 1998, applicant Mario Malabanan, who had

Notes of Cesnee Tan Page 26


Natural Resources 1st Exam (2017-2018)

purchased the property from Eduardo Velazco, filed an Constitution: lands of the public domain were classified into
application for land registration covering the property in the - agricultural, timber and mineral.
RTC in Tagaytay City, Cavite, claiming that the property Section 10, Article XIV of the 1973 Constitution: lands of
formed part of the alienable and disposable land of the the public domain were classified into - agricultural,
public domain, and that he and his predecessors-in-interest industrial or commercial, residential, resettlement, mineral,
had been in open, continuous, uninterrupted, public and timber or forest, and grazing land, with the reservation that
adverse possession and occupation of the land for more the law might provide other classifications.
than 30 years, thereby entitling him to the judicial
confirmation of his title. To prove such, Malabanan 1987 Constitution adopted the classification under the 1935
presented during trial a certification issued by the Constitution into agricultural, forest or timber, and mineral,
Community Environment and Natural Resources Office but added national parks. Under Section 2, Article XII of the
(CENRO) of the DENR. The RTC rendered judgment 1987 Constitution, only agricultural lands of the public
granting Malabanan‘s application for land registration. The domain may be alienated; all other natural resources may
Office of the Solicitor General (OSG) appealed the not be.
judgment to the CA, arguing that Malabanan had failed to Alienable and disposable lands of the State fall into two
prove that the property belonged to the alienable and categories:
disposable land of the public domain, and that the RTC (a) patrimonial lands of the State, or those classified as
erred in finding that he had been in possession of the lands of private ownership under Article 425 of the Civil
property in the manner and for the length of time required Code, without limitation; and
by law for confirmation of imperfect title. The CA (b) lands of the public domain, or the public lands as
promulgated its decision reversing the RTC and dismissing provided by the Constitution, but with the limitation that
the application for registration of Malabanan. Citing the the lands must only be agricultural. Consequently, lands
ruling in Republic v. Herbieto (Herbieto), the CA declared classified as forest or timber, mineral, or national parks are
that under Section 14(1) of the Property Registration not susceptible of alienation or disposition unless they are
Decree, any period of possession prior to the classification reclassified as agricultural. A positive act of the Government
of the land as alienable and disposable was is necessary to enable such reclassification, and the
inconsequential and should be excluded from the exclusive prerogative to classify public lands under existing
computation of the period of possession. Noting that the laws is vested in the Executive Department, not in the
CENRO-DENR certification stated that the property had courts. If, however, public land will be classified as neither
been declared alienable and disposable only on March 15, agricultural, forest or timber, mineral or national park, or
1982, Velazco‘s possession prior to March 15, 1982 could when public land is no longer intended for public service or
not be tacked for purposes of computing Malabanan‘s for the development of the national wealth, thereby
period of possession. effectively removing the land from the ambit of public
dominion, a declaration of such conversion must be made
DISCUSSION: in the form of a law duly enacted by Congress or by a
Classifications of land according to ownership Land may be Presidential proclamation in cases where the President is
classified as either of public dominion or of private duly authorized by law to that effect.
ownership. Disposition of alienable public lands Section 11 of the Public
Land Act (CA No. 141) provides the manner by which
It is of public dominion if it: alienable and disposable lands of the public domain, i.e.,
(a) is intended for public use; or agricultural lands, can be disposed of:
(b) belongs to the State, without being for public use, and (1) For homestead settlement; (2) By sale; (3) By lease;
is intended for some public service or for the development and (4) By confirmation of imperfect or incomplete titles;
of the national wealth. Land belonging to the State that is (a) By judicial legalization; or (b) By administrative
not of such character, or although of such character but no legalization (free patent).
longer intended for public use or for public service forms The core of the controversy herein lies in the proper
part of the patrimonial property of the State. Land that is interpretation of Section 11(4), in relation to Section 48(b)
other than part of the patrimonial property of the State, of the Public Land Act, which expressly requires possession
provinces, cities and municipalities is of private ownership if by a Filipino citizen of the land since June 12, 1945, or
it belongs to a private individual. earlier. Bearing in mind such limitations under the Public
Land Act, the applicant must satisfy the following
Pursuant to the Regalian Doctrine (Jura Regalia), all lands requirements in order for his application to come under
of the public domain belong to the State. All lands not Section 14(1) of the Property Registration Decree, to wit:
appearing to be clearly under private ownership are 1. The applicant, by himself or through his predecessor-in-
presumed to belong to the State. Also, public lands remain interest, has been in possession and occupation of the
part of the inalienable land of the public domain unless the property subject of the application;
State is shown to have reclassified or alienated them to 2. The possession and occupation must be open,
private persons. continuous, exclusive, and notorious;
3. The possession and occupation must be under a bona
Classifications of public lands according to alienability 1935 fide claim of acquisition of ownership;

Notes of Cesnee Tan Page 27


Natural Resources 1st Exam (2017-2018)

4. The possession and occupation must have taken place or for the development of the national wealth.
since June 12, 1945, or earlier; and PETITION DENIED
5. The property subject of the application must be an
agricultural land of the public domain. Rule different where land is not registrable as when it forms
In sum, these are the rules relative to the disposition of part of the public forest
public land or lands of the public domain,
namely: It is elementary in the law governing natural resources that
(1) As a general rule and pursuant to the Regalian Doctrine, forest land cannot be owned by private persons. It is not
all lands of the public domain belong to the State and are registrable and possession thereof, no matter how lengthy,
inalienable. Lands that are not clearly under private cannot convert it to private property, unless such lands are
ownership are also presumed to belong to the State and, reclassified and considered disposable and alienable.
therefore, may not be alienated or disposed;
(2) Exceptions: The applicant must secure a certification from the
(a) Agricultural lands of the public domain are rendered government that the land applied for by the applicant is
alienable and disposable through any of the exclusive alienable and disposable. This can be established by a
modes enumerated under Section 11 of the Public Land presidential proclamation, or an executive order, an
Act. If the mode is judicial confirmation of imperfect title administrative action, investigation reports of lands or
under Section 48(b) of the Public Land Act, the agricultural forest investigators or legislative acts or statue.
land subject of the application needs only to be classified
as alienable and disposable as of the time of the
application, provided the applicant‘s possession and
occupation of the land dated back to June 12, 1945, or Palomo vs CA
earlier. Thereby, a conclusive presumption that the (GR No. 95608, 1/21/1997)
applicant has performed all the conditions essential to a FACTS: In 1913, some 440,530 square meters of land in
government grant arises, and the applicant becomes the Albay were reserved for provincial park purposes by virtue
owner of the land by virtue of an imperfect or incomplete of EO No. 40. Of said area, 15 parcels of land were
title. By legal fiction, the land has already ceased to be part registered in the name of Diego Palomo by the Court of
of the public domain and has become private property. First Instance.
(b) Lands of the public domain subsequently classified or
declared as no longer intended for public use or for the In 1937, Diego Palomo donated these lands to his heirs
development of national wealth are removed from the Ignacio and Carmen Palomo. Claiming that the aforesaid
sphere of public dominion and are considered converted original certificates of title were lost during the Japanese
into patrimonial lands or lands of private ownership that occupation, Ignacio Palomo filed a petition for
may be alienated or disposed through any of the modes of reconstitution with the Court of First Instance of Albay on
acquiring ownership under the Civil Code. If the mode of May 30, 1950. The Register of Deeds of Albay issued
acquisition is prescription, whether ordinary or Transfer Certificates of Title Nos. 3911, 3912, 3913 and
extraordinary, proof that the land has been already 3914 sometime in October 1953.
converted to private ownership prior to the requisite
acquisitive prescriptive period is a condition sine qua non in On July 10, 1954, President Magsaysay issued Proclamation
observance of the law (Article 1113, Civil Code) No. 47 converting the area covered by EO 40 into the Tiwi
that property of the State not patrimonial in character shall Hot Spring National Park. The Palomos contended that they
not be the object of prescription. have been in possession of the subject lands and have
introduced improvements thereon.
In the case at bar, the petitioners failed to present sufficient
evidence to establish that they and their predecessors-in- ISSUE: Were the Original Certificate of Titles issued to the
interest had been in possession of the land since June 12, petitioners valid? –NO
1945.
Without satisfying the requisite character and period of HELD
possession - possession and occupation that is open, Before the Treaty of Paris in 1899, the lands, whether
continuous, exclusive, and notorious since June 12, 1945, agricultural, mineral, or forest were under the exclusive
or earlier - the land cannot be considered ipso jure patrimony and dominion of the Spanish crown.Private
converted to private property even upon the subsequent ownership of land could only be acquired through royal
declaration of it as alienable and disposable. Prescription concessions which were documented in various forms, such
never began to run against the State, such that the land as (1) Titulo Real or Royal Grant," (2) Concession Especial
has remained ineligible for registration under Section 14(1) or Special Grant, (3) Titulo de Compra or Title by Purchase
of the Property Registration Decree. Likewise, the land and (4) Informacion Posesoria or Possessory Information
continues to be ineligible for land registration under Section title obtained under the Spanish Mortgage Law or under the
14(2) of the Property Registration Decree unless Congress Royal Decree of January 26, 1889.
enacts a law or the President issues a proclamation
declaring the land as no longer intended for public service The decision of the CFI relied upon by petitioners were not

Notes of Cesnee Tan Page 28


Natural Resources 1st Exam (2017-2018)

signed by the judge but were merely certified copies of hand, it is the respondents' position that since the subject
notification to Diego Palomo bearing the signature of the parcels of land were only classified as alienable or
Clerk of Court. disposable on March 27, 1972, petitioner did not have any
title to confirm when he filed his application in 1963.
More importantly, the lands in question were not classified Neither was the requisite thirty years possession met.
as alienable lands. Since the lands were made part of a
reservation for provincial park purposes, they form part of A similar situation in the case of Reyes v. Court of Appeals,
the forest zone. Thus, they cannot be the valid subject of where a homestead patent issued to the petitioners'
alienation. predecessor-in-interest was cancelled on the ground that at
the time it was issued, the subject land was still part of the
Republic vs Bracewell public domain.
[G.R. No. 107427. January 25, 2000]
Facts: The controversy involves a total of nine thousand six In the said case, this Court ruled as follows —
hundred fifty-seven (9,657) square meters of land located Under the Regalian doctrine, all lands of the public domain
in Las Piñas, Metro Manila. belong to the State, and that the State is the source of any
In 1908, Maria Cailles, married to James Bracewell, Sr., who asserted right to ownership in land and charged with the
acquired the said parcels of land from the Dalandan and conservation of such patrimony. This same doctrine also
Jimenez families of Las Piñas; after which corresponding states that all lands not otherwise appearing to be clearly
Tax Declarations were issued in the name of Maria Cailles. within private ownership are presumed to belong to the
State (Director of Lands vs. Intermediate Appellate Court,
On January 16, 1961, Maria Cailles sold the said parcels of 219 SCRA 340).
land to her son, the petitioner, by virtue of a Deed of Sale
which was duly annotated and registered with the Registry Hence, the burden of proof in overcoming the presumption
of Deeds of Pasig, Rizal. Tax Declarations were thereafter of State ownership of lands of the public domain is on the
issued in the name of petitioner, cancelling the previous Tax person applying for registration. The applicant must show
Declarations issued to Maria Cailles. that the land subject of the application is alienable or
disposable. These petitioners failed to do.
On September 19, 1963, petitioner filed before the then
Court of First Instance of Pasig, Rizal an action for The homestead patent was issued to petitioners'
confirmation of imperfect title under Section 48 of predecessor-in-interest, the subject land belong to the
Commonwealth Act No. 141. inalienable and undisposable portion of the public domain.
Thus, any title issued in their name by mistake or oversight
The Director of Lands, represented by the Solicitor General, is void ab initio because at the time the homestead patent
opposed petitioner's application on the grounds that neither was issued to petitioners, as successors-in-interest of the
he nor his predecessors-in-interest possessed sufficient title original patent applicant, the Director of Lands was not
to the subject land nor have they been in open, continuous, then authorized to dispose of the same because the area
exclusive and notorious possession and occupation of the was not yet classified as disposable public land.
same for at least thirty (30) years prior to the application, Consequently, the title issued to herein petitioners by the
and that the subject land is part of the public domain. Bureau of Lands is void ab initio.

On May 3, 1989, the lower court issued an Order granting Neither has petitioner shown proof that the subject Forestry
the application of petitioner. The Solicitor General promptly Administrative Order recognizes private or vested rights
appealed to respondent Court which, on June 29, 1992, under which his case may fall. We only find on record the
reversed and set aside the lower court's Order. It also Indorsement of the Bureau of Forest Development from
denied petitioner's Motion for Reconsideration in its which no indication of such exemption may be gleaned.
Resolution of September 30, 1992.
Having found petitioner to have no cause of action for his
Issues: a) Whether or not the petitioner failed to prosecute application for confirmation of imperfect title, we see no
his action for an unreasonable length of time? need to discuss the other errors raised in this petition.

b) Whether the tax declarations attached to the complaint Republic vs Court of Appeals and Bernabe
do not constitute acquisition of the lands applied for? (GR No. L-40402, 3/16/1987)
FACTS: Lot No. 622 of the Mariveles Cadastre was declared
Held: The controversy is simple. On one hand, petitioner public land in a decision rendered before the last war in
asserts his right of title to the subject land under Section 48 Cadastral Case No. 19, LRC Cadastral Record No. 1097. On
(b) of Commonwealth Act No. 141, having by himself and July 6, 1965 such lot was segregated from the forest zone
through his predecessors-in-interest been in open, and released and certified by the Bureau of Forestry (BOF)
continuous, exclusive and notorious possession and as an agricultural Land for disposition under the Public Land
occupation of the subject parcels of land, under a bona fide Act. On April 26, 1967, Respondents filed in the CFI of
claim of acquisition or ownership, since 1908. On the other Bataan a petition to reopen Cadastral Case No. 19 to

Notes of Cesnee Tan Page 29


Natural Resources 1st Exam (2017-2018)

perfect their rights and register their titles to said lots. They FACTS: Apolonio Bautista Jr. acquired lot 17078 through
alleged that they acquired ownership and possession of succession, when his father Apolonio Sr. died in 1987. He
said parcels of land by purchase from the original owners applied judicial confirmation of imperfect title before MTC
thereof, whose possession of the same including that of the with testimonial evidence that his father was in possession
herein respondents, has always been continuous, open, of the said lot since 1969 and acquired it from Jardin and
active, exclusive, public, adverse and in the concept of Villanueva, through notarized Deed of Absolute Sale dated
owners for more than 30 years. The Director of Forestry February 15, 1971 and May 25, 1973 respectively. The MTC
filed an opposition to the above petition but later withdrew granted the application and declared him as the owner in
the same upon verification of findings that this portion of fee simple of the said land.
the timberland had already been released from the mass of The Government of the Philippines appealed before CA,
the public forests. Subsequently, the Acting Prov. Fiscal of contending that the testimony of Apolonio Jr. is based on
Bataan, for and in behalf of the Director of Lands filed his hearsay which should not be given probative value. The
opposition alleging that the land is still a Public Land and as application of judicial confirmation of imperfect title must
such cannot be the subject of a land registration comply with Sec. 48(b) of CA 141. The CA affirmed the
proceeding under Act 496. The lower court adjudicated in ruling of MTC, it pointed out that the Government of the
favor or respondent Bernabes, finding that the latter have Phil. did not timely object to the evidence presented.
complied with all the terms and conditions entitling them to ISSUE: W/N the grant of judicial confirmation of imperfect
a grant. This decision having become final, the title to Apolonio Jr proper?
Commissioner of Land Registration issued the
corresponding decrees of registration. On the other hand, RULING: The Supreme Court reversed the ruling of MTC
petitioner DL through the Solicitor Gen. filed a petition for and CA. It held that the requisite period of possession must
review of the decrees. Afterwards, he filed an Amended conform with Sec 48(b) of Public Land Act as amended by
Petition for Review, adding: that respondents executed RA 1942 which provides that any person who applies for
simulated deeds of sale conveying portions of the subject judicial confirmation, he or his predecessor in interest must
parcels to third parties for fictitious considerations in order have been in open, continuous, exclusive and notorious
to remove the same from the coverage of Sec. 38 of Act possession and occupation of alienable and disposable land
496, but in truth, buyers are mere dummies of petitioners; of public domain under the bonafide claim of ownership at
hence, not purchasers for value. The Court of First least 30 years since June 12, 1945 or earlier.
Instance denied this petition and on appeal, the CA
affirmed the questioned decision. Petitioner‘s Motion for In the present case, the lower court relied only on the
Reconsideration having been denied for lack of merit; testimony of Bautista Jr. that his father acquired the land
hence, this petition. and in possession since 1969. He failed to prove that the
transferor had in open, continuous, exclusive and notorious
ISSUE: WON the lots claimed by respondents could be possession of said land for at least 30 years since June 12,
legally be the subject of a juridical confirmation of Title 1945 or earlier. Bautista Jr. has no personal knowledge of
under Section 48 (b) of Commonwealth Act 141 as these facts. Lack of this evidence does not give the court
amended by Republic Act 1942. the right to grant a judicial confirmation of imperfect title in
favor of Bautista Jr.
HELD: No. The Supreme Court ruled that Sec. 48 (b) of CA
141, as amended, applies exclusively to public lands. E. Non-Registrable Properties
Forest lands or areas covered with forests are excluded.
Thus, possession of forest lands, however long cannot ripen The following cannot be the subject of registration:
into private ownership. A parcel of forest land is within the 1. Property of public dominion
exclusive jurisdiction of the Bureau of Forestry and beyond 2. Forest lands
the power and jurisdiction of the cadastral court to register 3. Watersheds
under the Torrens System. Thus, even if the reopening of 4. Mangrove swamps
the cadastral proceedings was at all possible, private 5. Mineral Lands
respondents have not qualified for a grant under Section 48 6. National Parks and protected areas
(b) of CA 141. They can only be credited with 1 year, 9 7. Military or naval protection
mos. and 20 days of possession and occupation of the lots 8. Foreshore lands and reclaimed lands
involved, counted from July 6, 1965 when the lots involved 9. Submerged lands
had been segregated from the forest zone and released by 10. Lakes
the BOF as an agricultural land for disposition under the 11. Navigable rivers
Public Land Act. As such, respondents and their 12. Creeks
predecessors in interest could not have possessed the lots 13. Reservations for public and semi-public purposes
for the required period of 30 years as disposable
agricultural land. Cases:

Republic v. Bautista, Jr., MIAA vs Court of Appeals


G.R. No. 166890, [June 28, 2016]) (GR No. 155650, 7/20/2006)

Notes of Cesnee Tan Page 30


Natural Resources 1st Exam (2017-2018)

Facts: Manila International Airport Authority (MIAA) Roman Catholic Bishop of Kalibo vs Municipality of
operates the Ninoy Aquino International Airport (NAIA) Buruanga
Complex in Parañaque City. The Office of the Government (GR No. 149145, 3/31/2006)
Corporate Counsel (OGCC) issued Opinion No. 061 which FACTS: Petitioner Roman Catholic Bishop of Kalibo was
states that the Local Government Code of 1991 withdrew allegedly the lawful owner and possessor of a parcel of
the exemption from real estate tax granted to MIAA. Thus, residential and commercial land, designated as Lot 138.
MIAA negotiated with City of Parañaque to pay the real The Roman Catholic Church was built in 1984 in the middle
estate tax imposed by the City. MIAA then paid some of the portion of the said lot and has been in existence since then
real estate tax already due. up to the present. That sometime in 1978, the Municipality
of Buruanga constructed its municipal building on the
MIAA received Final Notices of Real Estate Tax Delinquency northeastern portion of the Lot 138 after it obtained the
from the City of Parañaque for the taxable years 1992 to permission of the then parish priest of Buruanga on the
2001 amounting to 624M. The City of Parañaque, through condition that the municipality remove all the improvements
its City Treasurer, also issued notices of levy and warrants it constructed thereon if and when the petitioner needed
of levy on the Airport Lands and Buildings. The Mayor of the said lot.
the City of Parañaque threatened to sell at public auction When the municipal building was razed by fire in 1989,
the Airport Lands and Buildings should MIAA fail to pay the petitioner, through its counsel requested the officials of the
real estate tax delinquency. MIAA thus sought a clarification municipality to refrain from constructing its new building on
of OGCC Opinion No. 061. The OGCC issued Opinion No. the same site because it is the property of the church and it
147 clarifying OGCC Opinion No. 061 and pointed out that needed the lot for its social projects. However, the
Local Government Code requires persons exempt from real construction of the new municipal building on the same site
estate tax to show proof of exemption. The OGCC opined proceeded. Consequently, petitioner filed a complaint and
that Section 21 of the MIAA Charter is the proof that MIAA prayed that it be declared the lawful owner and possessor
is exempt from real estate tax. MIAA then filed with the of Lot 138. Petitioner contends that it does not allegedly
Court of Appeals an original petition for prohibition and lose its possession or ownership over the property if the
injunction, with prayer for preliminary injunction or possession or use by another of the same is by mere
temporary restraining order to restrain the City of tolerance.
Parañaque from imposing real estate tax on, levying In its answer, the municipality alleged that said
against, and auctioning for public sale the Airport Lands lot was surveyed as property of the municipality and that
and Buildings. However, this was dismissed due to because the said municipality alone had possessed the said land
MIAA filed it beyond the 60-day reglementary period. The under the claim of title exclusively for over fifty (50) years,
Court of Appeals also denied MIAA's motion for exclusive of all other rights and adverse to all other
reconsideration and supplemental motion for claimants.
reconsideration. Hence, MIAA filed a petition for review After due trial, the trial court declared
before the SC. petitioner as the lawful owner and possessor of Lot 138-B
and the Municipality of Buruanga as the lawful owner and
Issue: WON the Land and Buildings of MIAA are part of the possessor of Lots 138-A and 138-C, the said lots being
public dominion and thus cannot be the subject of levy and public plaza for public use. On appeal, the CA affirmed the
auction sale. ownership of petitioner over Lot 138-B but declared Lots
138-A and 138-C as property of public dominion.
Held: Yes. The Court held that the land and buildings of
MIAA are part of the public dominion. Since the airport is ISSUE: Whether or not petitioner‘s open, continuous,
devoted for public use, for the domestic and international exclusive and notorious possession and occupation of Lot
travel and transportation. Even if MIAA charge fees, this is 138 since 1894 and for many decades thereafter vests ipso
for support of its operation and for regulation and does not jure or by operation of law upon it a government grant, a
change the character of the land and buildings of MIAA as vested title, to the subject property.
part of the public dominion.
HELD: No. There was no question that petitioner has been
As part of the public dominion the land and buildings of in open, continuous, exclusive and notorious possession
MIAA are outside the commerce of man. To subject them to and occupation of Lot 138-B since 1894 as evidenced by
levy and public auction is contrary to public policy. Unless the church structure built thereon but there was no
the President issues a proclamation withdrawing the airport evidence to show that such possession and occupation
land and buildings from public use, these properties remain extended to Lots 138-A and 138-C beginning the same
to be of public dominion and are inalienable. As long as the period. No single instance of the exercise by the petitioner
land and buildings are for public use the ownership is with of proprietary acts or acts of dominion over these lots was
the Republic of the Philippines. established. Its unsubstantiated claim that the construction
of the municipal building as well as the subsequent
improvements thereon was by its tolerance does not
constitute proof of possession and occupation on the
petitioner‘s part. Absent the important requisite of open,

Notes of Cesnee Tan Page 31


Natural Resources 1st Exam (2017-2018)

continuous, exclusive and notorious possession and Philippine Islands. The lower court ordered and decreed
occupation thereon since 1894, no government grant or that said parcel of land be registered in the name of the
title to Lots 138-A and 138-C had vested upon the said applicant, J. H. Ankron, subject, however, to the right
petitioner ipso jure or by operation of law. of the Government of the Philippine Islands to open a road
Possession is open when it is patent, visible, thereon in the manner and conditions mentioned in said
apparent, notorious and not clandestine. It is continuous decision. On appeal, appellant, contented, among others
when uninterrupted, unbroken and not intermittent or that, that portions of said land cannot be registered in
occasional; exclusive when the adverse possessor can show accordance with the existing Land Registration Law for the
exclusive dominion over the land and an appropriation of it reason that they are manglares.
to his own use and benefit; and notorious when it is so
conspicuous that it is generally known and talked of by the Issue: WON the land in question cannot registered?
public or the people in the neighborhood. Use of land is
adverse when it is open and notorious. Ruling: The mere fact that a tract of land has trees upon it
or has mineral within it is not of itself sufficient to declare
Amunategui vs. Director of Forestry that one is forestry land and the other, mineral land. There
(GR No. L-27873, 11/29/1983) must be some proof of the extent and present or future
There were two petitions for review on certiorari value of the forestry and of the minerals. While, as we have
questioning the decision of the Court of Appeals which just said, many definitions have been given for
declared the disputed property as forest land, not subject "agriculture," "forestry," and "mineral" lands, and that in
to titling in favor of private persons, Borre and Amunategui. each case it is a question of fact, we think it is safe to say
The Director of Forestry, through the Provincial Fiscal of that in order to be forestry or mineral land the proof must
Capiz, also filed an opposition to the application for show that it is more valuable for the forestry or the mineral
registration of title claiming that the land was mangrove which it contains than it is for agricultural purposes. (Sec. 7,
swamp which was still classified as forest land and part of Act No. 1148.) It is not sufficient to show that there exists
the public domain. some trees upon the land or that it bears some mineral.
Another oppositor, Emeterio Bereber filed his opposition Land may be classified as forestry or mineral today, and, by
insofar as a portion of Lot No. 885 containing 117,956 reason of the exhaustion of the timber or mineral, be
square meters was concerned and prayed that title to said classified as agricultural land tomorrow. And vice-versa, by
portion be confirmed and registered in his name. reason of the rapid growth of timber or the discovery of
Issue: WON the lot in question can be subject of valuable minerals, lands classified as agricultural today may
registration and confirmation of title in the name of the be differently classified tomorrow. Each case must be
private person. decided upon the proof in that particular case, having
Held: The opposition of the Director of Forestry was regard for its present or future value for one or the other
strengthened by the appellate court's finding that timber purposes. We believe, however, considering the fact that it
licenses had to be issued to certain licensees and even Jose is a matter of public knowledge that a majority of the lands
Amunategui himself took the trouble to ask for a license to in the Philippine Islands are agricultural lands that the
cut timber within the area. It was only sometime in 1950 courts have a right to presume, in the absence of evidence
that the property was converted into fishpond but only to the contrary, that in each case the lands are agricultural
after a previous warning from the District Forester that the lands until the contrary is shown. Whatever the land
same could not be done because it was classified as "public involved in a particular land registration case is forestry or
forest‖. mineral land must, therefore, be a matter of proof. Its
A forested area classified as forest land of the public superior value for one purpose or the other is a question of
domain does not lose such classification simply because fact to be settled by the proof in each particular case. The
loggers or settlers may have stripped it of its forest cover. fact that the land is a manglar [mangrove swamp] is not
"Forest lands" do not have to be on mountains or in out of sufficient for the courts to decide whether it is agricultural,
the way places. Swampy areas covered by mangrove trees, forestry, or mineral land. It may perchance belong to one or
nipa palms, and other trees growing in brackish or sea the other of said classes of land. The Government, in the
water may also be classified as forest land. The possession first instance, under the provisions of Act No. 1148, may,
of forest lands, no matter how long, cannot ripen into by reservation, decide for itself what portions of public land
private ownership. Therefore, the lot in question never shall be considered forestry land, unless private interests
ceased to be classified as forest land of public domain. have intervened before such reservation is made. In the
latter case, whether the land is agricultural, forestry, or
Ankron vs Government of the Philippines mineral, is a question of proof. Until private interests have
(GR No. 14213, 8/23/1919) intervened, the Government, by virtue of the terms of said
This case involves an application for registration under the Act (No. 1148), may decide for itself what portions of the
Torrens system of a certain parcel of land. The only "public domain" shall be set aside and reserved as forestry
opposition of the said application which was presented by or mineral land
the Director of Lands was that the land in question was the
property of the Government of the United States under the Sta. Rosa Development Corporation vs CA
control and administration of the Government of the (GR No. 112526, 10/12/2001)

Notes of Cesnee Tan Page 32


Natural Resources 1st Exam (2017-2018)

FACTS: The case is a petition regarding Department of


Agrarian Reform Adjudication Board‘s (DARAB) order of We cannot ignore the fact that the disputed parcels of land
compulsory acquisition of petitioner‘s property under the form a vital part of an area that need to be protected for
Comprehensive Agrarian Reform Program (CARP). watershed purposes. The protection of watersheds ensures
an adequate supply of water for future generations and the
Petitioner Sta. Rosa Development Corporation (SRRDC), control of flashfloods that not only damage property but
was the registered owner of two parcel of land situated at cause loss of lives. Protection of watersheds is an
Brgy. Casile, Cabuyao, Laguna. According to them, these intergenerational responsibility that needs to be answered
lands are watersheds which provide clean and potable now.
(drinkable) water to the Canlubang community and that 90
light industries are located in that area. Although evidence of petitioners is strong, the Supreme
Court opines that the area must be maintained for
They were alleging respondents usurped its rights over watershed purposes for ecological and environmental
their property thereby destroying the ecosystem. Since the considerations despite the 88 families who are beneficiaries
said land provides water to the residents, respondents of the CARP. It is important that a larger view of the
sought an easement of a right of a way to and from situation be taken because of the thousands of residents
Barangay Castile, to which, by counterclaim, Sta. Rosa downstream if the watershed will not be protected and
sought ejectment against respondents. maintained for its natural purpose.

Respondents went to the DAR and filed a case for Despite Supreme Court‘s strong opinion of protection of
compulsory acquisition of the Sta. Rosa Property under the watersheds as an intergenerational responsibility, they,
Comprehensive Agrarian Reform Program. however ordered to DARAB to conduct a re-evaluation of
the case since the said land falls under exception.
Compulsory acquisition is the power of the government to
acquire private rights in land without the willing consent of Collado vs Court of Appeals
its owner or occupant in order to benefit the society. (GR No. 107764, 10/4/2002)
FACTS: Petitioner Edna T. Collado filed with the land
The said land was inspected by the Municipal and Agrarian registration court an application for registration of a parcel
Reform Officer, and upon consensus of the authorities of land (―Lot‖), situated in Antipolo Rizal. Attached to the
concerned, they decided that the said land must be placed application was a technical description, stating ―this survey
under compulsory acquisition. is inside IN-12 Mariquina Watershed.‖ The Solicitor General
filed oppositions to the application. Petitioners (Edna
Petitioners filed an objection on the ground that: Collado and her co-applicants) allege that they have
occupied the Lot since time immemorial. Their possession
· The area is not appropriate for agricultural purposes. has been open, public, notorious and in the concept of
· The area was rugged in terrain with slopes 18% and owners. They paid all real estate taxes and submitted
above. (which falls under the exception in compulsory evidence to prove that there have been 9 transfers of rights
acquisition of CARP) among them and their predecessors-in-interest. RTC ruled
· The occupants of the land were illegal settlers or in favor of the petitioners for having presented sufficient
(squatters) who by no means are entitled to the land as evidence to establish registrable title over the property.
beneficiaries. ISSUE: (1) WON petitioners have registrable title over the
Lot. NO.
ISSUE: Whether or not the property in question is covered
by CARP despite the fact that the entire property formed (2) Did petitioners acquire private rights over the parcel of
part of a watershed area prior to the enactment of R.A No. land prior to the issuance of EO 33? NO.
6657
· Whether the petition of land conversion of the parcels HELD: 1) Petitioners concede that the Lot is inside the
of land may be granted? literal description of Marikina Watershed Reservation
(MWR). Their main claim over the Lot is that ―all
HELD: Watershed is one of those enumerated by CARP to Presidential proclamations like the proclamation setting
be exempt from its coverage. aside the MWR are subject to private rights.‖ EO 33 (which
established the MWR) has a saving clause that the
· Art. 67 of PD 1067 provides that Any watershed or reservations are ―subject to existing private rights, if any
any area of land adjacent to any surface water or overlying there be.‖
any ground water may be declared by the Department of
Natural resources as a protected area. Under the Regalian Doctrine, all lands not otherwise
· Watersheds may be defined as an area appearing to be clearly within private ownership are
drained by a river and its tributaries and enclosed by a presumed to belong to the State. The Spaniards first
boundary or divide which separates it from adjacent introduced the doctrine to the Philippines through the Laws
watersheds. of the Indies and the Royal Cedulas, specifically, Law 14,

Notes of Cesnee Tan Page 33


Natural Resources 1st Exam (2017-2018)

Title 12, Book 4 of the Novisima Recopilacion de Leyes de policy. Indeed, all lands of the public domain as well as all
las Indias which laid the foundation that "all lands that natural resources enumerated in the Philippine Constitution
were not acquired from the Government, either by belong to the State.
purchase or by grant, belong to the public domain." Upon
the Spanish conquest of the Philippines, ownership of all Watershed Reservation is a Natural Resource: The term
"lands, territories and possessions" in the Philippines "natural resource" includes "not only timber, gas, oil coal,
passed to the Spanish Crown. minerals, lakes, and submerged lands, but also, features
which supply a human need and contribute to the health,
The Laws of the Indies were followed by the Ley welfare, and benefit of a community, and are essential to
Hipotecaria or the Mortgage Law of 1893. The Spanish the well-being thereof and proper enjoyment of property
Mortgage Law provided for the systematic registration of devoted to park and recreational purposes."
titles and deeds as well as possessory claims. The Royal
Decree of 1894 or the "Maura Law" partly amended the (2) An applicant must overcome the presumption that the
Mortgage Law as well as the Law of the Indies. The Maura land he is applying for is part of the public domain and that
Law was the last Spanish land law promulgated in the he has an interest to warrant registration in his name
Philippines. It required the "adjustment" or registration of arising from an imperfect title (may have been derived from
all agricultural lands, otherwise the lands would revert to old Spanish grants or titles). In the case at bar, petitioners
the state. were unable to acquire a valid and enforceable right or title
because of the failure to complete the required period of
Four years later, Spain ceded to the government of the possession (at least 30 years).
United States all rights, interests and claims over the
national territory of the Philippine Islands through the Assuming that the Lot was alienable and disposable land
Treaty of Paris of December 10, 1898. In 1903, the United prior to the issuance of EO 33 in 1904, EO 33 reserved the
States colonial government, through the Philippine Lot as a watershed. Since then, the Lot became non-
Commission, passed Act No. 926, the first Public Land Act, disposable and inalienable public land. At the time
which was described as follows: petitioners filed their application on April 25, 1985, the Lot
has been reserved as a watershed under EO 33 for 81 years
"Act No. 926, the first Public Land Act, was passed in prior to the filing of petitioners‘ application.
pursuance of the provisions of the Philippine Bill of 1902.
The law governed the disposition of lands of the public Director of Forestry vs. Villareal
domain. It prescribed rules and regulations for the (GR No. L-32266, 2/27/1989)
homesteading, selling and leasing of portions of the public FACTS: The said land consists of 178,113 square meters of
domain of the Philippine Islands, and prescribed the terms mangrove swamps located in the municipality of Sapian,
and conditions to enable persons to perfect their titles to Capiz. Ruperto Villareal applied for its registration on
public lands in the Islands. It also provided for the January 25, 1949, alleging that he and his predecessors-in-
"issuance of patents to certain native settlers upon public interest had been in possession of the land for more than
lands," for the establishment of town sites and sale of lots forty years. He was opposed by several persons, including
therein, for the completion of imperfect titles, and for the the petitioner on behalf of the Republic of the Philippines.
cancellation or confirmation of Spanish concessions and After trial, the application was approved by the Court of
grants in the Islands." In short, the Public Land Act First Instance of Capiz. The decision was affirmed by the
operated on the assumption that title to public lands in the Court of Appeals. The Director of Forestry then came to this
Name: I. Concept of Jura Regalia Natural Resources First Court in a petition for review on certiorari claiming that the
Set_ Philippine Islands remained in the government; and land in dispute was forestal in nature and not subject to
that the government‘s title to public land sprung from the private appropriation. He asks that the registration be
Treaty of Paris and other subsequent treaties between reversed. It is undisputed by the parties that the land in
Spain and the United States. The term "public land" dispute is a mangrove land HOWEVER the legal nature of
referred to all lands of the public domain whose title still mangrove swamps or manglares are still in contention.
remained in the government and are thrown open to Director of Forestry claims that it is forestall and is not
private appropriation and settlement, and excluded the disposable. On the other hand, Private respondents insists
patrimonial property of the government and the friar that it is alienable as agricultural land.
lands."
ISSUES: Are mangrove swamps classified as public forest
Thus, it is plain error for petitioners to argue that under the lands?
Philippine Bill of 1902 and Public Land Act No. 926, mere
possession by private individuals of lands creates the legal RULING: YES. Part of our public forest lands, they are not
presumption that the lands are alienable and disposable. alienable under the Constitution or are they considered
public agricultural lands; they may be acquired under
Both the 1935 and 1973 Constitutions prohibited the private ownership.
alienation of all natural resources except agricultural lands
of the public domain. The 1987 Constitution readopted this Mangrove swamps or manglares should be understood as

Notes of Cesnee Tan Page 34


Natural Resources 1st Exam (2017-2018)

comprised within the public forests of the Philippines as Fredia mineral claim. The Fredia mineral claim together
defined in the aforecited Section 1820 of the Administrative with other mineral claims owned by Atok has been declared
Code of 1917. The legislature having so determined, we under Tax Declaration No. 9535 and that in view of
have no authority to ignore or modify its decision, and in Presidential Decree No. 1214 an application for lease was
effect veto it, in the exercise of our own discretion. The filed by Atok covering the Fredia mineral claim.
statutory definition remains unchanged to date and, no less
noteworthy, is accepted and invoked by the executive On the other hand, private respondent Liwan Consi has a
department. More importantly, the said provision has not lot below the land of a certain Mr. Acay at Tuding Slide,
been challenged as arbitrary or unrealistic or Itogon, Benguet. He constructed a house thereon sometime
unconstitutional assuming the requisite conditions, to justify in 1964. The lot is covered by Tax Declaration No. 9462.
our judicial intervention and scrutiny. The law is thus When he first constructed his house below the lot of Mr.
presumed valid and so must be respected. We repeat our Acay he was told that it was not necessary for him to obtain
statement in the Amunategui case that the classification of a building permit as it was only a nipa hut. And no one
mangrove swamps as forest lands is descriptive of its legal prohibited him from entering the land so he was
nature or status and does not have to be descriptive of constructing a house thereon. It was only in January 1984
what the land actually looks like. That determination having when private respondent Consi repaired the said house that
been made and no cogent argument having been raised to people came to take pictures and told him that the lot
annul it, we have no duty as judges but to apply it. belongs to Atok. Private respondent Consi has been paying
taxes on said land which his father before him had occupied
It follows from all this that the land under contention being On January 1984, the security guards of Atok informed
admittedly a part of the mangrove swamps of Sapian, and Feliciano Reyes, Security Officer of Atok, that a construction
for which a minor forest license had in fact been issued by was being undertaken at the area of the Fredia mineral
the Bureau of Forestry from 1920 to 1950, it must be claim by private respondent Liwan Consi. Feliciano Reyes
considered forest land. It could therefore not be the subject instructed the cashier to go and take pictures of the
of the adverse possession and consequent ownership construction. Feliciano Reyes himself and other security
claimed by the private respondent in support of his guards went to the place of the construction to verify and
application for registration. To be so, it had first to be then to the police to report the matter.
released as forest land and reclassified as agricultural land
pursuant to the certification the Director of Forestry may On March 1, 1984, Atok filed a complaint for forcible entry
issue under Section 1827 of the Revised Administrative and detainer against Liwan Consi, which was dismissed
Code. after due hearing by the MTC of Itogon in favor of Liwan
Consi. Petitioner ATOK appealed to the RTC of Baguio,
The Respondent even showed, a survey of the land and its which reversed the decision of the MTC, ordering
tax declaration to support its claim, however the court held defendant Liwan Consi to vacate the premises of the Fredia
that the same is insufficient especially now that the land is Mineral claim, restoring possession thereof to the plaintiff
a forest land. Atok Big Wedge Mining Company. Defendant Liwan Cosi
was further ordered to remove and demolish the house he
Atok-Big Wedge Mining Corporation vs Court of constructed in the premises of the land of Fredia Mineral
Appeals, claim.
GR No. 88883, 1/18/1991)
FACTS: Fredia Mineral claim of about nine (9) hectares In a petition for review filed by Liwan Consi with the CA,
situated in Tuding, Itogon, Benguet, was located sometime the CA rendered its decision dismissing the subject forcible
between December 25, 1930 and December 31, 1930, a entry action, and further rule in part that: Liwan Consi had
period of six (6) days, by A.I. Reynolds in accordance with a possessory right over the property which may mature into
the provisions of the Act of Congress of July 1, 1902, better ownership on the basis of long-term possession under the
known as the Philippine Bill of 1902, in a so-called Public Land Law. Thus, it held that both Consi and ATOK
Declaration of Location. are of equal footing with regards to the subject lot, holding
possessory titles to the land. The petitioner through its long
The said Declaration of Location of mineral claim was duly term occupancy while respondent mining firm being the
recorded in the Office of the Mining Recorder sometime on claim locator and applicant for lease on the mineral claim.
January 2, 1931. Fredia mineral claim, together with other
mineral claims, was sold by A.I. Reynolds to Big Wedge ATOK filed a motion for reconsideration, which was denied
Mining Company, the earlier corporate name of Atok Big by the CA. Hence, this petition.
Wedge Mining Company, Inc. (Atok for short; herein
petitioner) in a Deed of Sale executed on November 2, ISSUE: Whether or not an individual's long term occupation
1931. Since then petitioner Atok has been in continuous of land of the public domain vests him with such rights over
and exclusive ownership and possession of said claim up to the same as to defeat the rights of the owner of that claim.
the present.
HELD: It is of no importance whether Benguet and Atok
Atok has paid the realty taxes and occupation fees for the had secured a patent for as held in the Gold Creek Mining

Notes of Cesnee Tan Page 35


Natural Resources 1st Exam (2017-2018)

Corporation case, for all physical purposes of ownership, sometime in 1964 when he constructed a house thereon.
the owner is not required to secure a patent as long as he Clearly, ATOK has superior possessory rights than private
complies with the provisions of the mining laws; his respondent, Liwan Consi, the former being "the one longer
possessory right, for all practical purposes of ownership, is in possession."
as good as though secured by patent (Republic v. Court of
Appeals, 160 SCRA 228 [1988]). It is therefore clear that from the legal viewpoint it was
really petitioner who was in actual physical possession of
In the case at bar, the evidence on record pointed that the the property. Having been deprived of this possession by
petitioner Atok has faithfully complied with all the the private respondent, petitioner has every right to sue for
requirements of the law regarding the maintenance of the ejectment.
said Fredia Mineral Claim.
The perfection of the mining claim converted the property With this ruling enunciated by the Court, it can further be
to mineral land and under the laws then in force removed it declared and held that petitioner Atok has the exclusive
from the public domain. By such act, the locators acquired right to the property in question.
exclusive rights over the land, against even the
government, without need of any further act such as the
purchase of the land or obtaining of a patent over it. As the Palomo vs Court of Appeals
land had become the private property of the locators, they [G.R. No. 95608. January 21, 1997]
had the right to transfer the same, as they did, to Benguet FACTS: In 1913, some 440,530 square meters of land in
and Atok. Albay were reserved for provincial park purposes by virtue
As in the instant petition, the record shows that the lot in of EO No. 40. Of said area, 15 parcels of land were
question was acquired through a Deed of Sale executed registered in the name of Diego Palomo by the Court of
between Atok and Fredia Mineral Claim. First Instance.

It is, therefore, evident that Benguet and Atok have In 1937, Diego Palomo donated these lands to his heirs
exclusive rights to the property in question by virtue of their Ignacio and Carmen Palomo. Claiming that the aforesaid
respective mining claims which they validly acquired before original certificates of title were lost during the Japanese
the Constitution of 1935 prohibited the alienation of all occupation, Ignacio Palomo filed a petition for
lands of the public domain except agricultural lands, subject reconstitution with the Court of First Instance of Albay on
to vested rights existing at the time of its adoption. The May 30, 1950. The Register of Deeds of Albay issued
land was not and could not have been transferred to the Transfer Certificates of Title Nos. 3911, 3912, 3913 and
private respondents by virtue of acquisitive prescription, nor 3914 sometime in October 1953.
could its use be shared simultaneously by them and the
mining companies for agricultural and mineral purposes On July 10, 1954, President Magsaysay issued Proclamation
(Ibid). No. 47 converting the area covered by EO 40 into the Tiwi
Hot Spring National Park. The Palomos contended that they
On the matter of possession, private respondent contends have been in possession of the subject lands and have
that his predecessor-in-interest has been in possession of introduced improvements thereon.
said lot even before the war and has in fact cultivated the
same. Since the subject lot is mineral land, private ISSUE: Were the Original Certificate of Titles issued to the
respondent's possession of the subject lot no matter how petitioners valid? –NO
long did not confer upon him possessory rights over the
same. HELD: Before the Treaty of Paris in 1899, the lands,
whether agricultural, mineral, or forest were under the
Furthermore, Article 538 of the New Civil Code provides: exclusive patrimony and dominion of the Spanish crown.
Private ownership of land could only be acquired through
Art. 538. Possession as a fact cannot be recognized at royal concessions which were documented in various forms,
the same time in two different personalities except in the such as (1) Titulo Real or Royal Grant," (2) Concession
cases of co-possession. Should a question arise regarding Especial or Special Grant, (3) Titulo de Compra or Title by
the fact of possession, the present possessor shall be Purchase and (4) Informacion Posesoria or Possessory
preferred; if there are two possessors, the one longer in Information title obtained under the Spanish Mortgage Law
possession; if the dates of the possession are the same, the or under the Royal Decree of January 26, 1889.
one who presents a title; and if all these conditions are
equal, the thing shall be placed in judicial deposit pending The decision of the CFI relied upon by petitioners were not
determination of its possession or ownership through signed by the judge but were merely certified copies of
proper proceedings. notification to Diego Palomo bearing the signature of the
Clerk of Court.
Since 1931 up to the present, petitioner ATOK has been in
continuous and exclusive possession of the Fredia mineral More importantly, the lands in question were not classified
claim while private respondent's possession started only as alienable lands. Since the lands were made part of a

Notes of Cesnee Tan Page 36


Natural Resources 1st Exam (2017-2018)

reservation for provincial park purposes, they form part of dismissed as forged his signature appearing in the same
the forest zone. Thus, they cannot be the valid subject of document and denied ever signing the same, let alone in
alienation. front of a notary public holding office outside of the LMB
premises. Pressing the point, Palad stated that he could not
Republic vs Southside Homeowners have had signed the conveying deed involving as it did a
(GR No. 156951, 9/22/2006) reservation area which, apart from its being outside of the
FACTS: The subject matter of these proceedings for LMB‘s jurisdiction, is inalienable in the first place.
declaration of nullity of title are parcels of land with a total For its part, then defendant SHAI presented an opposing
area of 39.99 hectares, more or less, known as the JUSMAG expert witness in the person of Police Inspector Redencion
housing area in Fort Bonifacio where, military officers, both Caimbon who testified that Palad‘s signature in Exhibit ―A‖
in the active and retired services, and their respective is genuine. Mrs. Virginia Santos, then SHAI president,
families, have been occupying housing units and facilities likewise testified, saying that applications to purchase were
originally constructed by the AFP. signed and then filed with the LMB by one Engr. Eugenia
Balis, followed by the payment in full of the contract price.
Private respondent SHAI is a non-stock corporation
organized mostly by wives of AFP military officers. Records Eventually, in a decision dated October 7, 1997, the trial
show that SHAI was able to secure from the Registry of court rendered judgment dismissing the Republic‘s
Deeds of the Province of Rizal a title – Transfer Certificate complaint as it considered the parcels covered by the deed
of Title in its name to the bulk of, if not the entire, JUSMAG in question as no longer part of the FBMR. Therefrom, the
area. Republic went on appeal to the CA which affirmed in toto
The Rizal Registry issued TCT No. 15084 on October 30, that of the trial court.
1991on the basis of a notarized Deed of Sale purportedly
executed on the same date by then Director Abelardo G. Hence, this petition of the Republic.
Palad, Jr. of the Lands Management Bureau (LMB) in favor
of SHAI.The total purchase price as written in the ISSUE: Was the JUSMAG area, during the period material,
conveying deed was P11,997,660.00 or P30.00 per square alienable or inalienable, as the case may be, and, therefore,
meter can or cannot be subject of a lawful private conveyance?
It appears that in the process of the investigation
conducted by the Department of Justice on reported land RULING: Petitioner Republic, correctly asserts the
scams at the FBMR, a copy of the aforesaid October 30, inalienable character of the JUSMAG area, the same having
1991deed of sale surfaced and eventually referred to the not effectively been separated from the military reservation
National Bureau of Investigation (NBI) for examination. The and declared as alienable and disposable.
results of the examination undertaken by NBI Document
Examiner Eliodoro Constantino reveals that the puported The President, upon the recommendation of the Secretary
signatures in the document are forgeries. of Environment and Natural Resources, may designate by
proclamation any tract or tracts of land of the public
On October 16, 1993, then President Fidel V.Ramos issued domain as reservations for the use of the Republic or any of
Memorandum Order No. 173 directing the Office of the its branches, or for quasi-public uses or purposes. Such
Solicitor General (OSG) to institute action towards the tract or tracts of land thus reserved shall be non-alienable
cancellation of TCT No. 15084 and the title acquired by the and shall not be subject to sale or other disposition until
Navy Officer‘s Village Association (NOVA) over a bigger again declared alienable. Consistent with the foregoing
parcel within the reservation. A month later, the OSG, in postulates, jurisprudence teaches that a military
behalf of the petitioner Republic, filed with the RTC of Pasig reservation, like the FBMR, or a part thereof is not open to
City the corresponding nullification and cancellation of title private appropriation or disposition and, therefore, not
suit against the private respondent SHAI, purported registrable, unless it is in the meantime reclassified and
signature thereon of Palad is a forgery; b) there are no declared as disposable and alienable public land. And until
records with the LMB of (i) the application to purchase and a given parcel of land is released from its classification as
(ii) the alleged payment of the purchase price; and c) the part of the military reservation zone and reclassified by law
property in question is inalienable, being part of a military or by presidential proclamation as disposable and alienable,
reservation established under Proclamation No. 423. its status as part of a military reservation remains,even if
On pre-trial the Republic, as plaintiff therein, marked (and incidentally it is devoted for a purpose other than as a
later offered in evidence)the Deed of Sale dated October military camp or for defense. The same is true in this case.
30, 1991 as its Exhibit "A,"and TCT No. 15084 as Exhibit There is no doubt that the JUSMAG area subject of the
"B."Respondent, then defendant SHAI adopted Exhibits "A" questioned October 30, 1991sale formed part of the FBMR
and ―B‖ as its Exhibits "1" and ―2,‖ respectively. as originally established under Proclamation No. 423. And
while private respondent SHAI would categorically say that
During the trial, the Republic presented as expert witness the petitioner Republic had not presented evidence that
NBI Document Examiner Eliodoro Constantino who testified ―subject land is within military reservation,‖and even dared
on NBI QDR No. 815-1093 and asserted that the signature to state that the JUSMAG area is the private property of the
of Palad in Exhibit ―A‖ is a forgery. For his part, Palad government and therefore removed from the concept of

Notes of Cesnee Tan Page 37


Natural Resources 1st Exam (2017-2018)

public domain per se its own evidence themselves belie its


posture as their evidence both the TCT and the Deed of 3. There is absolutely no record of the requisite public land
Sale technically described the property as situated in application to purchase required under Section 89 of the
Jusmag area located at Fort Bonifacio which is now Public Land Act. There is also no record of the deed of sale
renamed Fort Mckinley a declared a military reservation. and of documents usually accompanying an application to
purchase, inclusive of the investigation report and the
The Republic has, since the filing of its underlying property valuation. The Certification under the seal of the
complaint, invoked Proclamation No. 423. In the process, it LMB bearing date November 24, 1994 and issued/signed by
has invariably invited attention to the proclamation‘s Alberto Recalde, OIC, Records Management Division of the
specific area coverage to prove the nullity of TCT No. LMB pursuant to a subpoena issued by the trial court attest
15084, inasmuch as the title embraced a reserved area to this fact of absence of records. Atty. Alice B. Dayrit, then
considered inalienable, and hence, beyond the commerce Chief, Land Utilization and Disposition Division, LMB,
of man. testified having personally looked at the bureau record
book, but found no entry pertaining to SHAI.
The October 30, 1991 Deed of Sale purportedly executed
by Palad, assuming its authenticity, could not plausibly be 4. In its Answer as defendant a quo, respondent SHAI
the requisite classifying medium converting the JUSMAG states that the ―deed of sale specifically meritorious Official
area into a disposable parcel. And private respondent Receipt No. 6030203 as evidence of full payment of the
SHAI‘s unyielding stance that would have the Republic in agreed purchase price‖ An official receipt (O.R.) is
estoppel to question the transfer to it by the LMB Director doubtless the best evidence to prove payment. While it kept
of the JUSMAG area is unavailing. It should have realized referring to O.R. No. 6030203 as its evidence of the
that the Republic is not usually estopped by the mistake or required payment, it failed to present and offer the receipt
error on the part of its officials or agents. in evidence. We can thus validly presume that no such OR
exists or, if it does, that its presentation would be adverse
Since the parcels of land in question allegedly sold to the to SHAI.
private respondent are, or at least at the time of the A contract of sale is void where the price, which appears in
supposed transaction were, still part of the FBMR, the the document as paid has, in fact, never been paid.
purported sale is necessarily void ab initio.
5. The purchase price was, according to the witnesses for
Moreover, Article XII, Section 3[of the 1987 Constitution SHAI, paid in full in cash to the cashier of the LMB the
forbids private corporations from acquiring any kind of corresponding amount apparently coming in a mix of P500
alienable land of the public domain, except through lease and P100 denominations. Albeit plausible, SHAI‘s witnesses‘
for a limited period. account taxes credulity to the limit.

The interplay of compelling circumstances and inferences TCT No. 15084 of the Registry of Deeds of Rizal issued on
deducible from the case, also cast doubt on the authenticity the basis of such Deed are declared void and cancelled
of such deed, if not support a conclusion that the deed is
spurious. Republic vs Alagad
(G.R. No. L-66807 January 26, 1989)
1. Palad categorically declared that his said signature on Facts: On 11 October 1951, Melitona, Carmen (with spouse
the deed is a forgery. The NBI signature expert Espiridion Kolimlim), Justo, Carlos, Librada (with spouse
corroborated Palad‘s allegation on forgery.Respondent Emerson Abano), Demetrio, and Antonio Alagad filed an
SHAI‘s expert witness from the PNP, however, disputes the application for registration of their title over a parcel of land
NBI‘s findings. In net effect, both experts from the NBI and situated at Linga, Pila, Laguna, with an area of 8.1263
the PNP cancel each other out. hectares (survey plan Psu116971), which was amended
after the land was divided into two parcels, namely, Lot 1
2.Palad signed the supposed deed of sale in Manila, with an area of 5.2476 hectares and Lot 2 with an area of
possibly at the LMB office at Plaza Cervantes, Binondo. 2.8421 hectares (survey plan Psu-226971, amendment 2).
Even if he acted in an official capacity, Palad nonetheless The Republic opposed the application on the stereo-typed
proceeded on the same day to Pasig City to appear before ground that applicants and their predecessors have not
the notarizing officer. The deed was then brought to the been in possession of the land openly, continuously, publicly
Rizal Registry and there stamped ―Received‖ by the entry and adversely under a bona fide claim of ownership since
clerk. That same afternoon, or at 3:14 p.m. of October 30, 26 July 1894 and the land has not ceased to be a part of
1991to be precise, TCT No. 15084 was issued. In other the public domain. It appears that barrio folk also opposed
words, the whole conveyance and registration process was the application. On 16 January 1956, by virtue of a final
done in less than a day. The very unusual dispatch is quite judgment in said case, supplemented by orders issued on
surprising. Stranger still is why a bureau head, while in the 21 March 1956 and 13 August 1956, the Alagads were
exercise of his functions as the bureau‘s authorized declared owners of Lot 1 and the remaining portion, or Lot
contracting officer, has to repair to another city just to have 2, was declared public land. Decree N-51479 was entered
a deed notarized. and OCT 0- 401, dated 18 October 1956, was issued in the

Notes of Cesnee Tan Page 38


Natural Resources 1st Exam (2017-2018)

names of the Alagads. In August 1966, the Alagads filed negligent act or omission.
before the Municipal Court of Pila, Laguna (Civil Case 52) 3. Res judicata is not an impediment to reversion of
an action to evict the barrio folk occupying portions of Lot property; Republic v. CA, requisites for a prior judgment to
1. On 8 August 1968, judgment was rendered in the become a bar Res judicata is not an impediment to
eviction case ordering the barrio folk therein to return reversion of property. In Republic v. Court of Appeals, the
possession of the premises to the Alagads. The barrio folk Court stated that a certificate of title may be ordered
did not appeal. cancelled (Republic v. Animus, et al.), and the cancellation
The Republic filed a petition for ―annulment of title and may be pursued through an ordinary action therefor. This
reversion, insofar as the 1.42 hectare northwestern portion action cannot be barred by the prior judgment of the land
on end of Lot 1 is concerned, contending that such is registration court, since the said court had no jurisdiction
foreshore land, and that the Alagads could not have had an over the subject matter. And if there was no such
imperfect title to it as it was the barrio folk who filled up jurisdiction, then the principle of res judicata does not
the land to elevate the land to its present condition. The apply. For it is a well-settled rule that for a prior judgment
Court, on 6 October 1970, issued a writ of preliminary to constitute a bar to a subsequent case, the following
injunction enjoining the Provincial Sheriff of Laguna or his requisites must concur; (1) it must be a final judgment; (2)
deputies from enforcing the writ of execution issued in Civil it must have been rendered by a court having jurisdiction
Case 52, and the Alagads from selling, mortgaging, over the subject matter and over the parties; (3) it must be
disposing or otherwise entering into any transaction a judgment on the merits; and (4) there must be, between
affecting the area. The case was set for pre-trial on 6 July the first and second actions, identity of parties, identity of
1971, to which the attorney representing the Republic did subject matter and identity of cause of action (Municipality
not appear. On 16 July 1971, the court dismissed the of Daet vs. C4 93 SCRA 503; Mendoza vs. Arrieta, et al., 91
complaint. The Republic filed a motion for reconsideration, SCRA 113)
was set for hearing, and finally denied by the court. Appeal 4. Property of public dominion: Property for public use or
was made to the Court of Appeals, which sustained the trial public service ―Property‖, according to the Civil Code, ―is
court for failure to show in the record on appeal that the either of public dominion or of private ownership.‖ Property
appeal was perfected on time. Hence, the appeal. is of public dominion if it is (1) intended for public use, such
The Supreme Court reversed the decision of the lower as roads, canals, rivers, torrents, ports and bridges
courts, and reinstated the Republic‘s complaint and constructed by the State, banks, shores, roadsteads and
thus remanded the case to the trial court for further others of similar character;or if it (2) belongs to the State,
proceedings. without being for public use, and are intended for some
public service or for the development of the national
1. State cannot be bound by or estopped from the mistakes wealth.
or negligent acts of its officials or agents 5. Patrimonial property and property of public dominion
The State cannot be bound by, or estopped from, the ―All other property of the State which is not of the character
mistakes or negligent acts of its official or agents, much mentioned in article [420], is patrimonial property,‖
more, non-suited as a result thereof. This is so because the meaning to say, property ―open to disposition‖ by the
state as a persona in law is the judicial entity, which is the Government, or otherwise, property pertaining to the
source of any asserted right to ownership in land under the national domain, or public lands. Property of the public
basic doctrine embodied in the 1935 Constitution as well as dominion, on the other hand, refers to things held by the
the present charter. It is charged moreover with the State by regalian right. They are things res publicae in
conservation of such patrimony. There is need therefore of nature and hence, incapable of private appropriation. Thus,
the most rigorous scrutiny before private claims to portions under the present Constitution, ―[w]ith the exception of
thereof are judicially accorded recognition. Such primordial agricultural lands, all other natural resources shall not be
consideration, not the apparent carelessness, much less the alienated.‖
acquiescence of public officials, is the controlling norm. 6. Public Dominion, as to waters Article 502 provides that
2. Ramos v. Central Bank, and Nilo v. Romero not applicable ―(1) Rivers and their natural beds; (2) Continuous or
to the present case intermittent waters of springs and brooks running in their
The cases of Ramos v. Central Bank of the Philippines and natural beds and the beds themselves; (3) Waters rising
Nilo v. Romero, are not applicable. In Ramos, the Court continuously or intermittently on lands of public dominion;
applied estoppel upon finding of bad faith on the part of (4) Lakes and lagoons formed by Nature on public lands,
the State (the Central Bank) in deliberately reneging on its and their beds; (5) Rain waters running through ravines or
promises. In Nilo, the Court denied efforts to impugn the sand beds, which are also of public dominion; (6)
jurisdiction of the court on the ground that the defendant Subterranean waters on public lands; (7) Waters found
had been ―erroneously‖ represented in the complaint by the within the zone of operation of public works, even if
City Attorney when it should have been the City Mayor, on a constructed by a contractor; (8) Waters rising continuously
holding that the City Attorney, in any event, could have ably or intermittently on lands belonging to private persons, to
defended the City (Davao City). In both cases, it is seen the State, to a province, or to a city or municipality from
that the acts that gave rise to estoppel were voluntary and the moment they leave such lands; and (9) The waste
intentional in character, in which cases, it could not be said waters of fountains, sewers and public establishments‖ are
that the Government had been prejudiced by some of public dominion. It is also ordained in Article 44 of the

Notes of Cesnee Tan Page 39


Natural Resources 1st Exam (2017-2018)

Spanish Law of Waters of 3 August 1866 that ―natural to certain lots registered in its name. It eventually withdrew
ponds and lakes existing upon public lands and fed by the application and filed a petition for registration over the
public waters, belong to the public domain. Lakes, ponds, same foreshore land with the then Court of First Instance of
and pools existing upon the lands of private individuals, or Iloilo. The case was, however, archived as WESVICO‗s
the State or provinces, belong to the respective owners of representative could no longer be contacted, and later on,
such lands, and those situated upon lands of communal use WESVICO has ceased operations.
belong to their respective ‗pueblos.‘‖
7. Laguna de Bay is a lake (Colegio de San Jose case); F.F. Cruz & Co. (F.F. Cruz) filed with the Bureau of Lands,
Highest Ordinary Depth Laguna de Bay has long been Iloilo City a foreshore lease application over a foreshore
recognized as a lake. Laguna de Bay is a body of water land, a portion of which is adjacent to the lot previously
formed in depressions of the earth; it contains fresh water occupied by WESVICO. Sian Enterprises Inc. (SIAIN)
coming from rivers and brooks or springs, and is connected purchased the properties previously owned by WESVICO
with Manila Bay by the Pasig River. Inasmuch as Laguna de from the Development Bank of the Philippines. It
Bay is a lake, the Court must resort to the legal provisions subsequently filed a foreshore lease application over the
governing the ownership and use of lakes and their beds foreshore land adjacent to the properties it bought from
and shores, in order to determine the character and DBP.
ownership of the parcels of land in question. The recourse
to legal provisions is necessary, for under Article 74 of the Upon learning that 130 linear meters of the foreshore land
Law of Waters, ―the natural bed or basin of lakes is the subject of F.F. Cruz‘s foreshore lease application overlapped
ground covered by their waters when at their highest that covered by its foreshore lease application, SIAIN filed a
ordinary depth‖ and in which case, it forms part of the protest 8 alleging that it being the owner of the property
national dominion. When Laguna de Bay‘s waters are at adjoining the overlapping area, it should be given
their highest ordinary depth has been defined as the preference in its lease.
highest depth of the waters of Laguna de Bay during the
dry season, such depth being the ―regular, common, F.F. Cruz, argued that SIAIN must not be given preferential
natural, which occurs always or most of the time during the right since the area in dispute is classified as ―reclaimed‖
year. Otherwise, where the rise in water level is due to the and that the ownership was not by means of accretion. This
―extraordinary‖ action of nature, rainfall for instance, the argument has been sustained by the Land Management
portions inundated thereby are not considered part of the Bureau.
bed or basin of the body of water in question. It cannot
therefore be said to be foreshore land but land outside of Upon appeal to the DENR Secretary, SIAIN was upheld,
the public dominion, and land capable of registration as declaring that there was no basis to declare the area as
private property. ―reclaimed‖. F.F. Cruz however appealed to the Office of
8. Foreshore land the President which overturned the decision of the DENR
A foreshore land has been defined as ―that part of (the Secretary and found that the area is reclaimed. On appeal,
land) which is between high and low water and left dry by the Court of Appeals affirmed the decision. Hence, the
the flux and reflux of the tides,‖ or ―The strip of land that present petition. SIAIN contends that the evidence
lies between the high and low water marks and that is overwhelmingly proves that the disputed area is foreshore
alternatively wet and dry according to the flow of the tide.‖ land and not reclaimed land which thus entitles it
If the submergence, however, of the land is due to preferential rights over the
precipitation, it does not become foreshore, despite its
proximity to the waters. ISSUES: Whether the disputed land is a ―foreshore‖ or
9. Court not a trier of facts; not enough evidence to arrive ―reclaimed‖ area
a conclusive disposition; Remand
The case has to be decided alongside the above principles HELD: That the foreshore area had been reclaimed does
and regretfully, the Court cannot make a ruling because it is not remove it from its classification of foreshore area
not a trier of facts, and it is in possession of no evidence to subject to the preferential right to lease of the littoral
assist it in arriving at a conclusive disposition. The Court owner.
thus remanded the case to the court a quo to determine
whether or not the property subject of controversy is It bears noting that it was not the reclamation that brought
foreshore. the disputed foreshore area into existence. Such foreshore
area existed even before F.F. Cruz undertook its
SIAN Enterprise vs. FF Cruz reclamation. It was ―formed by accretions or alluvial
(GR no. 146616, 8/31/2006) deposits due to the action of the sea.‖ Following Santulan,
That the foreshore area had been reclaimed does not the littoral owner has preferential right to lease the same.
remove it from its classification of foreshore area subject to
the preferential right to lease of the littoral owner. Contrary to the ruling of the Office of the President, as
affirmed by the appellate court, littoral owner WESVICO
Western Visayas Industrial Corporation (WESVICO) filed a cannot be considered to have waived or abandoned its
foreshore lease application over the foreshore land adjacent preferential right to lease the disputed area when it

Notes of Cesnee Tan Page 40


Natural Resources 1st Exam (2017-2018)

subsequently filed an application for registration thereover. This is by way of Presidential Proclamation or a Legislative
For being a part of the public domain, ownership of the Act.
area could not be acquired by WESVICO. Its preferential
right remained, however. Its move to have the contested As a matter of ordinary land registration practice, a special
land titled in its name, albeit a faux pas, in fact more than patent is a patent to grant, cede and convey full ownership
proves its interest to utilize it. of alienable and disposable lands formerly covered by a
reservation or lands of the public domain and is issued
As correctly argued by SIAIN, were WESVICO‗s petition for upon the promulgation of a special law or act of Congress
registration which, as stated earlier, was archived by the or b the Secretary of Environment and Natural resources as
trial court, pursued but eventually denied, WESVICO would authorized by an Executive order of the president.
not have been barred from filing anew a foreshore lease
application. Parenthetically, the petition for registration of Cadastral Registration Proceedings
WESVICO was archived not on account of lack of interest This is a way of bringing public lands under the Torrens
but because it ceased operations due to financial reasons. system. Usually, this is initiated by the government to
facilitate the registration of the properties. Individual
F. Administrative Legalization (Free Patent) registrations initiated by an individual usually take a long
time.
Now, we go to the Administrative. You file it with the DENR.
A new law on this matter is the Residential Free Patent Act This is initiated by the Solicitor General. It is an in rem
issued on lands that are zoned as residential. If you proceeding because of the publication. The publication is a
remember, you can register it (residential/commercial and notice to the whole world, and if you fail to put your answer
residential/industrial). So file it in the CENRO of the place in a given period, you are barred from filing the same.
where the property is located. So if you remember the
qualifications are the following: A. Section 53 of CA 141
1. Any Filipino citizen who is an actual occupant of a
residential land (natural born, naturalized and dual Section53. It shall be lawful for the Director of Lands,
citizens) whenever in the opinion of the President the public
2. Of legal age (minors can file a case if they are interests shall require it, to cause to be filed in the proper
represented by their legal guardian); Court of First Instance, through the Solicitor-General or the
3. That in highly urbanized cities, the land should not officer acting in his stead, a petition against the holder,
exceed two hundred (200) square meters; in other claimant, possessor, or occupant of any land who shall not
cities, it should not exceed five hundred (500) have voluntarily come in under the provisions of this
square meters; in first class and second class chapter or of the Land Registration Act, stating in
municipalities, it should not exceed seven hundred substance that the title of such holder, claimant, possessor,
fifty (750) square meters; and in all other or occupant is open to discussion; or that the boundaries of
municipalities, it should not exceed one thousand any such land which has not been brought into court as
(1,000) square meters; aforesaid are open to question; or that it is advisable that
4. Under a bona fide claim of ownership the title to such lands be settled and adjudicated, and
5. Land must be used exclusively for residential praying that the title to any such land or the boundaries
purposes thereof or the right to occupancy thereof be settled and
6. Provided, further, That the land applied for is not adjudicated. The judicial proceedings under this section
needed for public service and/or public use. shall be in accordance with the laws on adjudication of title
in cadastral proceedings.
a. New RA 10023
B. Section 35-38 of PD 1529
Section 1. Qualifications. - Any Filipino citizen who is an
actual occupant of a residential land may apply for a Free Section 35. Cadastral Survey preparatory to filing of
Patent Title under this Act: Provided; That in highly petition.
urbanized cities, the land should not exceed two hundred
(200) square meters; in other cities, it should not exceed (a) When in the opinion of the President of the Philippines
five hundred (500) square meters; in first class and second public interest so requires that title to any unregistered
class municipalities, it should not exceed seven hundred lands be settled and adjudicated, he may to this end direct
fifty (750) square meters; and in all other municipalities, it and order the Director of Lands to cause to be made a
should not exceed one thousand (1,000) square meters; cadastral survey of the lands involved and the plans and
Provided, further, That the land applied for is not needed technical description thereof prepared in due form.
for public service and/or public use.
(b) Thereupon, the Director of Lands shall give notice to
persons claiming any interest in the lands as well as to the
F. Special Patents general public, of the day on which such survey will begin,
giving as fully and accurately as possible the description of

Notes of Cesnee Tan Page 41


Natural Resources 1st Exam (2017-2018)

the lands to be surveyed. Such notice shall be punished possible. The parcels shall be known as "lots" and shall on
once in the Official Gazette, and a copy of the notice in the plan filed in the case be given separate numbers by the
English or the national language shall be posted in a Director of Lands, which numbers shall be known as
conspicuous place on the bulletin board of the municipal "cadastral lot numbers". The lots situated within each
building of the municipality in which the lands or any municipality shall, as far as practicable, be numbered
portion thereof is situated. A copy of the notice shall also consecutively beginning with number "one", and only one
be sent to the mayor of such municipality as well as to the series of numbers shall be used for that purpose in each
barangay captain and likewise to the Sangguniang municipality. However in cities or townsites, a designation
Panlalawigan and the Sangguniang Bayan concerned. of the landholdings by blocks and lot numbers may be
employed instead of the designation by cadastral lot
(c) The Geodetic Engineers or other employees of the numbers.
Bureau of Lands in charge of the survey shall give notice
reasonably in advance of the date on which the survey of The cadastral number of a lot shall not be changed after
any portion of such lands is to begin, which notice shall be final decision has been entered decreasing the registration
posted in the bulletin board of the municipal building of the thereof, except by order of court. Future subdivisions of any
municipality or barrio in which the lands are situated, and lot shall be designated by a letter or letters of the alphabet
shall mark the boundaries of the lands by monuments set added to the cadastral number of the lot to which the
up in proper places thereon. It shall be lawful for such respective subdivisions pertain. The letter with which a
Geodetic Engineers and other employees to enter upon the subdivision is designated shall be known as its "cadastral
lands whenever necessary for the purposes of such survey letter": Provided, however, that the subdivisions of cities or
or the placing of monuments. townsites may be designated by blocks and lot numbers.

(d) It shall be the duty of every person claiming an interest C. ANSWER


in the lands to be surveyed, or in any parcel thereof, to
communicate with the Geodetic Engineer upon his request Section 37. Answer to petition in cadastral proceedings. Any
therefor all information possessed by such person claimant in cadastral proceedings, whether named in the
concerning the boundary lines of any lands to which he notice or not, shall appear before the court by himself or by
claims title or in which he claims any interest. some other authorized person in his behalf, and shall file an
(e) Any person who shall willfully obstruct the making of answer on or before the date of initial hearing or within
any survey undertaken by the Bureau of Lands or by a such further time as may be allowed by the court. The
licensed Geodetic Engineer duly authorized to conduct the answer shall be signed and sworn to by the claimant or by
survey under this Section, or shall maliciously interfere with some other authorized person in his behalf, and shall state
the placing of any monument or remove such monument, whether the claimant is married or unmarried, and if
or shall destroy or remove any notice of survey posted on married, the name of the spouse and the date of marriage,
the land pursuant to law, shall be punished by a fine of not his nationality, residence and postal address, and shall also
more than one thousand pesos or by imprisonment for not contain:
more than one year, or both.
(a) The age of the claimant;
B. PETITION; LOT NUMBERS
(b) The cadastral number of the lot or lots claimed, as
Section 36. Petition for registration. When the lands have appearing on the plan filed in the case by the Director of
been surveyed or plotted, the Director of Lands, Lands, or the block and lot numbers, as the case may be;
represented by the Solicitor General, shall institute original
registration proceedings by filing the necessary petition in (c) The name of the barrio and municipality in which the
the Court of First Instance of the place where the land is lots are situated;
situated against the holders, claimants, possessors, or
occupants of such lands or any part thereof, stating in (d) The names and addresses of the owners of the
substance that public interest requires that the title to such adjoining lots so far as known to the claimant;
lands be settled and adjudicated and praying that such
titles be so settled and adjudicated: (e) If the claimant is in possession of the lots claimed and
can show no express grant of the land by the government
The petition shall contain a description of the lands and to him or to his predecessors-in-interest, the answer shall
shall be accompanied by a plan thereof, and may contain state the length of time he has held such possession and
such other data as may serve to furnish full notice to the the manner in which it has been acquired, and shall also
occupants of the lands and to all persons who may claim state the length of time, as far as known, during which the
any right or interest therein. predecessors, if any, held possession;

Where the land consists of two or more parcels held or (f) If the claimant is not in possession or occupation of the
occupied by different persons, the plan shall indicate the land, the answer shall fully set forth the interest claimed by
boundaries or limits of the various parcels as accurately as him and the time and manner of his acquisition;

Notes of Cesnee Tan Page 42


Natural Resources 1st Exam (2017-2018)

decree. In one sense, there is no plaintiff and there is no


(g) if the lots have been assessed for taxation, their last defendant. In another sense, the Government is the
assessed value; and plaintiff and all the- claimants are defendants.

(h) The encumbrances, if any, affecting the lots and the As a general rule, courts should adhere to the usual rules of
names of adverse claimants, as far as known. practice, procedure, and evidence that governs registration
proceedings. However, in registration proceedings where so
D. HEARING; JUDGMENT; DECREE many parties are involved and action is taken quickly and
abruptly, opportunity should be given to parties to submit
Section 38. Hearing, Judgment, Decree. The trial of the additional corroborative evidence in support of their claims
case may occur at any convenient place within the province of title, if the ends of justice so require. This case was
in which the lands are situated and shall be conducted, and remanded back to the lower court with the church being
orders for default and confessions entered, in the same allowed to admit additional testimonies in the interest of
manner as in ordinary land registration proceedings and justice and ascertainment of truth.
shall be governed by the same rules. All conflicting interests
shall be adjudicated by the court and decrees awarded in
favor of the persons entitled to the lands or to parts thereof Abellera vs Farol
and such decrees shall be the basis for issuance of original (GR No. 48480, 7/30/1943)
certificates of title in favor of said persons and shall have Abellera, in a previous case concerning the same real estate
the same effect as certificates of title granted on involved herein, sued Hermegildo Balanag and others who
application for registration of land under ordinary land are either the same parties in this case or the latter's
registration proceedings. predecessors in interest, alleging ownership of the land. But
his complaint was dismissed by the Court of First Instance
on two grounds:
Cases: (1) prescription in favor of defendants; and
(2) the deed of donation of these lands to him had not
Director of Lands vs. Roman Catholic Archbishop of been formally accepted according to Article 633 of the Civil
Manila Code.
(GR No. 14869) Upon appeal to this Court, the judgment of the trial court
FACTS: was affirmed on the second ground aforementioned. It
In 1913, cadastral proceedings were conducted to settle the appears in that decision of this Court that after the
title to a considerable tract of land in the Province of Rizal. perfection of the appeal, Abellera executed a public
The Roman Catholic Archbishop of Manila (church) and document formally accepting the donation of the land, and
other private parties were claimants of 13 cadastral lots presented and deed of acceptance together with proofs of
that comprised the contested property. The lower court notification of acceptance to the donor, as ground for new
ruled in favor of the private claimants. Upon appeal, the trial.
church invoked that the composition title of the church with In July of 1918, or four months after the above-mentioned
the Spanish Government included the subject property. The decision of this Court, petitioner herein brought another
church then presented one witness and rested. The private action for recovery of the land against the same defendants
oppositors then called their respective witnesses to prove in the previous case.
title by possession, and rested. The church thereafter made The second suit was later dismissed by the Court of First
an offer to present additional testimony intended to show Instance and transferred to cadastral case No. 5 which
that the possession of the private claimants had been included the hacienda in question that had in the meantime
interrupted and that it was merely possession through the been subdivided into lots.
tolerance of the church. However, the counsel for the The latter through counsel moved that Abellera's claim over
oppositors objected to the entry of additional testimonies the lots concerned be dismissed on the grounds of res
which was sustained. judicata and prescription.

ISSUE: Issue:
Did the lower court err in refusing the entry the church‘s Did the cadastral court, on the ground of res
additional testimonies? judicata, have any power to entertain the motion to dismiss
Abellera's claim and bar him from presenting evidence to
RULING: prove his ownership of these lots?
The Court ruled in the affirmative.
Ruling:
The object of a cadastral petition is that the title to the Rule 132 of the Rules of Court provides:
various lots embraced in the survey may be settled and These rules shall not apply to land registration, cadastral
adjudicated. It is in the nature of a proceeding in rem, and election cases, naturalization and insolvency
promoted by the Director of Lands, somewhat akin to a proceedings, and other cases not herein provided for,
judicial inquiry and investigation leading to a judicial except by analogy or in a suppletory character and

Notes of Cesnee Tan Page 43


Natural Resources 1st Exam (2017-2018)

whenever practicable and convenient. the person making the conveyance or encumbrance is able
The Rules of Court may be applied in cadastral cases when to read and can understand the language in which the
two conditions are present: (1) analogy or need to instrument or conveyance or encumbrances is written.
supplement the cadastral law, and (2) practicability and Conveyances and encumbrances made by illiterate non-
convenience. Christian or literate non-Christians where the instrument of
The principal aim is to settle as much as possible all conveyance or encumbrance is in a language not
disputes over land and to remove all clouds over land titles, understood by the said literate non-Christians shall not be
as far a practicable, in a community. valid unless duly approved by the Chairman of the
To attain this purpose, the cadastral court should allow all Commission on National Integration.
claimants ample freedom to ventilate whatever right they
may assert over real estate, permitting them, in keeping Section121. Except with the consent of the grantee and the
with the law of evidence, to offer proofs in support of their approval of the Secretary of Natural Resources, and solely
allegations. for commercial, industrial, educational, religious or
We are, therefore, of the opinion that while in a cadastral charitable purposes or for a right of way, no corporation,
case res judicata is available to a claimant in order to association, or partnership may acquire or have any right,
defeat the alleged rights of another claimant, nevertheless title, interest, or property right whatsoever to any land
prior judgment can not set up in a motion to dismiss. granted under the free patent, homestead, or individual
sale provisions of this Act or to any permanent
Prohibited Alienations and Transfer of Private Lands improvement on such land.

Now there are limitations in the ownership of lands. This is The provisions of Section 124 of this Act to the contrary
not absolute. Prohibited alienations from the issuance of notwithstanding, any acquisition of such land, rights thereto
the patent or the issuance of title. After the lapse of the or improvements thereon by a corporation, association, or
period, you can alienate it. Otherwise, the transfers will be partnership prior to the promulgation of this Decree for the
void. This alienation extends to the sale between sons or purposes herein stated is deemed valid and binding;
daughters as well as even only a portion is sold. Provided, That no final decision of reversion of such land to
the State has been rendered by a court; And Provided,
Transfers of lands to aliens are not allowed except in cases further, That such acquisition is approved by the Secretary
of intestate succession. Former natural born Filipinos who of Natural Resources within six (6) months from the
have reacquired their Filipino citizenship under RA 9225 effectivity of this Decree.
may acquire the lands in the Philippines.
Section122. No land originally acquired in any manner
A. SECTIONS 118-124 of CA 141 under the provisions of this Act, nor any permanent
improvement on such land, shall encumbered, alienated, or
Section118. Except in favor of the Government or any of its transferred, except to persons, corporations, associations,
branches, units, or institutions, lands acquired under free or partnerships who may acquire lands of the public domain
patent or homestead provisions shall not be subject to under this Act or to corporations organized in the
encumbrance or alienation from the date of the approval of Philippines authorized therefor by their charters.
the application and for a term of five years from and after
the date of issuance of the patent or grant, nor shall they Except in cases of hereditary succession, no land or any
become liable to the satisfaction of any debt contracted portion thereof originally acquired under the free patent,
prior to the expiration of said period, but the improvements homestead, or individual sale provisions of this Act, or any
or crops on the land may be mortgaged or pledged to permanent improvement on such land, shall be transferred
qualified persons, associations, or corporations. or assigned to any individual, nor shall such land or any
permanent improvement thereon be leased to such
No alienation, transfer, or conveyance of any homestead individual, when the area of said land, added to that of his
after five years and before twenty-five years after issuance own, shall exceed one hundred and forty-four hectares. Any
of title shall be valid without the approval of the Secretary transfer, assignment, or lease made in violation hereof, shall
of Agriculture and Commerce, which approval shall not be be null and void.
denied except on constitutional and legal grounds.
Section123. No land originally acquired in any manner
Section119. Every conveyance of land acquired under the under the provisions of any previous Act, ordinance, royal
free patent or homestead provisions, when proper, shall be order, royal decree, or any other provision of law formerly
subject to repurchase by the applicant, his widow, or legal in force in the Philippines with regard to public lands,
heirs, within a period of five years from the date of the terrenos baldios y realengos, or lands of any other
conveyance. denomination that were actually or presumptively of the
public domain, or by royal grant or in any other form, nor
Section120. Conveyance and encumbrance made by any permanent improvement on such land, shall be
persons belonging to the so-called "non-Christian Filipinos" encumbered, alienated, or conveyed, except to persons,
or national cultural minorities, when proper, shall be valid if corporations or associations who may acquire land of the

Notes of Cesnee Tan Page 44


Natural Resources 1st Exam (2017-2018)

public domain under this Act or to corporate bodies term of five (5) years from and after the date of issuance of
organized in the Philippines whose charters authorize them the patent or grant, lands acquired under free patent or
to do so: Provided, however, That this prohibition shall not homestead provisions cannot be subject to encumbrance or
be applicable to the conveyance or acquisition by reason of alienation, nor shall they become liable to the satisfaction
hereditary succession duly acknowledged and legalized by of any debt contracted prior to the expiration of said period.
competent courts; Provided, further, That in the event of The only exception mentioned by the law is the sale or
the ownership of the lands and improvements mentioned in encumbrance in favor of the government or any of its
this section and in the last preceding section being branches, units or institutions.
transferred by judicial decree to persons, corporations or In a number of cases, we have consistently ruled that a
associations not legally capacitated to acquire the same sale of homestead within the five (5) year prohibitive period
under the provisions of this Act, such persons, corporations, is void ab initio and the same cannot be ratified nor can it
or associations shall be obliged to alienate said lands or acquire validity through the passage of time.
improvements to others so capacitated within the precise
period of five years; otherwise, such property shall revert to Republic vs Garcia
the Government. (GR No. L-11597, 5/27/1959)
Facts:
Section124. Any acquisition, conveyance, alienation,
transfer, or other contract made or executed in violation of Garcia was granted a homestead patent for his
any of the provisions of sections one hundred and eighteen, 23.21 hectares of land. After 3 years and 3 months later
one hundred and twenty, one hundred and twenty-one, one (April 14, 1950), he sold 19 hectares to the lot to several
hundred and twenty two, and one hundred and twenty- persons. For this reason, the CFI of Cotabato decreed the
three of this Act shall be unlawful and null and void from its reversion of the property in favor of the Government for
execution and shall produce the effect of annulling and violating the selling the property within the prohibitive
cancelling the grant, title, patent, or permit originally period of 5 years from date of issuance under Sec. 118 of
issued, recognized or confirmed, actually or presumptively, CA 141.
and cause the reversion of the property and its
improvements to the State. Garcia contends that he did not violate the said provision
since the sale was not registered and that it was not the
Cases: entire land was sold.

Gayapanao vs. IAC Issue: Whether or not Garcia violated the Sec. 118 of CA
(GR No. 68109, 7/17/1991) 141.
FACTS: This is a petition for review on certiorari filed by
Severino Gayapanao and his siblings questioning the Held. Yes, he did.
decision of the IAC in upholding the validity of the sale of
the land between their father Constantino and their sister. The Supreme Court held that it is enough that the property,
The 2 hectare land, subject of this case is part of 10 in whole or in part, was alienated or encumbered within the
hectare homestead land registered in the name of prohibitive period except in favor of the Government, Sec.
Constantino Gayapanao under OCT. The final order of the 118 of CA 141 partly provides: Except in favor of the
Director of Lands for the issuance of patent was issued on Government or any its branches, units, or institutions, lands
December 10, 1937. On November 15, 1938, Constantino acquired under free patent or homestead provision shall not
Gayapanao executed a private deed entitled kasulatan ng be subject to encumbrance of alienation from the date of
bilihan in favor of Simeona Gayapanao and his husband. the approval of the application and for a term of five years
from and after the date of issuance of the patent or grant,
RTC‘s decision: The contract of sale between Simeona nor shall they become liable to the satisfaction of any debt
Gayapanao and her father is null and void for having been contracted prior to the expiration of said period, but the
executed with the 5 year prohibitory period provided under improvements or crops on the land may be mortgaged or
Section 118 of the Public Land Law pledged to qualified persons, association, or corporations.
CA‘s decision: It reversed the decision of the RTC and
uphold the validity of the sale. Such alienation is a sufficient cause for reversion to the
State of the whole grant. In granting a homestead to an
ISSUE: WON CA was correct in upholding the validity of the applicant, the law imposes as a condition that the land
sale? should not be encumbered, sold or alienated within five
years from the issuance of the patent. Sec. 124 of CA 141
RULING: provides: Any acquisition, conveyance, alienation, transfer,
No, the provision of law which prohibits the sale or or other contract made or executed in violation of any of
encumbrance of the homestead within 5 years after the the provisions of section118, 120, 121, 122, and 123 of this
grant is mandatory. Act shall be unlawful and null and void from its execution
and shall produce the effect of annulling and cancelling the
From the date of the approval of the application and for a grant, title, patent, or permit originally issued, recognized

Notes of Cesnee Tan Page 45


Natural Resources 1st Exam (2017-2018)

or confirmed, actually or presumptively, and cause the Lee Chuy filed a motion for reconsideration but was denied
reversion of the property and its improvements to the
State.
Issue: WON a formal offer to redeem accompanied with
Lee Chuy Realty Corp vs Court of Appeals tender of payment a condition precedent to the filing of an
(GR No. 104114, 12/4/1995) action for the valid exercise of the right of legal
redemption; is the filing of the action with consignation
A valuable piece of land located at Meycauyan, Bulacan, equivalent to a formal offer to redeem
with an area of 24,576 sq. m. and covered by OCT No. 0-
5290 is disputed by Lee Chuy Realty Corporation and Marc Held: No.In Hulganza v. Court of Appeals14 the Court,
Realty and Development Corp. Such land was originally co- citing previous decisions, declared that the formal offer to
owned by Ruben Jacinto(one-sixth), Dominador, Arsenio, redeem, accompanied by a bona fide tender of the
Liwayway all surnamed Bascara and Ernesto redemption price, within the prescribed period is only
jacinto(collectively owned the remaining five-sixths). essential to preserve the right of redemption for future
enforcement beyond such period of redemption and within
On Feb. 4, 1981, Ruben Jacinto sold his one-sixth pro- the period prescribed for the action by the statute of
indiviso share to LEE CHUY REALTY which was registered limitations. Where, as in the instant case, the right to
30 April 1981. On 5 May 1989 the Bascaras and Ernesto redeem is exercised through judicial action within the
Jacinto also sold their share to MARC REALTY which was reglementary period the formal offer to redeem,
registered on 16 October 1989. accompanied by a bona fide tender of the redemption
Lee Chuy Realty claims that it was never informed of the price, while proper, may be unessential. The filing of the
existence of the sale between Marc Realty and the action itself is equivalent to a formal offer to redeem.
Bascaras/Jacinto. Marc Realty insists that Lee Chuy verbally
notified of the sale and was given a copy of the deed of In sum, the formal offer to redeem is not a distinct step or
sale. condition sine qua non to the filing of the action in Court
for the valid exercise of the right of legal redemption. What
On 13 November 1989 LEE CHUY REALTY filed a complaint constitutes a condition precedent is either a formal offer to
for legal redemption against MARC REALTY and consigned redeem or the filing of an action in court together with the
in court a manager's check for 614,400. MARC REALTY consignation of the redemption price within the
insisted that the complaint be dismissed for failure to state reglementary period.
a cause of action there being no allegation of prior valid
tender of payment or a prior valid notice of consignation. The doctrine in Tolentino, Tioseco and Belisario cases was
jettisoned by the Court of Appeals on the ground that they
On Dec 26, 1990, the trial court ruled in favour of Lee Chuy do not involve legal redemption by a co-owner but by a
Realty which stated that there was a valid tender of mortgagor. It concluded that the application of the rules on
payment and consignation. It also stated that neither a legal redemption by a co-owner differs from the legal
separate offer to redeem nor a formal notice of redemption by a mortgagor. But the law does not
consignation is necessary for the reason that the filing of distinguish; neither should we. For sure, the principle in the
the action itself, within the period of redemption, is aforecited cases is applicable regardless of whether the
equivalent to a formal offer to redeem. redemptioner is a co-owner or a mortgagor. Public policy
favors redemption regardless of whether the redemptioner
On 1 February 1991 MARC REALTY filed a Petition for is a co-owner or mortgagor, although perhaps with unequal
Certiorari, Prohibition with Temporary Restraining Order force and effect since each is given a fixed but different
and/or Writ of Preliminary Injunction which was referred to period. A co-owner desirous of exercising his right of legal
the Court of Appeals. The CA reversed the decision of the redemption is given a period of thirty (30) days from notice
lower court and ruled that "a prior tender or offer of of the sale within which to avail of the right to redeem.15
redemption is a prerequisite or precondition to the filing of Under the free patent or homestead provisions of the Public
an action for legal redemption‖ and that "there must be Land Act a period of five (5) years from the date of
tender of the redemption price within the required period conveyance is provided,16 the five-year period to be
because the policy of the law is not to leave the purchaser's reckoned from the date of the sale and not from the date of
title in uncertainty beyond the established 30-day period. registration in the office of the Register of Deeds.17 The
redemption of extrajudicially foreclosed properties, on the
MARC REALTY contends that prior tender of payment is a other hand, is exercisable within one (1) year from the date
condition precedent to the filing of an action in court in of the auction sale as provided for in Act No. 3135.
order to validly exercise the right of legal redemption. LEE
CHUY REALTY however argues that the filing of the action B. SECTION 7, ARTICLE XII, 1987 Constitution
itself is equivalent to a formal offer to redeem, which is a
condition precedent to the valid exercise of the right of SECTION 7. Save in cases of hereditary succession, no
legal redemption. private lands shall be transferred or conveyed except to
individuals, corporations, or associations qualified to

Notes of Cesnee Tan Page 46


Natural Resources 1st Exam (2017-2018)

acquire or hold lands of the public domain. Hence, petition was dismissed.

SECTION 8. Notwithstanding the provisions of Section 7 of


this Article, a natural-born citizen of the Philippines who has Frenzel vs Catito
lost his Philippine citizenship may be a transferee of private (GR No. 143958, July 11, 2003)
lands, subject to limitations provided by law. FACTS: Petitioner – Alfred Fritz Frenzel, is an Australian
Electrical Engineer who once worked with New Guinea
Cases: Airlines, started doing business in the Philippines since
1974, and two years later got married to a Filipina Teresita
Ong Ching Po vs. Court of Appeals Santos. They were separated bread and board in 1981 and
(239 SCRA 341) two years later met Ederlina Catito as a masseuse in one of
FACTS: night clubs in Sydney Australia
• Ong Joi Jong, sold a parcel of land located at
Fundidor Street, San Nicolas to private respondent Soledad Frenzel got enamored with Ederlina and convinced her to
Parian on 23 July 1947. The sale was evidenced by a setup wholesome business in Manila for her own as he was
notarized Deed of Sale, written in English, and was willing to help her. Ederlina returned and later Alfred
registered with the Register of Deeds of Manila which in followed and helped her setup a Edorial Beauty saloon with
turn issued TCT no. 9260 dated 2 September 1947 in the his personal funds –let alone Ederlina as sole registered
name of private respondent. According to private owner. Alfred went back to work in Papue New Guinea as
respondent, she entrusted the administration of the lot and airline pilot while Ederlina too went to Germany to file her
building to Ong Ching Po when she and her husband settled divorce with her husband –Muller.
in Iloilo, however when her husband died she demanded
that the lot be vacated since she was going to sell it, When Alfred returned to the Philippines to live with
petitioners refused to vacate said premises. On 19 March Ederlina–he bought house and lot or real properties in QC
1984, Parian filed a case for unlawful detainer against Ong for her with his own funds and agreeing to name it under
Ching Po. The MTC dismissed her case and was reaffirmed Ederlina alone accepting that as a foreigner he is not
by both the RTC and CA. allowed to own a real estate property under the Philippine
Laws. Alfred decided to stay for good in the Philippines and
• On the other hand, on 6 December 1983, Ong sold his personal properties in Australia with proceeds later
Ching Po executed a Deed of Absolute Sale conveying the used to buy other 2-more properties in Davao City –again,
same property to his children Jimmy and David Ong. On 12 in Ederlina‘s name only. There was also an occasion when
December 1985, the Ong‘s filed an action for reconveyance Ederlina deposited some USD 250,000 in their joint-
and damages against Parian in RTC, Manila. Upon the accounts.
private respondent‘s motion, this was consolidated with her
action for quieting of title against the petitioners. The RTC Meanwhile, Ederlina‘s divorce case with her German
rendered a decision in favor of private respondent and was husband was denied and with the prospect that she could
confirmed by the CA, hence this petition. be charged with bigamy along with Alfed who himself is still
married. This started their common law marriage
ISSUE: Whether or not Ong Ching Po could legally acquire deterioration and soon enough Alfred got penniless and
the property? filed his claim on the properties listed in Ederlina‘s name
alone as well as charges the return of all other asset
HELD: No. Petition is dismissed. bought with his own funds.

RATIO: Ong Ching Po cannot legally claim ownership of ISSUE: Will the good faith doctrine and applicable Articles
the disputed property since the capacity to acquire private in Civil code 1416, In combination with RA Nos. 133,
lands is dependent on the capacity to acquire or hold lands 4381 and Rep. Act No. 4882, the proceeds of the sale
of the public domain. Private land may be transferred or would be remitted to him, by way of refund for the money
conveyed only to individuals or entities ―qualified to acquire he used to purchase the said properties. To bar the
lands of the public domain‖. Petitioner Ong Ching Po was a petitioner from recovering the subject properties, or at the
Chinese citizen and therefore is disqualified from acquiring very least, the money used for the purchase and to bar the
and owning real property. Further, despite the documentary petitioner to the proceeds thereof is to enrich the
evidence provided by petitioners, they failed to provide respondent at his expense, and can this withstand the
evidence as to the genuineness and due execution of the Constitutional proscription on the property ownership for
deed of sale. Likewise, the tax receipts, tax declaration, the aliens in the Philippines?
rental receipts, deed of sale and TCT were in Ong‘s
possession, these were not reflective of dominion or Hence, this petition, after successive defeat from lower and
ownership as even a mere administrator or manager may appellate court.
lawfully perform payment duties relative to his appointment
as such. It was also noteworthy that the tax receipts and HELD No. The petition is Denied.
rental receipts were in the name of Parian‘s husband.

Notes of Cesnee Tan Page 47


Natural Resources 1st Exam (2017-2018)

The Court ruled on evidence on record: to wit; to recover the funds used to buy it since it was purchased
in violation of Section 7, Article XII of the Constitution.
―The three parcels of land subject of the complaint were Upon appeal, the Court of Appeals granted Helmut Muller‘s
not mortgaged to the petitioner by the owners thereof but prayer for reimbursement for the Antipolo property.
were sold to the respondent as the vendee, albeit with the
use of the petitioners personal funds.‖ The sales of three ISSUE: WON Helmut Muller is entitled to reimbursement of
parcels of land in favor of the petitioner who is a foreigner the funds used to acquire the Antipolo property.
is illegal per se. The transactions are void ab initio because
they were entered into in violation of the Constitution. HELD: NO. Respondent was aware of the constitutional
Thus, to allow the petitioner to recover the properties or prohibition and expressly admitted his knowledge thereof to
the money used in the purchase of the parcels of land this Court. He declared that he had the Antipolo property
would be subversive of public policy. titled in the name of petitioner because of the said
prohibition. His attempt at subsequently asserting or
This finds support under Section 7, Article XII of the 1987 claiming a right on the said property cannot be sustained.
Philippine Constitution which states: ―Save in cases of Thus, in the instant case, respondent cannot seek
hereditary succession, no private lands shall be transferred reimbursement on the ground of equity where it is clear
or conveyed except to individuals, corporations or that he willingly and knowingly bought the property despite
associations qualified to acquire or hold lands of the public the constitutional prohibition. To allow reimbursement
domain.‖ would in effect permit respondent to enjoy the fruits of a
property which he is not allowed to own. Thus, it is likewise
The court stated: MEMO CUM ALTERIUS DETER proscribed by law.
DETREMENTO PROTEST (No person should unjustly enrich
himself at the expense of another). An action for recovery Lee vs. Director of Lands
of what has been paid without just cause has been (GR No. 128195, October 3, 2001)
designated as an accion in rem verso. Facts: Sometime in March 1936, Carmen, Francisco, Jr.,
Ramon, Lourdes, Mercedes, Concepcion, Mariano, Jose,
This provision does not apply if, as in this case, the action is Loreto, Manuel, Rizal, and Jimmy, all surnamed Dinglasan
proscribed by the Constitution or by the application of the sold to Lee Liong, A Chinese citizen, a parcel of land with
pari delicto doctrine. an approximate area of 1,631 square meters, designed as
lot 398 and covered by Original Certificate of Title No.
It may be unfair and unjust to bar the petitioner from filing 3389.
an accion in rem verso over the subject properties, or from However, in 1948, the former owners filed with the Court of
recovering the money he paid for the said properties, but, First Instance, Capiz an action against the heirs of Lee
as Lord Mansfield stated in the early case of Holman vs. Liong for annulment of sale and recovery of land. The
Johnson. ‖The objection that a contract is immoral or illegal plaintiffs assailed the validity of the sale because of the
as between the plaintiff and the defendant, sounds at all constitutional prohibition against aliens acquiring ownership
times very ill in the mouth of the defendant. It is not for his of private agriculture land, including residential, commercial
sake, however, that the objection is ever allowed; but it is or industrial land.
founded in general principles of policy, which the defendant Plaintiffs appealed to the Supreme Court and ruled thus:
has the advantage of, contrary to the real justice, as pari delicto (in sales of real estate to aliens incapable of
between him and the plaintiff, therefore, the petition is holding title thereto by virtue of provision of the
bereft of merits. Constitution, both the vendor and vendee are deemed to
have committed the constitutional violation and thus the
Muller vs. Muller courts will not afford protection to either party).
(GR no. 149615, August 29, 2006) On July 1, 1968, the same former owners (Dinglasans) filed
Elena Buenaventura Muller and Helmut Muller are husband with the Court of First Instance and action for recovery of
and wife in this case. They wed and resided in Germany the same parcel of land. On Sept. 23, 1968, the heirs of
until they decided to permanently reside in the Philippines Lee Liong file with the trial. Both cases were elevated to
in 1992. By this time, they bought a house in Antipolo, Rizal the Supreme Court but were dismissed holding the suit
using the proceeds that they got from selling the house the barred by res judicata. On Sept. 7, 1993, Elizabeth Manuel-
Helmut Muller inherited from his parents in Germany. Lee and Pacita Yu Lee filed with the RTC, Roxas City a
The marriage, however, did not last. They were eventually petition for reconstitution of title of Lot No. 398 of Capiz.
separated and Helmut Muller filed for separation of their They were the widows of the heirs of Lee Liong, the owner
properties. The trial court then dissolved the absolute of the lot. Both widows received a parcel of land through
community of property and ordered the equal partition of succession from their deceased husbands. RTC ordered the
their personal properties located within the country, reconstitution of the lost and destroyed certificate of said
excluding those acquired by gratuitous title during title of lot. On Jan. 25, 1995, OSG filed with the Court of
marriage. As to the Antipolo property, the court held that it Appeals a petition for annulment of judgment of
was acquired using the paraphernal funds of Helmut, reconstitution alleging that petitioners were not the proper
however, he cannot recover said property, nor have a right parties in the reconstitution of title, since Lee Liong did not

Notes of Cesnee Tan Page 48


Natural Resources 1st Exam (2017-2018)

acquire title to the lot because he was a Chinese citizen and Wanda.
was constitutionally not qualified to own the subject land. Jorge and Roberto opposed the project of partition on the
CA decided, declaring the judgment of reconstitution to be grounds: (a) that the provisions for vulgar substitution in
void. Hence this petition. favor of Wanda de Wrobleski with respect to the widow's
usufruct and in favor of Juan Pablo Jankowski and Horacio
Issue/s: V. Ramirez, with respect to Wanda's usufruct are invalid
1. WON OSG has the capacity to object the reconstitution. because the first heirs Marcelle and Wanda) survived the
2. WON constitutional prohibition still exists considering the testator; (b) that the provisions for fideicommissary
land is now under the ownership of Lee‘s heirs thru substitutions are also invalid because the first heirs are not
succession. related to the second heirs or substitutes within the first
degree, as provided in Article 863 of the Civil Code; (c) that
Held: the grant of a usufruct over real property in the Philippines
1. Yes. in favor of Wanda Wrobleski, who is an alien, violates
The fact that the Court did not annul the sale of the land t Section 5, Article III of the Philippine Constitution; and that
an alien did not validate the transaction. For it was still (d) the proposed partition of the testator's interest in the
contrary to the constitutional proscription against aliens Santa Cruz (Escolta) Building between the widow Marcelle
acquiring lands of the public or private domain. However, and the appellants, violates the testator's express win to
the proper party to assail the illegality of the transaction as give this property to them. Nonetheless, the lower court
was not the parties to the transaction. The proper party to approved the project of partition in its order dated May 3,
assail the sale is the Solicitor General. This was what was 1967.
done in this case when the Solicitor General initiated an
action for annulment of judgement of reconstitution of title. ISSUE: Whether or not the usufruct over real properties of
While it took the Republic more than sixty years to assert it, the estate in favor of Wanda is void because it violates the
it is not barred from initiating such action. Prescription constitutional prohibition against the acquisition of lands by
never lies against the State. aliens.
2. No.
The constitutional proscription on alien ownership of lands RULING: The 1935 Constitution which is controlling
of the public or private domain was intended to protect provides as follows: SEC. 5. Save in cases of hereditary
lands from falling in the hands of non- Filipinos. In this succession, no private agricultural land shall be transferred
case, however, there would be no more public policy or assigned except to individuals, corporations, or
violated since the land is in the hands of Filipinos qualified associations qualified to acquire or hold lands of the public
to acquire and own such land. ―If the land is invalidly domain in the Philippines. (Art. XIII.)
transferred to an alien who subsequently becomes a citizen The court a quo upheld the validity of the usufruct given to
or transfers it to a citizen, the flaw in the original Wanda on the ground that the Constitution covers not only
transaction is considered cured and the title of the succession by operation of law but also testamentary
transferee is rendered valid.‖ succession. We are of the opinion that the Constitutional
provision which enables aliens to acquire private lands does
Ramirez vs. Vda de Ramirez not extend to testamentary succession for otherwise the
(111 SCRA 704) prohibition will be for naught and meaningless. Any alien
FACTS OF THE CASE: The main issue in this appeal is the would be able to circumvent the prohibition by paying
manner of partitioning the testate estate of Jose Eugenio money to a Philippine landowner in exchange for a devise
Ramirez among the principal beneficiaries, namely: his of a piece of land.
widow Marcelle Demoron de Ramirez; his two
grandnephews Roberto and Jorge Ramirez; and his This opinion notwithstanding, we uphold the usufruct in
companion Wanda de Wrobleski. favor of Wanda because a usufruct, albeit a real right, does
Jose Eugenio Ramirez, a Filipino national, died in Spain on not vest title to the land in the usufructuary and it is the
December 11, 1964, with only his widow as compulsory vesting of title to land in favor of aliens which is proscribed
heir. His will was admitted to probate by the Court of First by the Constitution.
Instance of Manila, on July 27, 1965. Maria Luisa Palacios
was appointed administratrix of the estate. The widow In view of the foregoing, the estate of Jose Eugenio
Marcelle is a French who lives in Paris, while the companion Ramirez was ordered distributed as follows:
Wanda is an Austrian who lives in Spain. Moreover, the
testator provided for substitutions. On June 23, 1966, the One-half (1/2) thereof to his widow as her legitime;
administratrix submitted a project of partition as follows:
the property of the deceased is to be divided into two One-half (1/2) thereof which is the free portion to Roberto
parts. One part shall go to the widow in satisfaction of her and Jorge Ramirez in naked ownership and the usufruct to
legitime; the other part or "free portion" shall go to Jorge Wanda de Wrobleski with a simple substitution in favor of
and Roberto Ramirez. Furthermore, one third (1/3) of the Juan Pablo Jankowski and Horace V. Ramirez.
free portion is charged with the widow's usufruct and the
remaining two-thirds (2/3) with a usufruct in favor of The Supreme Court held that they are allowed because

Notes of Cesnee Tan Page 49


Natural Resources 1st Exam (2017-2018)

they are former natural born Filipinos. That is explicitly Constitution.


provided under the Constitution. The limitation only
provided is under the Foreign Investments Act limiting the In the case at bar, the spouses were
size of the area. undoubtedly natural-born Filipino citizens at the time of the
acquisition of the properties and by virtue thereof, acquired
Republic vs Court of Appeals vested rights thereon, tacking in the process, the
(235 SCRA 567) possession in the concept of owner and the prescribed
On June 17, 1978, respondent spouses bought Lots, as period of time held by their predecessors-in-interest under
their residence with a total area of 91.77 sq. m. situated in the Public Land Act.
San Pablo City, from one Cristeta Dazo Belen. At the time of
the purchase, respondent spouses where then natural-born But what should not be missed in the
Filipino citizens. disposition of this case is the fact that the Constitution itself
allows private respondents to register the contested parcels
On February 5, 1987, the spouses filed an application for of land in their favor. Sections 7 and 8 of Article XII of the
registration of title of the two (2) parcels of land before the Constitution contain the following pertinent provisions, to
RTC of San Pablo City. This time, however, they were no wit:
longer Filipino citizens and have opted to embrace
Canadian citizenship through naturalization. Sec. 8. Notwithstanding the provisions of Section 7 of this
Article, a natural-born citizen of the Philippines who has lost
RTC rendered a decision confirming the title of the his Philippine citizenship may be a transferee of private
Spouses. Republic filed an opposition to the decision of lands, subject to limitations provided by law. (Emphasis
RTC. On appeal, CA affirmed the decision of RTC. supplied)

Republic submits that private respondents have not Section 8, Article XII of the 1987 Constitution above quoted
acquired proprietary rights over the subject properties is similar to Section 15, Article XIV of the then 1973
before they acquired Canadian citizenship through Constitution which reads:
naturalization to justify the registration thereof in their
favor. It maintains that even privately owned unregistered Sec. 15. Notwithstanding the provisions of Section 14 of
lands are presumed to be public lands under the principle this Article, a natural-born citizen of the Philippines who has
that lands of whatever classification belong to the State lost his citizenship may be a transferee of private land, for
under the Regalian doctrine. Thus, before the issuance of use by him as his residence, as the Batasang Pambansa
the certificate of title, the occupant is not in the juridical may provide.
sense the true owner of the land since it still pertains to the
State. Petitioner further argued that it is only when the Pursuant thereto, Batas Pambansa Blg. 185 was passed into
court adjudicates the land to the applicant for confirmation law, the relevant provision of which provides:
of title would the land become privately owned land, for in
the same proceeding, the court may declare it public land, Sec. 2. Any natural-born citizen of the Philippines who has
depending on the evidence. lost his Philippine citizenship and who has the legal capacity
to enter into a contract under Philippine laws may be a
ISSUE: WON a foreign national apply for registration of transferee of a private land up to a maximum area of one
title over a parcel of land which he acquired by purchase thousand square meters, in the case of urban land, or one
while still a citizen of the Philippines, from a vendor who hectare in the case of rural land, to be used by him as his
has complied with the requirements for registration under residence. In the case of married couples, one of them may
the Public Land Act (CA 141)? YES avail of the privilege herein granted; Provided, That if both
shall avail of the same, the total area acquired shall not
HELD: exceed the maximum herein fixed.
Since the spouses‘ predecessor in interest has In case the transferee already owns urban or rural lands for
been in open, continuous and exclusive possession for at residential purposes, he shall still be entitled to be a
least 30 years of alienable public land, such possession ipso transferee of an additional urban or rural lands for
jure converts the same to private property (Recall residential purposes which, when added to those already
ruling of Director of Lands v. IAC). This means that owned by him, shall not exceed the maximum areas herein
occupation and cultivation for more than 30 years by an authorized.
applicant and his predecessors-in-interest, vest title on such
applicant so as to segregate the land from the mass of From the adoption of the 1987 Constitution up to the
public. present, no other law has been passed by the legislature on
the same subject. Thus, what governs the disposition of
What is important is that private respondents were formerly private lands in favor of a natural-born Filipino citizen who
natural-born citizens of the Philippines, and as transferees has lost his Philippine citizenship remains to be BP 185.
of a private land, they could apply for registration in
accordance with the mandate of Section 8, Article XII of the Even if private respondents were already Canadian citizens

Notes of Cesnee Tan Page 50


Natural Resources 1st Exam (2017-2018)

at the time they applied for registration of the properties in complaint before the CFI Cotabato to declare the FP and
question, said properties as discussed above were already OCT void and order the reversion of the lang to the mass of
private lands; consequently, there could be no legal public domain on the ground that the land covered was
impediment for the registration thereof by respondents in timber/forest which is not disposable under PLA. The said
view of what the Constitution ordains. The parcels of land lot was plotted by the Bureau of Forestry LC 700 to be
sought to be registered no longer form part of the public inside the area which was reverted to the catefory of public
domain. They are already private in character since private forest, whereas the application of Timbol was filed on 06-3-
respondents' predecessors-in-interest have been in open, 1969 11 years after, That the FP was obtained fraudulently
continuous and exclusive possession and occupation as Timbol never occupied and cultivated the land.
thereof under claim of ownership prior to June 12, 1945 or • Timbol invoked the ruling of Ramirez vs
since 1937. The law provides that a natural-born citizen of Ramirez which ruled that Certificate of Title based on the
the Philippines who has lost his Philippine citizenship may patent had became indefeasible in view of the lapse of the
be a transferee of a private land up to a maximum area of one-year period prescribed under Section 38 of the Land
1,000 sq.m., if urban, or one (1) hectare in case of rural Registration Act for review of a decree of title on the
land, to be used by him as his residence (BP 185). ground of fraud. The Court ruled to dismiss the complaint
hence this petition.
It is undisputed that private respondents, as vendees of a
private land, were natural-born citizens of the Philippines. Issue: WON the Director of Lands under the case of
For the purpose of transfer and/or acquisition of a parcel of Ramirez or the Bureau of Forestry may initiated reversion of
residential land, it is not significant whether private the subject land.
respondents are no longer Filipino citizens at the time they
purchased or registered the parcels of land in question. Held: NO.Timbol‘s Free Patent was void
What is important is that private respondents were formerly • The defense of indefeasibility of a certificate of
natural-born citizens of the Philippines, and as transferees title issued pursuant to a free patent does not lie against
of a private land, they could apply for registration in the state in an action for reversion of the land covered
accordance with the mandate of Section 8, Article XII of the thereby when such land is a part of a public forest or of a
Constitution. Considering that private respondents were forest reservation.
able to prove the requisite period and character of • It is the Bureau of Forestry (Not Director of
possession of their predecessors-in-interest over the subject Lands like in Ramirez case) that has jurisdiction and
lots, their application for registration of title must perforce authority over the demarcation, protection, management,
be approved. reproduction, occupancy and use of all public forests and
forest reservations and over the granting of licenses for the
Remedies to recover private land from disqualified taking of products therefrom, including stone and earth
Alien: Action for Reversion (Section 1816 of the Revised Administrative Code).
• (note in Ramirez, no forest land was involved,
If your client is an individual who has a pre-existing right but only Public Agricultural land).
over the property, you file an action for cancellation. • A patent is void at law if the officer who issued
the patent had no authority to do so.
A. Who can initiate • In the case at bar the party seeking the nullity
Cases: of the title and reversion of the land is the state itself which
Republic vs Animas is specifically authorized under Section 101 of the Public
(GR No. L-37682, 3/29/1974) Land Act to initiate such proceedings as an attribute of
(NAG PATENT SYA SA SOUTH COT THEN TAGA DAVAO sovereignty, a remedy not available to a private individual.
SYA) • The complaint alleges in its paragraph 8 that
applicant Isagani Du Timbol was never in possession of the
Facts: land. He is living in Davao City and the land is in South
The land covered by the free patent and title in question Cotabato, it was fenced by one Hermogenes Chilsot;no
was originally applied for by Precila Soria, who on February improvements nor cultivation introduced thereof and it was
23, 1966, transferred her rights to the land and its not surveyed.
improvements to defendant Isagani Du Timbol who filed his
application therefor on February 3, 1969, as a transferee B. When Proper/improper
from Precila Soria.
Cases:
Republic vs Umali
• 12 12, 1969:Free Patent was issued by the (GR No. 80687, 4/10/1989)
President of Phils. The land in question is situated in Tanza, Cavite, and
• 07-20-1970 after the transmittal of the patent consists of 78,865 square meters. It was originally
to ROD Gensan, OCT was issued in the name of Isagani Du purchased on installment from the government on July 1,
Timbol 1910 by Florentina Bobadilla, who allegedly transferred her
• 08-5-1971, Bureau of Forestry filed a rights thereto in favor of Martina, Tomasa, Gregorio and

Notes of Cesnee Tan Page 51


Natural Resources 1st Exam (2017-2018)

Julio, all surnamed Cenizal, in 1922. Tomasa and Julio We find that the private respondents are transferees in
assigned their shares to Martina, Maria and Gregorio. In good faith and for value of the subject property and that
1971, these three assignees purportedly signed a joint the original acquisition thereof, although fraudulent, did not
affidavit which was filed with the Bureau of Lands to affect their own titles. These are valid against the whole
support their claim that they were entitled to the issuance world, including the government.
of a certificate of title over the said land on which they said
they had already made full payment. On the basis of this Piñero vs Director of Lands
affidavit, the Secretary of Agriculture and Natural Resources (GR No. L-36507, 12/14/1974)
executed Deed No. V-10910 (Sale Certificate No. 1280) on We hold the appeal to be meritorious. In the light of the
September 10, 1971, in favor of the said affiants. facts disclosed in the foregoing stipulation, We reiterate
Subsequently, on October 13, 1971, TCT No. 55044 Cebedo vs. Director of Lands, G.R. No. L-12777, May 22,
(replacing Bobadilla's OCT No. 180) was issued by the 1961, 2 SCRA 25, wherein We held that it is not only the
register of deeds of Cavite in favor of Maria Cenizal, right but the duty of the Director of Lands to conduct the
Gregorio Cenizal, and (in lieu of Martina Cenizal) Rosalina investigation of any alleged fraud in securing a free patent
Naval, Luz Naval, and Enrique Naval. and the corresponding title to a public land and to file the
corresponding court action for the reversion of the same to
When the complaint for reversion was filed on October 10, the State, if the facts disclosed in the course of such
1985, the registered owners of the land, following several investigation should so warrant. Consequently, prohibition
transfers, were Remedios Miclat under TCT No. 80392, Juan cannot be issued to enjoin such an investigation despite the
C. Pulido under TCT No. 80393, and Rosalina, Luz and existence of a Torrens title.
Enrique Naval under TCT No. 80394. They were named as
defendants and asked to return the property to the State Indeed, it is to be clarified that Section 91 of the Public
on the aforestated grounds of forgery and fraud. The Land Act leaves no other alternative to the Director of
plaintiff claimed that Gregorio Cenizal having died on Lands. The provision reads thus:
February 25, 1943, and Maria Cenizal on January 8, 1959,
they could not have signed the joint affidavit dated August SEC. 91. The statements made in the application shall be
9, 1971, on which Deed No. V-10910 (Sale Certificate No. considered as essential conditions and parts of any
1280) was based. The present holders of the property concession, title, or permit issued on the basis of such
claiming to be innocent purchasers for value and not privy application, and any false statement therein or omission of
to the alleged forgery, contend that the action cannot lie facts altering, changing, or modifying the consideration of
against them. the facts set forth in such statements, and any subsequent
modification, alteration, or change of the material facts set
ISSUE: forth in the application shall ipso facto produce the
WON the sale itself be considered null and void from the cancellation of the concession, title, or permit granted. It
start, as the petitioner insists, so as to make all titles shall be the duty of the Director of Lands, from time to time
derived therefrom also ineffectual, void ab initio. and whenever he may deem it advisable, to make the
necessary investigations for the purpose of ascertaining
HELD: whether the material facts set out in the application are
No. Section 44 of P.D. 1529 (then Sec. 39 of the Land Reg. true, or whether they continue to exist and are maintained
Act), states that, every registered owner receiving a and preserved in good faith, and for the purpose of such
certificate of title in pursuance of a decree of registration, investigation, the Director of Lands is hereby empowered to
and every subsequent purchaser of registered land taking a issue subpoenas and subpoenas duces tecum and, if
certificate of title for value and in good faith, shall hold the necessary, to obtain compulsory process from the courts. In
same free from all encumbrances except those noted on every investigation made in accordance with this section,
the certificate and any of the encumbrances which may be the existence of bad faith, fraud, concealment, or
subsisting, and enumerated in the law. Under said fraudulent and illegal modification of essential facts shall be
provision, claims and liens of whatever character, except presumed if the grantee or possessor of the land shall
those mentioned by law as existing against the land prior to refuse or fail to obey a subpoenas or subpoenas duces
the issuance of certificate of title, are cut off by such tecum lawfully issued by the Director of Lands or his
certificate if not noted thereon, and the certificate so authorized delegates or agents, or shall refuse or fail to
issued binds the whole world, including the government. give direct and specific answers to pertinent questions, and
The Torrens system was adopted in this country because it on the basis of such presumption, an order of cancellation
was believed to be the most effective measure to guarantee may issue without further proceedings.
the integrity of land titles and to protect their indefeasibility
once the claim of ownership is established and recognized. Underlying this section and providing its justification is the
If a person purchases a piece of land on the assurance that Regalian doctrine embodied in Section 1 of Article XIII of
the seller‘s title thereto is valid, he should not run the risk the Constitution of 1935, in force during the material dates
of being told later that his acquisition was ineffectual after of the events herein involved, declaring that "all
all. agricultural, timber, and mineral lands of the public domain
. . and other natural resources of the Philippines belong to

Notes of Cesnee Tan Page 52


Natural Resources 1st Exam (2017-2018)

the State...." And under Krivenko vs. Register of Deeds, 79 complaint


Phil. 461, "the scope of this constitutional provision, Basis Fraud/deceit Beyond Director of lands
according to its heading and its language, embraces all Real party State Person whi has a pre-
lands of any kind of the public domain, its purpose being to in interest existing right
establish a permanent and fundamental policy for the Effect of Reconvey Reconvey to rightful owner
conservation and utilization of all natural resources of the Grant back to
Nation." Accordingly, the right to acquire disposable lands government/
from the State through any of the means provided for in public domain
the Public Land Act, Commonwealth Act 141, must
necessarily be subject to the reservation expressly made in Cases:
above quoted Section 91 to the effect that "the statements
made in the application shall be considered as essential Kionisala vs. Dacut
conditions and parts of any ... title ... issued on the basis of (GR No. 147379, 2/27/2002)
such application" and that "any false statement therein or On 7 September 1990 Lot No. 1017 was granted a free
omission of facts altering, changing or modifying the patent to petitioners Heirs of Ambrocio Kionisala under Free
consideration of the facts set forth in such statements, and Patent No. 603393.
any subsequent modification, alteration or change of the And on 13 November 1991 Lot 1015 was bestowed upon
material facts set forth in the application shall ipso facto1 Isabel Kionisala, one of the impleaded heirs of Ambrocio
produce the cancellation of the concession, title or permit Kionisala under Free Patent No. 101311-91-904.
granted." Thereafter, on 19 November 1990 Lot 1017 was registered
under the Torrens system and was issued OCT No. P-19819
It is true that under Section 122 of the Land Registration in petitioners‘ name, while on 5 December 1991 Lot No.
Act, a Torrens title issued on the basis of a free patent or a 1015 was registered in the name of Isabel Kionisala under
homestead patent is as indefeasible as one judicially OCT No. P-20229.
secured. And in repeated previous decisions of this Court On 19 December 1995 Heirs of Dacut filed a complaint for
that indefeasibility has been emphasized by Our holding declaration of nullity of titles, reconveyance and damages
that not even the Government can file an action for against Heirs of Kionisala, with the RTC of Manolo Fortich,
annulment, but at the same time, it has been made clear Bukidnon of the two (2) parcels of land known as Lot No.
that an action for reversion may be instituted by the 1017 and Lot No. 1015 with areas of 117,744 square
Solicitor General, in the name of the Republic of the meters and 69,974 square meters respectively, located in
Philippines.2 It is to the public interest that one who Pongol, Libona, Bukidnon.
succeeds in fraudulently acquiring title to a public land In support of their causes of action for declaration of nullity
should not be allowed to benefit therefrom, and the State of titles and reconveyance, private respondents claimed
should, therefore, have an ever existing authority, thru its absolute ownership of Lot 1015 and 1017 even prior to the
duly authorized officers, to inquire into the circumstances issuance of the corresponding free patents and certificates
surrounding the issuance of any such title, to the end that of title. They further alleged in their complaint –
the Republic, thru the Solicitor General or any other officer • Are absolute and exclusive owners by virtue of
who may be authorized by law, may file the corresponding inheritance from their late father, Honorio Dacut
action for the reversion of the land involved to the public • In actual possession and cultivation of two
domain, subject thereafter to disposal to other qualified parcels of agricultural lands for more than thirty (30) years.
persons in accordance with law. In other words, the • They discovered that the petitioners, without
indefeasibility of a title over land previously public is not a their knowledge and consent, fraudulently applied for
bar to an investigation by the Director of Lands as to how patent the said parcels of land and as a result thereof
such title has been acquired, if the purpose of such certificates of titles had been issued to them
investigation is to determine whether or not fraud had been • That the patents issued to defendants are null
committed in securing such title in order that the and void
appropriate action for reversion may be filed by the • That in the remote possibility that said
Government. certificates of title cannot be declared as null and void,
plaintiffs, being the absolute and exclusive owners of the
Nothing said above, however, should be understood as parcels of land titled by the defendants, are entitled to
holding that the Court has found that the titles of appellees reconveyance
have been in fact fraudulently secured. That matter may be On 7 February 1996 petitioners filed their answer to the
resolved only after the Director of Lands shall have finished complaint and asserted the following affirmative defenses –
his investigation. • That the complaint states no cause of action;
• That the cause of action, if any, is barred by
C. Reversion vs Action for Nullity of land titles statute of limitations, prescription of action or by equitable
principle of laches;
REVERSION CANCELLATION/NULLITY • 10. That x x x it is only the Director of Lands
Allegations State owned Pre-existing right of (now DENR) through the Office of the Solicitor General that
in the ownership/ possession has the authority to file annulment of Free Patent or

Notes of Cesnee Tan Page 53


Natural Resources 1st Exam (2017-2018)

Homestead Patent issued by the Director of Lands or DENR; complaint all the facts necessary to seek the nullification of
On 3 December 1996 the trial court dismissed the the free patents as well as the certificates of title covering
complaint on the ground that the cause of action of private Lot 1015 and Lot 1017. Clearly, they are the real parties in
respondents was truly for reversion so that only the interest in light of their allegations that they have always
Director of Lands could have filed the complaint. been the owners and possessors of the two (2) parcels of
On 23 December 1996 private respondents moved for land even prior to the issuance of the documents of title in
reconsideration of the order of dismissal but on 3 June petitioners‘ favor, hence the latter could only have
1997 the motion was denied by the trial court. committed fraud in securing them.
On 7 June 1997 private respondents appealed the order of It is not essential for private respondents to specifically
dismissal to the Court of Appeals. state in the complaint the actual date when they became
On 15 February 2000 the appellate court reversed the order owners and possessors of Lot 1015 and Lot 1017.
of dismissal. It ruled that while the allegations in the With respect to the purported cause of action for
complaint were insufficient for purposes of an ordinary civil reconveyance, it is settled that in this kind of action the free
action for declaration of nullity of a certificate of title since patent and the certificate of title are respected as
the actual date when private respondents became owners incontrovertible. What is sought instead is the transfer of
of Lots 1015 and 1017 prior to the issuance of the the property, in this case the title thereof, which has been
corresponding free patents and certificates of title was not wrongfully or erroneously registered in the defendant‘s
specifically indicated in the complaint, nonetheless the name. All that must be alleged in the complaint are two (2)
allegations therein were comprehensive enough to facts which admitting them to be true would entitle the
constitute a cause of action for reconveyance. plaintiff to recover title to the disputed land, namely, (1)
Petitioners moved for reconsideration of the CA Decision that the plaintiff was the owner of the land and, (2) that
but the appellate court denied the motion for lack of merit, the defendant had illegally dispossessed him of the
hence this petition for review. same.17
ISSUE: Whether the complaint state a cause of action for The Court ruled that private respondents have sufficiently
declaration of nullity of the free patents and certificates of pleaded (in addition to the cause of action for declaration of
title for Lot 1015 and Lot 1017, or alternatively a cause of free patents and certificates of title) an action for
action for reconveyance of these two lots. reconveyance, more specifically, one which is based on
Whether the cause of action, has prescribed. implied trust. An implied trust arises where the defendant
First issue: The court ruled that the complaint does not allegedly acquires the disputed property through mistake or
allege an action for reversion which private respondents fraud so that he would be bound to hold and reconvey the
would obviously have no right to initiate, but that it property for the benefit of the person who is truly entitled
sufficiently states either a cause of action for declaration of to it.
nullity of free patents and certificates of title over Lot 1015 In the complaint, private respondents clearly assert that
and Lot 1017 or alternatively a cause of action for they have long been the absolute and exclusive owners and
reconveyance of these two pieces of realty, wherein in in actual possession and cultivation of Lot 1015 and Lot
either case private respondents are the real parties in 1017 and that they were fraudulently deprived of ownership
interest. thereof when petitioners obtained free patents and
An ordinary civil action for declaration of nullity of free certificates of title in their names. These allegations
patents and certificates of title is not the same as an action certainly measure up to the requisite statement of facts to
for reversion. The difference between them lies in the constitute an action for reconveyance.
allegations as to the character of ownership of the realty Second. The court ruled that neither the action for
whose title is sought to be nullified. In an action for declaration of nullity of free patents and certificates of title
reversion, the pertinent allegations in the complaint would of Lot 1015 and Lot 1017 nor the action for reconveyance
admit State ownership of the disputed land. On the other based on an implied trust of the same lots has prescribed.
hand, a cause of action for declaration of nullity of free "A free patent issued over private land is null and void, and
patent and certificate of title would require allegations of produces no legal effects whatsoever. Quos nullum est,
the plaintiff‘s ownership of the contested lot prior to the nullum producit effectum."
issuance of such free patent and certificate of title as well Moreover, private respondents‘ claim of open, public,
as the defendant‘s fraud or mistake; as the case may be, in peaceful, continuous and adverse possession of the two (2)
successfully obtaining these documents of title over the parcels of land and its illegal inclusion in the free patents of
parcel of land claimed by plaintiff. In such a case, the nullity petitioners and in their original certificates of title, also
arises strictly not from the fraud or deceit but from the fact amounts to an action for quieting of title which is
that the land is beyond the jurisdiction of the Bureau of imprescriptible.
Lands to bestow and whatever patent or certificate of title The action for reconveyance based on implied trust, on the
obtained therefor is consequently void ab initio. The real other hand, prescribes only after ten (10) years from 1990
party in interest is not the State but the plaintiff who and 1991 when the free patents and the certificates of title
alleges a pre-existing right of ownership over the parcel of over Lot 1017 and Lot 1015, respectively, were registered.
land in question even before the grant of title to the Obviously the action had not prescribed when private
defendant. respondents filed their complaint against petitioners on 19
It is obvious that private respondents allege in their December 1995. At any rate, the action for reconveyance in

Notes of Cesnee Tan Page 54


Natural Resources 1st Exam (2017-2018)

the case at bar is also significantly deemed to be an action titles and to strip them of any probative value as evidence
to quiet title for purposes of determining the prescriptive of ownership. It had clearly set a deadline for the filing of
period on account of private respondents‘ allegations of applications for registration of all Spanish titles under the
actual possession of the disputed lots. In such a case, the Torrens system (i.e., six months from its effectivity or on 14
cause of action is truly imprescriptible. August 1976), after which, the Spanish titles may no longer
In sum, the grounds relied upon in petitioners‘ desire to be presented to prove ownership.
dismiss the complaint of private respondents in Civil Case
No. 95-312 cannot be impressed with merit. By this All holders of Spanish titles should have filed applications
decision, however, the court is not foreclosing the for registration of their title on or before 14 August 1976. In
presentation of evidence during trial on the merits that Lot a land registration proceeding, the applicant should present
1015 and Lot 1017 are not private property and that private to the court his Spanish title plus proof of actual possession
respondents are not truly the owners thereof. This and of the real property. However, if such land registration
other issues on the merits must follow where the proceeding was filed and initiated after 14 August 1976,
preponderant evidence lies. the applicant could no longer present his Spanish title to
WHEREFORE, the instant Petition for Review is DENIED for the court to evidence his ownership of the real property,
lack of merit. The Decision of the Court of Appeals dated 15 regardless of whether the real property was in his actual
February 2000 reversing the Order dismissing the possession.
Complaint in Civil Case No. 95-312 entitled Heirs of Honorio
Dacut, namely, Visaminda Orevillo, Violeta Dacut, Josephine Therefore, the fact that petitioners were in actual
Dacut and Elizabeth Dacut v. Heirs of Ambrocio Kionisala, possession of the Subject Property when they filed the
namely, Ana, Isabel, Grace, Ophelia, Joven and Camilo, all Complaint with the trial court on 29 April 1996 does not
surnamed Kionisala, and Isabel Kionisala is AFFIRMED with exclude them from the application of P.D. No. 892, and
the understanding that private respondents Heirs of their Spanish title remain inadmissible as evidence of their
Honorio Dacut as plaintiffs therein may proceed on the ownership of the Subject Property, whether in a land
basis of their causes of action of declaration of nullity of registration proceeding or in an action to remove a cloud on
free patents and certificates of titles and/or reconveyance or to quiet title.
based on an implied trust, with claim for damages. The
proceedings in the trial court shall commence forthwith The preceding discussion does not bar holders of Spanish
within thirty (30) days from notice of the finality of this titles from claiming ownership of the real property on some
Decision without unnecessary delay. other basis, such as those provided in either the Land
Registration Decree[41] or the Public Land Act.[42]
Evangelista vs. Santiago Petitioners though failed to allege any other basis for their
(GR No. 157447, 4/29/2005) titles in their Complaint aside from possession of the
This Court cannot sustain petitioners argument. Actual Subject Property from time immemorial, which this Court
proof of possession only becomes necessary because, as has already controverted; and the Spanish title, which is
the same whereas clause points out, Spanish titles are already ineffective to prove ownership over the Subject
subject to prescription. A holder of a Spanish title may still Property.
lose his ownership of the real property to the occupant who
actually possesses the same for the required prescriptive Therefore, without legal or equitable title to the Subject
period.[38] Because of this inherent weakness of a Spanish Property, the petitioners lacked the personality to file an
title, the applicant for registration of his Spanish title under action for removal of a cloud on, or quieting of, title and
the Torrens system must also submit proof that he is in their Complaint was properly dismissed for failing to state a
actual possession of the real property, so as to discount the cause of action. In view of the dismissal of the case on this
possibility that someone else has acquired a better title to ground, it is already unnecessary for this Court to address
the same property by virtue of prescription. the issue of prescription of the action.

Moreover, legislative intent must be ascertained from a Wherefore, this Court DENIES the instant petition and
consideration of the statute as a whole, and not just a AFFIRMS the Decision of the Court of Appeals, dated 29
particular provision alone. A word or phrase taken in the July 2002, and the Order of the Regional Trial Court of San
abstract may easily convey a meaning quite different from Mateo, Rizal, Branch 77, dated 05 February 1999,
the one actually intended and evident when the word or dismissing petitioners Complaint for failure to state a cause
phrase is considered with those with which it is associated. of action.
An apparently general provision may have a limited
application if read together with other provisions of the SO ORDERED.
statute.[39]
Roco vs. Gimeda
The fourth whereas clause of P.D. No. 892 should be (GR No. L-11651, 12/27/1958)
interpreted and harmonized with the other provisions of the Appeal from a judgment of the Court of First Instance of
whole statute.[40] Note that the tenor of the whole Cebu, Hon. Jose S. Rodriguez, presiding, dismissing the
presidential decree is to discontinue the use of Spanish complaint upon petition of defendants, on the ground that

Notes of Cesnee Tan Page 55


Natural Resources 1st Exam (2017-2018)

it fails to state a cause of action. It is to be noted that the petition does not seek for a
reconsideration of the granting of the patent or of the
The complaint makes the following allegations: that before decree issued in the registration proceeding. The purpose is
August 22, 1918, Espiridiona Caramihan, owned and not to annul the title but to have it conveyed to plaintiffs.
possessed two parcels of land known as lots Nos. 2741 and Fraudulent statements were made in the application for the
3082 of the Barili Cadastral Survey No. 219, covered by tax patent and no notice thereof was given to plaintiffs, nor
declarations Nos. 01865 and 01854; that upon the death of knowledge of the petition known to the actual possessors
said Espiridiona Caramihan on August 22, 1918, said lands and occupants of the property. The action is one based on
were partitioned equally among her children, who similarly fraud and under the law, it can be instituted within four
possessed and cultivated their respective shares and paid years from the discovery of the fraud. (Art. 1146, Civil
the taxes thereon; that in the years 1925 to 1927, through Code, as based on Section 3, paragraph 43 of Act No. 190.)
ignorance and inadvertence of the heirs, the said lots were It is to be noted that as the patent here has already been
declared public land in a cadastral proceeding; that issued, the land has the character of registered property in
Espiridiona occupied said lands openly, adversely, accordance with the provisions of Section 122 of Act No.
continuously and publicly, planting coconut and fruit trees 496, as amended by Act No. 2332, and the remedy of the
and building her dwelling house thereon, and that said party who has been injured by the fraudulent registration is
improvements and house are still on said lots; that the an action for reconveyance. (Director of Lands vs. Register
present plaintiffs acquired their rights to the lots by of Deeds, 92 Phil., 826; 49 Off. Gaz. [3] 935; Section 55 of
purchase from the heirs of the original owner Esperidiona Act No. 496.)
Caramihan; that on or about December 7, 1940, Juan
Gimeda, defendant, filed an application for a free patent to The order of dismissal appealed from is, therefore, reversed
said lands, surreptitiously and fraudulently, without and the case is returned to the court a quo for further
knowledge of the owners and possessors, and on proceedings in accordance with law.
December 7, 1940, the Director of Lands issued an order
and in accordance therewith, on September 17, 1951, the Republic v. Hachero,
Bureau of Lands issued patent No. 51552 in the name of G.R. No. 200973, [May 30, 2016])
defendant Juan Gimeda; that the plaintiffs and their original Facts:
predecessor-in-interest have always been in the actual, In 1996, Amor Hachero (Hachero) filed his Free Patent
physical, continuous and uninterrupted possession of the Application No. 045307-969 covering Lot No. 1514, CAD-
said parcels of land and defendant Juan Gimeda applied for 1150-D (subject land) before the Community Environment
and obtained his patent thereto without notice to them and and Natural Resources Office (CENRO) of Palawan. The
without their knowledge, and secured the approval of his said application for free patent was later approved by the
patent by fraudulent statements, alleging that he was the Provincial Environment and Natural Resources Officer
only heir of Espiridiona Caramihan and the only occupant of (PENRO) of Palawan based on the following findings,
the land; and that by such false and fraudulent statements among others that the land applied for had been classified
the Bureau of Lands approved his application and ordered as alienable and disposable and, therefore, subject to
the issuance of his patent. disposition under the Public Land Law;
On October 15, 1998, Free Patent No. 045307-98-9384 was
The defendant filed an answer to the complaint, then issued to Hachero and the subject land was registered
amended the said answer and alleges that he is the under Original Certificate of Title (OCT) No. E-18011 on
youngest among the children of Espiridiona Caramihan; May 7, 1999.
denies the allegations made in the complaint as to the After an inspection and verification were conducted by the
acquisition by false and fraudulent means of the said lands; CENRO in 2000, it was discovered that the subject land,
alleges that the complaint states no cause of action. He covered by OCT No. E-18011, was still classified as
presents a counterclaim for P5,000 and P10,000 as moral timberland and so not susceptible of private ownership
and exemplary damages, respectively, and P500 as under the Free Patent provision of the Public Land Act.
attorney's fees. Plaintiffs deny this counterclaim. Hence, the Republic filed the Complaint for the Cancellation
of Free Patent No. 045307-98-9384 and OCT No. E-18011
Later on defendant presented a motion to dismiss, alleging and for Reversion, which was docketed as Civil Case No.
that the complaint alleges no cause of action, arguing that 3726.
as the title in his favor was issued on October 17, 1951 and The RTC rendered its decision in favor of Hachero on the
action was filed on July 15, 1954, the action was filed more ground that the free patent and title had already been
than two years after the issuance of the patent, beyond the issued after Hachero was found to have complied with all
one-year period provided by law. The authorities cited for the requirements; that it was the Republic itself thru the
this defense are the case of Director of Lands vs. Gutierrez DENR-CENRO, Coron, which brought the subject land under
David, 50 Phil, 797; Villarosa vs. Sarmiento, 46 Phil., 814; the operation of the Torrens System. The CA affirmed the
Cabanos vs. Register of Deeds, 40 Phil., 620; Sumcad vs. RTC decision.
Judge of the Court of First Instance, et al., 96 Phil., 946; 51 Issue:
Off. Gaz., [5] 2413. 1. Won the petition for cancellation of the title
and reversion of the subject lot is meritorious.

Notes of Cesnee Tan Page 56


Natural Resources 1st Exam (2017-2018)

2. WON the Government is estopped by its contemplation of the Public Land Law."
previous acts. But then again, the Court had several times in the past
recognized the right of the State to avail itself of the
Ruling: remedy of reversion in other instances when the title to the
1. Cancellation of title and reversion proper land is void for reasons other than having been secured by
where there exists a mistake or oversight in granting free fraud or misrepresentation.
patent over inalienable land Be that as it may, the mistake or error of the officials or
agents of the BOL in this regard cannot be invoked against
Reversion is an action where the ultimate relief sought is to the government with regard to property of the public
revert the land back to the government under the Regalian domain. It has been said that the State cannot be estopped
doctrine. Considering that the land subject of the action by the omission, mistake or error of its officials or agents.
originated from a grant by the government, its cancellation
therefore is a matter between the grantor and the grantee.
xxx. In Estate of the Late Jesus S. Yujuico v. Republic Republic v. Espinosa,
(Yujuico case), reversion was defined as an action which G.R. No. 186603, [April 5, 2017])
seeks to restore public land fraudulently awarded and The State failed to prove that the property was classified as
disposed of to private individuals or corporations to the forest land at the time of the grant of the cadastral decree
mass of public domain. It bears to point out, though, that and issuance of title to Espinosa.
the Court also allowed the resort by the Government to
actions for reversion to cancel titles that were void for In land registration proceedings, the applicant has the
reasons other than fraud, i.e., violation by the grantee of a burden of overcoming the presumption of State ownership.
patent of the conditions imposed by law; and lack of It must establish, through incontrovertible evidence, that
jurisdiction of the Director of Lands to grant a patent the land sought to be registered is alienable or disposable
covering inalienable forest land or portion of a river, even based on a positive act of the government. 30 Since
when such grant was made through mere oversight.d cadastral proceedings are governed by the usual rules of
In the case at bench, although the Republic's action for practice, procedure, and evidence, a cadastral decree and a
cancellation of patent and title and for reversion was not certificate of title are issued only after the applicant proves
based on fraud or misrepresentation on the part of all the requisite jurisdictional facts-that they are entitled to
Hachero, his title could still be cancelled and the subject the claimed lot, that all parties are heard, and that evidence
land reverted back to the State because the grant was is considered.31 As such, the cadastral decree is a
made through mistake or oversight. This could probably be judgment which adjudicates ownership after proving these
the reason why, shortly after one (1) year from the jurisdictional facts.32
issuance of OCT No. E-18011 to Hachero, the DENR
personnel conducted another investigation and verification Here, it is undisputed that Espinosa was granted a cadastral
on the subject land. It would appear that they suspected decree and was subsequently issued OCT No. 191-N, the
that a mistake was made in their issuance of the patent as predecessor title of Caliston's TCT No. 91117. Having been
the subject land had not been reclassified or released as granted a decree in a cadastral proceeding, Espinosa can
alienable or disposable land. It remained plotted within the be presumed to have overcome the presumption that the
timberland classification zone. land sought to be registered forms part of the public
domain. 33 This means that Espinosa, as the applicant, was
2. Prescription and estoppel cannot able to prove by incontrovertible evidence that the property
lie against the State is alienable and disposable property in the cadastral
proceedings.

At any rate, it is a time-honored principle that the statute of This is not to say, however, that the State has no remedy to
limitations or the lapse of time does not run against the recover the property if indeed it is part of the inalienable
State. Jurisprudence also recognizes the State's immunity lands of the public domain. The State may still do so
from estoppel as a result of the mistakes or errors of its through an action for reversion, as in the present case.
officials and agents.
It must be emphasized that a certificate of title issued Reversion is the remedy where the State, pursuant to the
under an administrative proceeding pursuant to a Regalian doctrine, seeks to revert land back to the mass of
homestead patent, as in the instant case, is as indefeasible the public domain. 34 It is proper when public land is
as a certificate of title issued under a judicial registration fraudulently awarded and disposed of to private individuals
proceeding, provided the land covered by said certificate is or corporations.35 There are also instances when we
a disposable public land within the contemplation of the granted reversion on grounds other than fraud, such as
Public Land Law. Yet, we emphasize that our statement in when a "person obtains a title under the Public Land Act
the aforequoted case that a certificate of title issued which includes, by oversight, lands which cannot be
pursuant to a homestead patent becomes indefeasible after registered under the Torrens system, or when the Director
one year, is subject to the proviso that "the land covered by of Lands did not have jurisdiction over the same because it
said certificate is a disposable public land within the is of the public domain."36

Notes of Cesnee Tan Page 57


Natural Resources 1st Exam (2017-2018)

case, the State offered in evidence a land classification map


In this case, the State, through the Solicitor General, to prove that at the time the land was decreed to the
alleges neither fraud nor misrepresentation in the cadastral original owner, it had not yet been released and still fell
proceedings and in the issuance of the title in Espinosa's within the forest zone. However, the map did not
favor. The argument for the State is merely that the conclusively state the actual classification of the land at the
property was unlawfully included in the certificate of title time it was adjudicated to the original owner. We thus ruled
because it is of the public domain. that the State failed to prove that the titles should be
annulled- Finally, we find the need to emphasize that in an
Since the case is one for reversion and not one for land action to annul a judgment, the burden of proving the
registration, the burden is on the State to prove that the judgment's nullity rests upon the petitioner. The petitioner
property was classified as timberland or forest land at the must establish by clear and convincing evidence that the
time it was decreed to Espinosa.37 To reiterate, there is no judgment is fatally defective. When the proceedings were
burden on Caliston to prove that the property in question is originally filed by the Republic before the Court of Appeals,
alienable and disposable land. 38 At this stage, it is the petitioner contended that when the decree in favor of
reasonable to presume that Espinosa, from whom Caliston De Perio was issued by Judge Ostrand in 1912 the parcels
derived her title, had already established that the property of land were still part of the inalienable public forests.
is alienable and disposable land considering that she However, petitioner's case rested solely on land
succeeded in obtaining the OCT over it.39 In this reversion classification maps drawn several years after the issuance
proceeding, the State must prove that there was an of the decree in 1912. These maps fail to conclusively
oversight or mistake in the inclusion of the property in establish the actual classification of the land in 1912 and
Espinosa' s title because it was of public dominion. This is the years prior to that. Before this Court, petitioner
consistent with the rule that the burden of proof rests on reiterates said contention and refers, for the first time, to a
the party who, as determined by the pleadings or the 1908 proclamation reserving the land in Zambales as a
nature of the case, asserts the affirmative of an issue.40 naval reservation and alleging that the subject parcels of
land are parts thereof. These, for reasons discussed earlier,
Here, the State hinges its whole claim on its lone piece of are insufficient to overcome the legal presumption in favor
evidence, the land classification map prepared in 1986. of the decree's regularity, more so when we consider that
notice of the application for registration and the date of
The records show, however, that LC Map No. 2978 was not hearing thereof, addressed to the Attorney General, the
formally offered in evidence. The rules require that Director of Lands, the Director of Public Works and the
documentary evidence must be formally offered in evidence Director of Forestry, among others, was published in the
after the presentation of testimonial evidence, and it may Official Gazette and that Governor General Smith's
be done orally, or if allowed by the court, in writing.41 Due Proclamation of 1908 itself recognizes private rights.48
process requires a formal offer of evidence for the benefit
of the adverse party, the trial court, and the appellate We stress that our ruling is not inconsistent with the
courts.42 This gives the adverse party the opportunity to doctrine that forest lands are outside the commerce of man
examine and oppose the admissibility of the evidence.43 and unsusceptible of private appropriation.1âwphi1 Neither
When evidence has not been formally offered, it should not are we changing the rule on imprescriptibility of actions for
be considered by the court in arriving at its decision.44 Not reversion. We are merely deciding on the facts as proved by
having been offered formally, it was error for the trial court the record. To allow a reversion based on a classification
to have considered the survey map. Consequently, it also made at the time when the property was already declared
erred in ordering the reversion of the property to the mass private property by virtue of a decree would be akin to
of the public domain on the basis of the same. expropriation of land without due process of law.49

Moreover, even assuming that the survey can be admitted At this juncture, we agree with the CA' s application of
in evidence, this will not help to further the State's SAAD AgroIndustries, Inc., 50 which involved a complaint
cause.1âwphi1 This is because the only fact proved by the for annulment of title and reversion of a lot covered by a
map is one already admitted by the State, that is, that the free patent and original title. To support its claim that the
land was reclassified in 1986.45 This fact does not address lot was part of the timberland and forest reserve, the State
the presumption/conclusion that Espinosa has, at the time submitted as evidence a photocopy of a land classification
of the cadastral proceedings conducted in 1955, proved map. This map also became the basis of the testimonies of
that the land is alienable and disposable, as evidenced by City Environment and Natural Resources Office officers
the decree issued in his favor in 1962. declaring that the lot falls within the timberland or forest
reserve. The State, however, failed to submit either a
II certified true copy or an official publication of the map,
prompting the trial court to deny its admission in evidence.
The reclassification of the area where the property is After proceedings, the trial court dismissed the complaint
located in 1986 should not prejudice Espinosa and her due to the State's failure to show that the subject lot
successor-in-interest.46 Apropos is the case of Sta. Monica therein is part of the timberland or forest reserve or has
Industrial and Dev't Corp. v. Court of Appeals. 47 In that been classified as such before the issuance of the free

Notes of Cesnee Tan Page 58


Natural Resources 1st Exam (2017-2018)

patent and the original title. The CA, relying on the map, Constitutions. The Court has always recognized and upheld
reversed the trial court. the Regalian doctrine as the basic foundation of the State's
property regime. Nevertheless, in applying this doctrine, we
When the case was brought before this court, we reinstated must not lose sight of the fact that in every claim or right
the trial court's decision. We held that the photocopy of the by the Government against one of its citizens, the
land classification map cannot be considered in evidence paramount considerations of fairness and due process must
because it is excluded under the best evidence rule. We be observed. Respondent in this case failed to show that
emphasized that all parties, including the Government, are the subject lot is part of timberland or forest reserve it
bound by the rules of admissibility and must comply with it- adverted to. In the face of the uncontroverted status of
The rules of admissibility must be applied uniformly. The Free Patent No. 473408 and OCT No. 0-6667 as valid and
same rule holds true when the Government is one of the regular issuances, respondent's insistence on the
parties. The Government, when it comes to court to litigate classification of the lot as part of the forest reserve must be
with one of its citizens, must submit to the rules of rejected. 52
procedure and its rights and privileges at every stage of the
proceedings are substantially in every respect the same as These principles laid down in SAAD Agro-Industries, Inc.
those of its citizens; it cannot have a superior advantage. undoubtedly apply here. As part of fair play and due
This is so because when a [sovereign] submits itself to the process, the State is as bound by the rules on formal offer
jurisdiction of the court and participates therein, its claims of evidence as much as every private party is. More, the
and rights are justiciable by every other principle and rule State's subsequent reclassification of the area where the
applicable to the claims and rights of the private parties property is situated cannot be used to defeat the rights of a
under similar circumstances. Failure to abide by the rules private citizen who acquired the land in a valid and regular
on admissibility renders the L.C. Map submitted by proceeding conducted 24 years earlier.
respondent inadmissible as proof to show that the subject
lot is part of the forest reserve. 51 The result would have been different had the State proved
that the property was already classified as part of forest
We went on to explain that even if the map was admitted in land at the time of the cadastral proceedings and when title
evidence to prove that the lot was classified as part of the was decreed to Espinosa in 1962. However, it failed to
timberland or forest reserve, the classification was made discharge this burden; the grant of title which carries with it
long after private interests had intervened. Not only was the presumption that Espinosa had already proved the
the lot already occupied and cultivated, a free patent and a alienable character of the property in the cadastral
certificate of title were also awarded and issued years proceedings stands. To grant the reversion based on a
ahead of the classification- Even assuming that the L.C. subsequent reclassification, more so on lack of evidence,
Map submitted by respondent is admissible in evidence, still would amount to taking of private property without just
the land in question can hardly be considered part of the compensation and due process of law.53 This, however, is
timberland or forest reserve. L.C. Map No. 2961, which not what our Constitution envisions; fairness and due
purports to be the "correct map of the areas demarcated as process are paramount considerations that must still be
permanent forest pursuant of the provisions of P.D. No. 705 observed.54
as amended" was made only in 1980. Thus, the delineation
of the areas was made nine (9) years after Orcullo was
awarded the free patent over the subject lot.

xxx

Obviously, private interests have intervened before


classification was made pursuant to P.D. No. 705. Not only
has Orcullo by herself and through her predecessors-
ininterest cultivated and possessed the subject lot since
1930, a free patent was also awarded to her and a title
issued in her name as early as 1971. In fact, it appears that
the issuance of the free patent and certificate of title was
regular and in order. Orcullo complied with the requisites
for the acquisition of free patent provided under
Commonwealth Act No. 141 (Public Land Act), as certified
by the Director of Lands and approved by the Secretary of
Agriculture and Natural Resources.

xxx

The Regalian doctrine is well-enshrined not only in the


present Constitution, but also in the 1935 and 1973

Notes of Cesnee Tan Page 59

También podría gustarte